Head and Neck Flashcards

1
Q

A patient presents with an enlarged lymph node in the anterior compartment of the neck. An FNAB returns with a histologic diagnosis of epidermoid carcinoma. CT and panendoscopy fail to reveal a primary lesions. Which of the following is the MOST appropriate course of action?

A. Observation with expectant management

B. Excisional biopsy to confirm the diagnosis

C. Radiation therapy directed at the oropharynx and neck bilaterally

D. Chemotherapy combined with radiation therapy directed at the ipsilateral neck

E. Modified radical neck dissection followed by adjuvant radiation therapy directed at the ipsilateral neck

A

C. Radiation therapy directed at the oropharynx and neck bilaterally

See NCCN guidelines for occult lesions

How well did you know this?
1
Not at all
2
3
4
5
Perfectly
2
Q

A 75-year-old male pipe smoker presents with a 2.0cm ulcerated lesion along the vermillion border of his lower lip on the right side. There is no palpable adenopathy appreciated on physical exam. Computed tomography and panendoscopy are otherwise unrevealing. What is the MOST appropriate intervention?

A. Wide local excision only

B. Wide local excision with sentinel lymph node biopsy

C. Wide local excision with selective neck dissection containing levels I, II, and III

D. Wide local excision with modified radical neck dissection

E. Wide local excision with adjuvant radiation therapy

A

A. Wide local excision only

Lip cancer is very low for nodal mets.

How well did you know this?
1
Not at all
2
3
4
5
Perfectly
3
Q

You are called to evaluate a 67-year-old female who is postoperative day 14 from a tracheostomy. The patient’s nurse tells you that a few moments ago, she suctioned out a large amount of clotted blood from inside the patient’s tracheostomy and upper airway. Since then she has not noticed any further bleeding but is requesting that you evaluate the patient as soon as possible. Which of the following is the MOST likely source of bleeding?

A. Fistule between the trachea and carotid artery

B. Fistula between the trachea and innominate artery

C. Fistula between the trachea and esophagus

D. Bleeding from the skin edge of the tracheostomy site

E. Upper airway inflammation

A

B. Fistula between the trachea and innominate artery

How well did you know this?
1
Not at all
2
3
4
5
Perfectly
4
Q

Which of the following is the MOST appropriate next step for suspected (controlled) bleeding post-tracheostomy?

A. Overinflation of tracheostomy cuff

B. Digital compression of the bleeding source against the manubrium

C. Bronchoscopy

D. Digitial compression of the ipsilateral carotid artery

E. Immediate primary repair of the bleeding vessel in the operating room

A

A. Overinflation of tracheostomy cuff

If it is rebleeding/active

How well did you know this?
1
Not at all
2
3
4
5
Perfectly
5
Q

A 65-year-old woman comes to the office because she has swelling of the left side of her face with left-sided facial weakness. On examination, a left parotid mass is palpated. Fine-needle aspiration biopsy of the specimen shows few malignant epithelial cells. Which of the following is the MOST likely diagnosis?

A. Adenocarcinoma

B. Adenoid cystic carcinoma

C. Carcinoma ex pleomorphic adenoma

D. Mucoepidermoid carcinoma

E. Squamous cell carcinoma

A

D. Mucoepidermoid carcinoma

How well did you know this?
1
Not at all
2
3
4
5
Perfectly
6
Q

A 61-year-old male has a biopsy performed of a new mass located at the angle of the mandible. The results describe a low grade acinic cell carcinoma of the parotid gland. During the resection, the mass is contained in the superficial lobe and is adjacent to the facial nerve.

Which of the following is the BEST treatment for the patient?

A. Superficial parotidectomy with facial nerve preservation

B. Total parotidectomy with radical neck dissection

C. Superficial parotidectomy and adjuvant radiation therapy

D. Total parotidectomy with adjuvant radiation therapy

E. En bloc parotidectomy including resection of facial nerve

A

C. Superficial parotidectomy and adjuvant radiation therapy

How well did you know this?
1
Not at all
2
3
4
5
Perfectly
7
Q

Which of the following is an indication for a radical neck dissection for a parotid gland tumor?

A. Warthin’s tumor involving both parotid glands

B. High grade mucoepidermoid lesion with no palpable neck nodes

C. Pleomorphic adenoma with malignant transformation

D. Low grade adenoid cystic carcinoma with neural invasion

E. Basal cell adenoma

A

B. High grade mucoepidermoid lesion with no palpable neck nodes

How well did you know this?
1
Not at all
2
3
4
5
Perfectly
8
Q

Which of the following is the most common site of minor salivary gland malignancies?

A. Buccal mucosa

B. Floor of the mouth

C. Lip

D. Palate

E. Tongue

A

D. Palate

How well did you know this?
1
Not at all
2
3
4
5
Perfectly
9
Q

What are the borders of the posterior cervical triangle?

A. SCM, trapezius, middle third of clavicle

B. Midline, hyoid, anterior digastric

C. Mandible, SCM, midline

D. SCM, clavicle, mandible

E. SCM, trapezius, mandible

A

A. SCM, trapezius, middle third of clavicle

How well did you know this?
1
Not at all
2
3
4
5
Perfectly
10
Q

Which of the following are true about leukoplakia of the vocal cords?

A. Up to 40% risk of progression to invasive carcinoma.

B. Ulceration is particularly suggestive of possible
malignancy.

C. Initial therapy includes antihistamines.

D. Biopsy should be considered only after 6 months of conservative therapy.

A

Answer: B

Leukoplakia of the vocal fold represents a white patch (which cannot be wiped off) on the mucosal surface, usually on the superior surface of the true vocal cord.

Rather than a diagnosis per se, the term leukoplakia describes a finding on laryngoscopic examination. The significance of this finding is that it may represent squamous hyperplasia, dysplasia, and/or car- cinoma.

Lesions exhibiting hyperplasia have a 1 to 3% risk of progression to malignancy. In contrast, that risk is 10 to 30% for those demonstrating dysplasia.

Furthermore, leukoplakia may be observed in association with inflammatory and reactive pathologies, including polyps, nodules, cysts, granulomas, and papillomas. Features of ulceration and erythroplasia are particularly suggestive of possible malignancy.

A history of smoking and alcohol abuse should also prompt a malignancy workup.

In the absence of suspected malignancy, conservative measures are used for 1 month. Any lesions that progress, persist, or recur should be considered for excisional biopsy specimen.

(See Schwartz 10th ed., p. 573.)

How well did you know this?
1
Not at all
2
3
4
5
Perfectly
11
Q

Factors associated with increased incidence of head and neck cancers include all of the following EXCEPT

A. Human papillomavirus (HPV) exposure

B. Ultraviolet light exposure

C. Plummer-Vinson syndrome

D. Reflux esophagitis

A

Answer: D

Human papillomavirus (HPV) is an epitheliotropic virus that has been detected to various degrees within samples of oral cavity squamous cell carcinoma.

Infection alone is not considered sufficient for malignant conversion; however, results of multiple studies suggest a role of HPV in a subset of head and neck squamous cell carcinoma.

Multiple reports reflect that up to 40 to 60% of current diagnoses of tonsillar carcinoma demonstrate evidence of HPV types 16 or 18.

Environmental ultraviolet light exposure has been associated with the development of lip cancer. The projection of the lower lip, as it relates to this solar exposure, has been used to explain why the majority of squamous cell carcinomas arise along the vermilion border of the lower lip.

In addition, pipe smoking also has been associated with the development of lip carcinoma.

Factors such as mechanical irritation, thermal injury, and chemical exposure have been described as an explanation for this finding.

Other entities associated with oral malignancy include Plummer-Vinson syndrome (achlorhydria, iron-deficiency anemia, mucosal atrophy of mouth, pharynx, and esophagus), chronic infection with syphilis, and immunocompromised status (30-fold increase with renal transplant).

(See Schwartz 10th ed., p. 579.)

How well did you know this?
1
Not at all
2
3
4
5
Perfectly
12
Q

Features of oral tongue carcinoma include all of the following EXCEPT

A. Presentation as ulcerated exophytic mass

B. May involve submandibular and upper cervical lymph nodes

C. Can result in contralateral paresthesias

D. CO2 laser useful for excision of small early tumors

A

Answer: C

Tumors of the tongue begin in the stratified epithelium of the surface and eventually invade into the deeper muscular structures.

The tumors may present as ulcerations or as exophytic masses. The regional lymphatics of the oral cavity are to the submandibular space and the upper cervical lymph nodes.

The lingual nerve and the hypoglossal nerve may be directly invaded by locally extensive tumors. Involvement can result in ipsilateral paresthesias and deviation of the tongue on protrusion with fasciculations and eventual atrophy.

Tumors on the tongue may occur on any surface, but are most commonly seen on the lateral and ventral surfaces.

Primary tumors of the mesenchymal components of the tongue include leiomyomas, leiomyosarcomas, rhabdomyosarcomas, and neurofibromas.

Surgical treatment of small (T1-T2) primary tumors is wide local excision with either primary closure or healing by sec- ondary intention.

The CO2 laser may be used for excision.

(See Schwartz 10th ed., p. 583.)

How well did you know this?
1
Not at all
2
3
4
5
Perfectly
13
Q

Branchial cleft cysts, if enlarged, should be removed because of which of the following

A. Prone to becoming secondarily infected

B. Prone to cause acute airway obstructions

C. Possible premalignant concerns

D. Association with severe halitosis

A

Answer: A

Congenital branchial cleft remnants are derived rom the branchial cleft apparatus that persists after fetal development.

There are several types, numbered according to their corresponding embryologic branchial cleft.

First branchial cleft cysts and sinuses are associated intimately with the external auditory canal (EAC) and the parotid gland.

Second and third branchial cleft cysts are found along the anterior border of the sternocleidomastoid (SCM) muscle and can produce drainage via a sinus tract to the neck skin.

Secondary infections can occur, producing enlargement, cellulitis, and neck abscess that requires operative drainage.

(See Schwartz 10th ed., p. 598.)

How well did you know this?
1
Not at all
2
3
4
5
Perfectly
14
Q

All of the following are FALSE about salivary gland neoplasms EXCEPT

A. Account or less than 2% of all head and neck neoplasms

B. If in minor salivary glands, less likely to be malignant than if in the parotid gland

C. Computed tomography (CT) scanning is more accurate than magnetic resonance imaging (MRI) in detecting lesions

D. Oncocytomas are usually malignant

A

Answer: A

Tumors of the salivary gland are relatively uncommon and represent less than 2% of all head and neck neoplasms.

About 85% of salivary gland neoplasms arise within the parotid gland. The majority of these neoplasms are benign, with the most common histology being pleomorphic adenoma (benign mixed tumor).

In contrast, approximately 50% of tumors arising in the submandibular and sublingual glands are malignant.

Tumors arising from minor salivary gland tissue carry an even higher risk for malignancy (75%).

Diagnostic imaging is standard for the evaluation of salivary gland tumors. Magnetic resonance imaging (MRI) is the most sensitive study to determine soft tissue extension and involvement of adjacent structures. Benign epithelial tumors include pleomorphic adenoma (80%), monomorphic adenoma, Warthin tumor, oncocytoma, or sebaceous neoplasm.

(See Schwartz 10th ed., p. 599.)

How well did you know this?
1
Not at all
2
3
4
5
Perfectly
15
Q

All of the following are true about tracheostomy EXCEPT

A. Should be performed in patients anticipated to be intubated more than 2 weeks

B. Improves patient discomfort as compared to long term
oropharyngeal intubation

C. Usually spontaneously close within 2 months of removal

D. Does not obligate patient to loss of speech

A

Answer: C

The avoidance of prolonged orotracheal and nasotracheal intubation decreases the risk of laryngeal and subglottic injury and potential stenosis, facilitates oral and pulmonary suctioning, and decreases patient’s discomfort.

When the tracheostomy is no longer needed, the tube is removed and closure of the opening usually occurs spontaneously over a 2-week period.

Placement of a tracheostomy does not obligate a patient to loss of speech.

When a large cuffed tracheostomy tube is in place, expecting a patient to be capable of normal speech is impractical. However, after a patient is downsized to an uncuffed tracheostomy tube, intermittent finger occlusion or Passy-Muir valve placement will allow a patient to communicate while still using the tracheostomy to bypass the upper airway.

(See Schwartz 10th ed., p. 602.)

How well did you know this?
1
Not at all
2
3
4
5
Perfectly
16
Q

Discuss the nodal levels.

A

Level I: Submandibular and submental triangles

Level II-IV: Jugular chain of nodes

Level V: Posterior triangle

Level VI: Central compartment (located at the anterior neck area, inferior to the thyroid gland)

How well did you know this?
1
Not at all
2
3
4
5
Perfectly
17
Q

Management of patients with nodal SCCA, with unknown primary?

A

Do Radical Neck Dissection of the involved side, and postoperative radiation from the nasopharynx to the entire neck.

How well did you know this?
1
Not at all
2
3
4
5
Perfectly
18
Q

Types of Neck Dissections?

A

1) Radical Neck Dissection
Removal of all lymph-bearing tissues in the neck, from levels I-V, together with the SCM, IJV, and SAN.

2) Modified Radical Neck Dissection
Removal of all lymph bearing tissues in the neck from levels I-V, with preservation of the SCM, IJV, and SAN.

3) Extended Radical Neck Dissection
Same with RND, plus removal of other tissues (strap muscles, trachea, etc.)

4) Selective Neck Dissection*
* The first three types are used in staging the neck (clinical N0)
a. Supraomohyoid Neck Dissection (SOHND)
- Removal of levels I-III
- For oral cavity, facial, salivary gland lesions

b. Lateral Neck Dissection
- Removal of levels II-IV
- For pharyngeal and laryngeal lesions

c. Central Node Compartment Dissection
- Removal of level VI
- For thyroid CA cases wherein a total thyroidectomy is performed

d. Posterolateral Neck Dissection
- Removal of level II-V together with the occipital and posterior auricular nodes
- Scalp lesions

How well did you know this?
1
Not at all
2
3
4
5
Perfectly
19
Q

Most common cancer types in the scalp?

A

1) SCCA
2) Basal Cell CA
3) Soft tissue sarcoma

How well did you know this?
1
Not at all
2
3
4
5
Perfectly
20
Q

Surgical margins for excision?

A

SCCA = 1cm

Basal Cell CA = 1cm

How well did you know this?
1
Not at all
2
3
4
5
Perfectly
21
Q

What diagnostic tools are needed for scalp lesions suspicious for cancer?

A

If movable, no need for any labs.

If fixed, do at least a skull x-ray.

How well did you know this?
1
Not at all
2
3
4
5
Perfectly
22
Q

How do you confirm the diagnosis for scalp lesions suspicious for cancer?

A

SCCA: Punch or incision biopsy

Basal Cell CA and Soft Tissue Sarcoma: Tru-cut biopsy

How well did you know this?
1
Not at all
2
3
4
5
Perfectly
23
Q

How do you assess the neck for scalp lesions?

A

SCCA: Drains through lymphatics hence neck dissection is needed.

  • For lesions in the occipital area, drainage would be to the occipital and posterior auricular nodes.
  • For lesions in the temporal, parietal and frontal areas, drainage would be to the peri-parotid nodes.

Basal cell CA and sarcomas generally do not metastasize through lymphatics, hence there is no need for a neck dissection.

How well did you know this?
1
Not at all
2
3
4
5
Perfectly
24
Q

How do you treat SCCA of the scalp?

A

Wide excision with 1cm margins.

If the mass is movable, leave the periosteum intact. Reconstruct with skin graft.

If the mass is fixed, check for bony involvement. An outer table craniectomy may be needed, or in less invasive types, removal of the periosteum is required. Reconstruct with rotational scalp flap plus skin graft to donor site.

For the neck:

  • Occipital lesions: Posterolateral neck dissection
  • Temporal, parietal, and frontal lesions: parotidectomy and neck dissection
How well did you know this?
1
Not at all
2
3
4
5
Perfectly
25
Q

What is the 2/3 rule for salivary gland tumors?

A

2/3 of all lesions in the salivary glands would be benign.

2/3 of these lesions would be a pleomorphic adenoma.

2/3 of these will be located in the parotid gland.

How well did you know this?
1
Not at all
2
3
4
5
Perfectly
26
Q

What are the major salivary glands?

A

Parotid
Submandibular
Sublingual

How well did you know this?
1
Not at all
2
3
4
5
Perfectly
27
Q

How many minor salivary glands does the oral cavity have?

A

700 - 1000 minor salivary glands

How well did you know this?
1
Not at all
2
3
4
5
Perfectly
28
Q

What is the treatment for benign salivary gland lesions?

A

Parotid

  • If mass is big enough to extend beyond the facial nerve, do a total parotidectomy. Otherwise, do a superficial parotidectomy.
  • A pleomorphic adenoma has a pseudocapsule. There are finger-like projections extending beyond the visible capsule. Hence if there is no significant margin during resection, there is a very high recurrence rate.

Submandibular gland
- Once a tumor grows, remove the entire submandibular gland.

How well did you know this?
1
Not at all
2
3
4
5
Perfectly
29
Q

What is the treatment for malignant lesions of the salivary glands?

A

No need to do biopsy. Basis for diagnosis would be clinical– look for:

  • fixation/mobility
  • consistency (hard)
  • facial nerve involvement
  • nodal involvement
  • skin involvement
  • extension to the oral cavity.

Determine two things: size and grade.

Only 3 types of malignancies have a low-grade and a high-grade component:

1) Mucoepidermoid carcinoma
2) Acinic cell carcinoma
3) Adenocarcinoma

T1: ≤2cm
T2: >2cm, but ≤4cm
T3: >4cm
T4: Extension to adjacent structures

For T1-T2 (low-grade): Superficial parotidectomy or submandibular gland excision

T3-T4 (low-grade), any T, high grade lesions with no clinically palpable neck nodes (N0):
Total Parotidectomy + SOHND + Postoperative RT to the wall

*Remember that submandibular gland removal is part of level I dissection.

How well did you know this?
1
Not at all
2
3
4
5
Perfectly
30
Q

How to determine the histologic type of malignancy?

A

You could opt to do frozen section.

How well did you know this?
1
Not at all
2
3
4
5
Perfectly
31
Q

What are the complications of surgery for the salivary gland tumors?

A

Submandibular Gland Excision

  • injury to the nerve causing loss of taste (chorda tympanii of facial nerve CN VII) and sensation (lingual nerve of trigeminal CN V3)
  • injury to the marginal mandibular branch of the facial nerve.

Parotidectomy

  • Frey’ syndrome: aka auriculotemporal syndrome, characterized by gustatory sweating and flushing. Diagnosed by a minor starch-iodine test. The starch-iodine test consists of painting the patient’s postsurgical affected region with iodine, in which the starch turns blue/brown in the presence of iodine and sweat. Treated with topical antiperspirants and botulinum toxin A.
  • Fistula formation: Treatment is conservative with application of pressure dressings. Problem is in failure to ligate Stensen’s duct.
How well did you know this?
1
Not at all
2
3
4
5
Perfectly
32
Q

Risk factors for oral cavity cancers?

A

Smoking (chronic irritation)
Alcohol intake
Ill-fitting dentures
Premalignant lesion: Leukoplakia

How well did you know this?
1
Not at all
2
3
4
5
Perfectly
33
Q

Principles in treatment of oral cavity cancer?

A

T1-T2: Wide excision

T3-T4: Wide excision + SOHND (if cN0) + Postop radiotherapy

*Same as salivary gland lesions.

How well did you know this?
1
Not at all
2
3
4
5
Perfectly
34
Q

How do you confirm your diagnosis for oral cavity cancers?

A

Punch biopsy.

But your major concern is the extent of the primary cancer. By size and proximity, check for mandibular involvement using a panorex view.

If only periosteum is involved, do a marginal mandibulectomy (1cm inferior portion of the mandible is left in place, 1cm because any smaller and the bone will weaken and fracture).

What is the principle behind this? The periosteum is a very good barrier against tumor invasion. The malignant cells will slide over it and enter the bone through the natural openings for tooth growth.

If there is gross bony involvement, do segmental mandibulectomy.

How well did you know this?
1
Not at all
2
3
4
5
Perfectly
35
Q

Management of cancers of the floor of the mouth and tongue?

A

The floor of the mouth and tongue are both considered to be midline structures. Therefore, lymphatic spread is to both sides of the neck.

T3-T4 lesions with N0 neck:
Bilateral SOHND

One side N0, other side N+:
Do SOHND in the N0 side, and a full neck dissection in the N+ side.

Both N+:
Do a full neck dissection on both sides.

Never do bilateral Radical Neck Dissection. Removal of both IJVs will cause cerebral edema. Only ONE IJV can be removed.

The tongue is notorious for having skip node metastases. They may have enlarged nodes in level V, even without palpable nodes in level I-IV. Hence tongue malignancies warrant a bilateral full neck dissection.

How well did you know this?
1
Not at all
2
3
4
5
Perfectly
36
Q

How is reconstruction done for the oral cavity?

A

Soft tissue defects can be restored with myocutaneous flaps.

Bony defects can be restored with titanium plates, bone grafts, and wires.

Bone grafts can only be used in defects <6cm. It can not be used for larger defects because the bone will resorb.

How well did you know this?
1
Not at all
2
3
4
5
Perfectly
37
Q

Management of laryngeal lesions?

A

As in the pharynx, the newest treatment modality involves chemotherapy and radiotherapy for all stages, because of the emphasis on organ preservation. However, it is yet to be adapted in our local setting.

Remember that the larynx is divided into 3 regions: supraglottic, glottic and subglottic, with SUBGLOTTIC having the POOREST prognosis.

Glottic lesions do NOT metastasize to the neck, because there are NO lymphatic channels in this area.

How well did you know this?
1
Not at all
2
3
4
5
Perfectly
38
Q

Agents for chemotherapy in head and neck cancers?

A

Chemotherapy is used to downgrade tumors, making them more amenable for surgery.

Agents: Cisplatin and 5-FU

Dosage: Cisplatin 50mg/kg/dose
5-FU 50mg/kg/day

What to check prior to administration:

1) WBC count
2) BUN/Crea (Nephrotoxic)

Administration: IV, ensure patient is well-hydrated prior to starting it

  • Cisplatin given on Day 1 x 4 hours
  • 5-FU given for 5 straight days as bolus.
How well did you know this?
1
Not at all
2
3
4
5
Perfectly
39
Q

What is the procedure of choice for the mandible in patients with oral cavity cancer with periosteal involvement of the mandible on panorex?

a. marginal mandibulectomy
b. hemimandibulectomy
c. segmental mandibulectomy
d. total mandibulectomy

A

a. marginal mandibulectomy

How well did you know this?
1
Not at all
2
3
4
5
Perfectly
40
Q

A 1.5 cm mass at the anterior left side of the tongue turns out to be malignant on biopsy. The appropriate management is:

a. Hemiglossectomy with radiation to the ipsilateral neck
b. Hemiglossectomy with modified neck dissection
c. Wide excision plus radiation to the ipsilateral neck
d. Wide excision

A

d. Wide excision

How well did you know this?
1
Not at all
2
3
4
5
Perfectly
41
Q

A 47-year old woman underwent surgery for a 2.5 cm follicular neoplasm diagnosed by FNAB. The frozen section turned to be benign and a lobectomy was carried out. The final histopath however is follicular carcinoma. In this patient:

a. No further surgery is immediately indicated
b. RAI therapy should be instituted
c. Completion thyroidectomy should be done
d. Thyroid suppression therapy is indicated

A

c. Completion thyroidectomy should be done

How well did you know this?
1
Not at all
2
3
4
5
Perfectly
42
Q

A 50-yr old female presented with a small neck mass, cardiac arrhythmia, recurrent dehydration, weakness, weight loss, and microhematuria. The most important diagnostic procedure in this case is:

a. Renal and ureteral ultrasonography
b. FNAB of the neck mass
c. Serum calcium and parathormone
d. Ultrasonography of the neck

A

c. Serum calcium and parathormone

How well did you know this?
1
Not at all
2
3
4
5
Perfectly
43
Q

Order of thoracic outlet neurovascular structures (anterior to posterior):

A

Subclavian vein, phrenic nerve, anterior scalene muscle, subclavian artery, middle scalene muscle

How well did you know this?
1
Not at all
2
3
4
5
Perfectly
44
Q

What nerve runs posterior to the middle scalene muscle?

A

Long thoracic nerve

How well did you know this?
1
Not at all
2
3
4
5
Perfectly
45
Q

What nerve(s) runs in the posterior neck triangle?

A

Accessory nerve and the brachial plexus

How well did you know this?
1
Not at all
2
3
4
5
Perfectly
46
Q

Which muscles are innervated by the accessory nerve?

A

Sternocleidomastoid, trapezius, and platysma

How well did you know this?
1
Not at all
2
3
4
5
Perfectly
47
Q

Name the nerve that crosses the internal carotid artery 1 to 2 cm above the carotid bifurcation:

A

Hypoglossal nerve

How well did you know this?
1
Not at all
2
3
4
5
Perfectly
48
Q

The hypoglossal nerve supplies motor innervation to all of the muscles of the tongue except:

A

Palatoglossus

How well did you know this?
1
Not at all
2
3
4
5
Perfectly
49
Q

The phrenic nerve lies on top of what muscle?

A

Anterior scalene

How well did you know this?
1
Not at all
2
3
4
5
Perfectly
50
Q

Name the branches of the facial nerve:

A

Temporal, zygomatic, buccal, marginal mandibular, cervical branches

How well did you know this?
1
Not at all
2
3
4
5
Perfectly
51
Q

Which branch of the facial nerve is most often injured in carotid surgery?

A

Marginal mandibular nerve

How well did you know this?
1
Not at all
2
3
4
5
Perfectly
52
Q

Name the branches of the trigeminal nerve:

A

Ophthalmic (V1), maxillary (V2), mandibular (V3)

How well did you know this?
1
Not at all
2
3
4
5
Perfectly
53
Q

What nerve is found within the carotid sheath?

A

Vagus

How well did you know this?
1
Not at all
2
3
4
5
Perfectly
54
Q

The recurrent laryngeal nerve innervates all of the muscles of the larynx except:

A

Cricothyroid muscle

How well did you know this?
1
Not at all
2
3
4
5
Perfectly
55
Q

Which nerve is responsible for sensory innervation of the larynx above the level of the vocal folds?

A

Internal branch of the superior laryngeal nerve

How well did you know this?
1
Not at all
2
3
4
5
Perfectly
56
Q

Which nerve is responsible for sensory innervation of the larynx below the level of the vocal folds?

A

Recurrent laryngeal nerve

How well did you know this?
1
Not at all
2
3
4
5
Perfectly
57
Q

Name the branches of the thyrocervical trunk:

A

STAT:

Suprascapular artery. Transverse cervical artery. Ascending cervical artery. inferior Thyroid artery

How well did you know this?
1
Not at all
2
3
4
5
Perfectly
58
Q

What ls the first branch of the external carotid artery?

A

Superior thyroid artery

How well did you know this?
1
Not at all
2
3
4
5
Perfectly
59
Q

Name the blood supply to the nose:

A

Anterior/posterior ethmoidal arteries off the ophthalmic artery
Superior labial artery from the facial artery
Sphenopalatine artery off the internal maxillary artery

How well did you know this?
1
Not at all
2
3
4
5
Perfectly
60
Q

What nerve innervates the strap muscles?

A

Ansa cervicalis

How well did you know this?
1
Not at all
2
3
4
5
Perfectly
61
Q

Define the regions of the cervical lymph nodes:

A
Level I: submental and submandibular nodes 
Level II: upper jugular nodes
Level III: middle jugular nodes
Level IV: lower jugular nodes
Level V: posterior triangle
Level VI: anterior compartment 
Level VII: upper mediastinal nodes
How well did you know this?
1
Not at all
2
3
4
5
Perfectly
62
Q

What structure divides the parotid gland into superficial and deep lobes?

A

Facial nerve

How well did you know this?
1
Not at all
2
3
4
5
Perfectly
63
Q

What is the name of the duct in the parotid gland?

A

Stenson duct

How well did you know this?
1
Not at all
2
3
4
5
Perfectly
64
Q

What is the name of the duct in the submandibular gland?

A

Wharton duct

How well did you know this?
1
Not at all
2
3
4
5
Perfectly
65
Q

What is the most common nerve injured in parotid surgery?

A

Greater auricular nerve

How well did you know this?
1
Not at all
2
3
4
5
Perfectly
66
Q

What is the most common type of cancer of the oral cavity, pharynx, and larynx?

A

Squamous cell carcinoma

How well did you know this?
1
Not at all
2
3
4
5
Perfectly
67
Q

What are the biggest risk factors for head and neck cancer?

A

Tobacco and alcohol

How well did you know this?
1
Not at all
2
3
4
5
Perfectly
68
Q

What is the most common location for an oral cavity cancer?

A

Lower lip

How well did you know this?
1
Not at all
2
3
4
5
Perfectly
69
Q

What head and neck cancer is Epstein-Barr virus infection associated with?

A

Nasopharyngeal squamous cell carcinoma

How well did you know this?
1
Not at all
2
3
4
5
Perfectly
70
Q

What is the treatment for nasopharyngeal squamous cell carcinoma associated with Epstein-Barr virus?

A

Primary radiation

How well did you know this?
1
Not at all
2
3
4
5
Perfectly
71
Q

Oral cavity cancer is most likely to spread to which regional lymph nodes?

A

Submental and submandibular (level I}

How well did you know this?
1
Not at all
2
3
4
5
Perfectly
72
Q

What is the most common benign head and neck tumor in adults?

A

Hemangioma

How well did you know this?
1
Not at all
2
3
4
5
Perfectly
73
Q

What is the most common tumor of the nasopharynx in children?

A

Lymphoma

How well did you know this?
1
Not at all
2
3
4
5
Perfectly
74
Q

What is the most common benign neoplasm of the nose and paranasal sinuses?

A

Papilloma

How well did you know this?
1
Not at all
2
3
4
5
Perfectly
75
Q

What is the most common benign lesion of the larynx?

A

Papilloma

How well did you know this?
1
Not at all
2
3
4
5
Perfectly
76
Q

What is the treatment for an early glottic squamous cell carcinoma?

A

Primary radiation versus conservative surgical resection-laser versus endoscopic (if recurs, perform cordectomy)

How well did you know this?
1
Not at all
2
3
4
5
Perfectly
77
Q

What is the treatment for an advanced glottic squamous cell carcinoma (stages 3 and 4)?

A

Laryngectomy,
modified radical neck dissection, and
postoperative radiation

How well did you know this?
1
Not at all
2
3
4
5
Perfectly
78
Q

Where do head and neck tumors most often distantly metastasize?

A

Lung

How well did you know this?
1
Not at all
2
3
4
5
Perfectly
79
Q

Which disorder involves dysphagia, esophagitis, iron-deficiency anemia, spoon fingers, and an increased incidence of oral cavity cancer?

A

Plummer-Vinson syndrome

How well did you know this?
1
Not at all
2
3
4
5
Perfectly
80
Q

What are the indications for a radical neck dissection?

A

Clinically positive lymph nodes
Fixed cervical mass
High rate of suspicion for metastatic disease

How well did you know this?
1
Not at all
2
3
4
5
Perfectly
81
Q

In oral cavity cancer, when is a modified radical neck dissection indicated?

A

Clinically positive nodes
Bone invasion
Tumor size >4 cm

How well did you know this?
1
Not at all
2
3
4
5
Perfectly
82
Q

Which structures are taken in a radical neck dissection?

A
Accessory nerve, 
cervical branch of the facial nerve, 
internal jugular vein, 
ipsilateral thyroid, omohyoid, stemocleidomastoid muscle, 
submandibular gland, 
sensory nerves C2 to C5
How well did you know this?
1
Not at all
2
3
4
5
Perfectly
83
Q

What is spared in a modified radical neck dissection?

A

Type I: spinal accessory nerve
Type II: spinal accessory nerve, internal jugular vein
Type III: spinal accessory nerve, internal jugular vein, sternocleidomastoid (SCM)

How well did you know this?
1
Not at all
2
3
4
5
Perfectly
84
Q

What percentage of salivary tumors are benign?

A

80%

How well did you know this?
1
Not at all
2
3
4
5
Perfectly
85
Q

What percentage of salivary tumors are in the parotid?

A

80%

How well did you know this?
1
Not at all
2
3
4
5
Perfectly
86
Q

What is the most common benign salivary tumor?

A

Pleomorphic adenoma

How well did you know this?
1
Not at all
2
3
4
5
Perfectly
87
Q

What percentage of pleomorphic adenomas undergo malignant degeneration?

A

5%

How well did you know this?
1
Not at all
2
3
4
5
Perfectly
88
Q

What is the second most common benign tumor of the salivary glands?

A

Warthin tumor

How well did you know this?
1
Not at all
2
3
4
5
Perfectly
89
Q

What percentage of Warthin tumors are bilateral?

A

10%

How well did you know this?
1
Not at all
2
3
4
5
Perfectly
90
Q

What is the most common salivary gland tumor in children?

A

Hemangioma

How well did you know this?
1
Not at all
2
3
4
5
Perfectly
91
Q

What is the most common malignant tumor of the salivary glands?

A

Mucoepidermoid carcinoma

How well did you know this?
1
Not at all
2
3
4
5
Perfectly
92
Q

What is the second most common malignant salivary gland tumor?

A

Adenoid cystic carcinoma

How well did you know this?
1
Not at all
2
3
4
5
Perfectly
93
Q

What is an important oncologic feature of adenoid cystic carcinoma?

A

Very sensitive to radiation; therefore, it can be considered as the treatment modality, although resection is the mainstay of treatment.

How well did you know this?
1
Not at all
2
3
4
5
Perfectly
94
Q

What is the name ofthe vascular plexus located in the anterior portion of the nasal septum, which is responsible for 90% of epistaxis?

A

Kiesselbach plexus

How well did you know this?
1
Not at all
2
3
4
5
Perfectly
95
Q

What percentage of epistaxis can be controlled with anterior or combined anterior and posterior nasal packing?

A

95%

How well did you know this?
1
Not at all
2
3
4
5
Perfectly
96
Q

Which arteries can be ligated in order to control epistaxis?

A

Ethmoid artery, internal maxillary artery

How well did you know this?
1
Not at all
2
3
4
5
Perfectly
97
Q

Why does a nasal septa! hematoma require emergent treatment?

A

The accumulation of blood in the nasal septum may deprive the septal cartilage of its blood supply from the perichondrium.

It requires immediate incision and drainage, septal splinting, and antibiotics to prevent avascular necrosis of the septal cartilage and subsequent saddle nose deformity.

How well did you know this?
1
Not at all
2
3
4
5
Perfectly
98
Q

What is the initial management for a traumatic cerebrospinal fluid (CSF) leak?

A

Head elevation
Avoid nose blowing/straining
With/without antibiotics

How well did you know this?
1
Not at all
2
3
4
5
Perfectly
99
Q

Which diagnostic test can be used to confirm whether fluid ls CSF?

A

B2-transferrin

How well did you know this?
1
Not at all
2
3
4
5
Perfectly
100
Q

What is the treatment for a persistent CSF leak (>4-6weeks)?

A

Surgical repair

How well did you know this?
1
Not at all
2
3
4
5
Perfectly
101
Q

What ls the treatment for a CSF leak associated with meningitis?

A

Surgical repair

How well did you know this?
1
Not at all
2
3
4
5
Perfectly
102
Q

What is the most common cause of laryngeal stenosis?

A

Trauma

How well did you know this?
1
Not at all
2
3
4
5
Perfectly
103
Q

Most common location for an esophageal foreign body:

A

Upper esophagus at the thoracic inlet

How well did you know this?
1
Not at all
2
3
4
5
Perfectly
104
Q

Above what level should a tracheostomy be placed to avoid the complication of a tracheo-innominate fistula?

A

Above the third tracheal ring

How well did you know this?
1
Not at all
2
3
4
5
Perfectly
105
Q

What is the initial treatment for a peritonsillar abscess?

A

IV antibiotics and needle aspiration

How well did you know this?
1
Not at all
2
3
4
5
Perfectly
106
Q

What is the treatment for a retropharyngeal abscess?

A

IV antibiotics and surgical drainage through the posterior pharynx or neck

How well did you know this?
1
Not at all
2
3
4
5
Perfectly
107
Q

What is the treatment for a parapharyngeal abscess?

A

IV antibiotics, incision and drainage through the lateral neck, and leave a drain in place

How well did you know this?
1
Not at all
2
3
4
5
Perfectly
108
Q

What is the initial treatment for acute suppurative parotitis?

A

Antibiotics, IV fluids, sialogogues, warm compresses

How well did you know this?
1
Not at all
2
3
4
5
Perfectly
109
Q

When is it necessary to operate on acute suppurative parotitis?

A

When there is no clinical improvement after 12 hours of treatment

How well did you know this?
1
Not at all
2
3
4
5
Perfectly
110
Q

Which muscle is innervated by the external branch of the superior laryngeal nerve?

A. Posterior cricoarytenoid 
B. Lateral cricoarytenoid 
C. Thyroidarytenoid
D. Cricothyroid
E. Interarytenoid
A

Answer: D.

The cricothyroid muscle is the only muscle of the larynx innervated by the external branch of the superior laryngeal nerve.

All other laryngeal muscles are innervated by the recurrent laryngeal nerve.

111
Q
2. A 48-year-old male with massive head trauma does not have a gag reflex. Which nerve is responsible for the sensory nerve fibers that carry this reflex?
A. Marginal mandibular nerve 
B. Recurrent laryngeal nerve 
C. Facial nerve
D. Glossopharyngeal nerve
E. Vagus nerve
A

Answer: D.

The afferent limb of the gag reflex comes from the glossopharyngeal nerve (cranial nerve IX).

112
Q

The most common cause of a neck mass in males over the age of 60 is:

A. Laryngocele
B. Metastatic carcinoma
C. Branchial cleft cyst
D. Bacterial adenitis
E. Carotid aneurysm
A

Answer: B.

Most neck masses in males over the age of 60 are a result of metastatic spread from cancers in the chest and abdomen.

These cancers frequently metastasize via lymphatic spread, and the most common location for the neck mass is within the supraclavicular fossa.

113
Q

A second-year internal medicine resident stops you in the hall with a painless, 2-cm, slowly enlarging parotid mass. What does this most likely represent?

A. Hemangioma
B. Warthin tumor
C. Pleomorphica denoma
D. Mucoepidermoid carcinoma 
E. Adenoid cystic carcinoma
A

Answer: C.

Pleomorphic adenomas are the most common benign salivary gland tumors, comprising 85% ofall salivary gland neoplasms.

Tumors of the parotid gland are the most common of the salivary gland tumors, with a 5 times higher incidence than tumors of the minor salivary glands.

The vast majority of tumors of the parotid gland are benign in nature.

114
Q
A 14-year-old boy presents to the ER with epistaxis after being struck in the face with a baseball. Examination reveals a compressible, nontender mass involving the nasal septum. What is the most common complication resulting from this condition?
A. Delayed hemorrhage
B. Cavernous sinus thrombosis
C. Sepsis
D. Osteomyelitis
E. Saddle nose deformity
A

Answer: E.

The physical examination reveals findings consistent with a nasal septal hematoma.

The most common complication of this condition is a saddle nose deformity as a result of avascular necrosis from an infected septal hematoma.

115
Q

You are admitting a patient in the ER and previous records indicate she underwent a radical neck dissection. This means she has had:

A. An excisional biopsy of a neck mass

B. Some of the lymph nodes removed in the anterior and posterior neck triangles

C. All the lymph nodes removed but no vital structures removed from her neck

D. All the lymph nodes removed and the spinal accessory nerve, the internal jugular vein, and the sternocleidomastoid removed

E. All the lymph nodes removed and the spinal accessory nerve, the internal jugular vein, external carotid artery, and the sternocleidomastoid removed

A

Answer: D.

A radical neck dissection includes removal of all the ipsilateral lymph nodes from levels I through V along with the spinal accessory nerve, the sternocleidomastoid muscle, and the internal jugular vein.

A modified radical neck dissection removes all the lymph nodes but preserves one or all of the non-lymphatic structures.

116
Q

What is the treatment for a Warthin tumor

A. Observation
B. Surgical enucleation
C. Total parotidectomy with resection of the facial nerve
D. Total parotidectomy with postoperative radiation
E. Superficial parotidectomy

A

Answer: E.

A Warthin tumor is the second most common benign tumor of the parotid gland.

It has an extremely low rate of malignant transformation (0.3%), as well as a low risk of recurrence (1.8% after local excision).

The treatment is superficial parotidectomy, sparing the facial nerve.

Simple enucleation is avoided because it increases the risk of recurrence and facial nerve injury.

117
Q

A 72-year-old man presents with a painless, gradually enlarging mass in the region of the left parotid with facial asymmetry and left-sided facial weakness. Following physical examination, what is the next step?

A. Superficial parotidectomy
B. Total parotidectomy with modified radical neck dissection
C. Fine-needle aspiration (FNA) of the mass and MRI of the head and neck
D. Esophagogastroduodenoscopy and colonoscopy
E. Complete metastatic workup

A

Answer: C.

The next step in the workup includes an FNA and MRI of the head and neck.

Given the patient’s age and symptoms, tissue diagnosis is required for definitive diagnosis, and MRI is necessary to evaluate the extent of disease.

Local spread is common, but the incidence of metastatic disease with malignant salivary gland tumors is rare.

118
Q

Following parotidectomy your patient has sweating in his retroauricular region while eating. This is most likely caused by:

A. Recurrent tumor
B. Cross-innervation of the vagus and sympathetic nerves to the skin
C. Cross-innervation of the auriculotemporal nerve and sympathetic nerves to the skin
D. Cross-innervation to the glossopharyngeal nerve and sympathetic nerves to the skin

A

Answer: C.

Gustatory sweating following parotidectomy is known as Frey syndrome.

Injury to the auriculotemporal nerve can occur during parotid surgery.

The auriculotemporal nerve carries sympathetic fibers to the sweat glands of the scalp and parasympathetic fibers to the parotid gland.

Following injury and subsequent regeneration, the fibers can cross-innervate, leading to gustatory sweating.

119
Q
A 10-year-old boy presents with a cyst and a cyst tract near the angle of his mandible. This cyst has had recurrent infections in it. This cyst most likely connects to the:
A. External auditory canal 
B. The tonsillar pillar
C. The nasal septum
D. Thoracic duct
A

Answer: A.

First branchial cleft cysts typically originate at the angle of the mandible and often extend to the external auditory canal.

Second branchial cleft cysts are found along the anterior border of the sternocleidomastoid muscle and connect with the tonsillar pillar.

120
Q

During neck exploration in a trauma patient, injury is suspected to a nerve that runs about 2 cm above the carotid bifurcation. What is the likely physical examination finding?
A. Hoarseness
B. Inability to make high-pitched sounds
C. Ipsilateral tongue deviation
D. Contralateral tongue deviation
E. Facial droop

A

Answer: C.

The hypoglossal nerve runs about 2 cm above the carotid bifurcation and is the motor nerve of the tongue, except for the palatoglossus.

The tongue would deviate to the same side of the hypoglossal nerve injury.

Hoarseness and an inability to make high-pitched sounds are the result of injury to vagus nerve branches, namely the recurrent laryngeal and superior laryngeal nerve, respectively.

121
Q

A 35-year-old male with a 2-week history of toothache and fever for the last 36 hours presents to the emergency department with fever, nausea, and jaw pain. On examination, he is found to have tachycardia, tachypnea, and fever of 38.7C. The patient has halitosis, and a tender collection at the base of the oral cavity is pushing the tongue up and back. What is the next best step?

A. Initiation of broad-spectrum antibiotics
B. Establishing 2 large-bore IV access sites
C. Intubation
D. Head and neck CT scan with IV contrast
E. Incision and drainage of the abscess

A

Answer: C.

This is a classical description of Ludwig angina, which is an acute infection of the floor of the mouth.

It can rapidly spread to deeper structures and create an airway emergency.

Securing the airway, as in the presented case, would be the first step.

All the other choices would come next, with surgical drainage and antibiotics in the majority of cases.

122
Q

A 64-year-old male university professor is referred to your clink by his primary care physician for a neck mass that had been noticed by the patient for the last few weeks. He states that he has never had a similar problem in the past and is in a good state of health. He wants to know the cause. What is the next step?

A. Panendoscopy (laryngoscopy, EGD, and bronchoscopy)
B. Fine needle aspiration
C. Core needle biopsy
D. History and physical exam
E. Empiric antibiotic therapy for 2 weeks and return to clinic

A

Answer: D.

No pertinent clinical information is provided in the question.

History and physical exam is the first step in any case, including board scenarios.

All other choices are possibilities, depending on what direction the history and physical exam point to.

123
Q

You are being called to labor and delivery to examine a newborn with oral abnormality. He is born at term to a 38-year-old woman G1P1 who had an uneventful pregnancy. The only abnormality that you identified is a deft lip (primary palate), which also involves the alveolus. When would you plan to repair the deft lip?

A. As soon as anesthesiology clears the patient for the operating room
B. After the first choking episode
C. At 10 weeks when the patient is about 10lb
D. At 12 months
E. Electively before schoolage

A

Answer: C.

A rule-of-thumb for repair of the cleft lip is at 10 weeks, 10 lb, and hemoglobin of 10 g/dL. The nasal deformity is also repaired at the same time.

Cleft palate is usually repaired at 12 months to avoid the negative effect of the maxillofacial growth.

124
Q

A 3-year-old boy of a recent immigrant family is brought to the ER with a high fever, throat pain, and drooling. The patient is noticed to have stridor and a leaning-forward position. Lateral neck x-ray showed the thumbprint sign. What is the likely cause?

A. Hemophilus influenzae 
B. Asthma
C. Rhinovirus
D. Laryngeal foreign body 
E. Streptococcal infection
A

Answer: A.

Epiglottitis is rarely seen in the United States due to immunization against H. influenzae type B. Its prevalence has recently increased due to parental refusal of immunization of their children. It is mainly seen in children of 3 to 5 years of age. It can cause airway obstruction and become an emergency. The mainstay of treatment is antibiotics after securing the airway.

125
Q

A 68-year-old woman is evaluated for a 1-month history of worsening dyspnea and hoarseness. Fiberoptic laryngoscopy demonstrates left vocal cord paralysis with full mobility of the right true vocal cord. Findings on physical examination are otherwise unremarkable. Computed tomography (CT) of the chest demonstrates a large mediastinal mass. Which of the following statements is most accurate?

A. The left recurrent laryngeal nerve (RLN) branches off of the vagus nerve and passes around the left subclavian artery back to the larynx.

B. A nonrecurrent RLN can be associated with a retroesophageal right subclavian artery.

C. The RLN supplies all the muscles of the larynx except the posterior cricoarytenoid muscle.

D. The vagus nerve exits the skull through the carotid canal.

E. The RLN enters the larynx through the thyrohyoid membrane to innervate the larynx.

A

ANSWER: B

COMMENTS: The left RLN separates from the vagus in the mediastinum, wraps around the aortic arch at the ductus arteriosus, and then ascends back along the tracheoesophageal groove toward the larynx.

The right RLN divides off the vagus and passes around the right subclavian artery to travel to the larynx.

The RLN supplies all the muscles of the larynx except the cricothyroid muscle, which is innervated by the external branch of the superior laryngeal nerve.

The superior laryngeal nerve also has an internal branch that supplies sensation to the larynx above the true vocal cords.

The RLN provides sensation below this area.

The vagus nerve exits the skull base on both sides through the jugular foramen, not the carotid canal.

The internal branch of the superior laryngeal nerve, not the RLN, enters the thyrohyoid membrane.

The RLN travels along the tracheoesophageal groove and enters the larynx just superior to the cricoid cartilage.

Nonrecurrent nerves occur most commonly on the right side in up to 1%–2% of patients and can be associated with a retroesophageal right subclavian vein.

126
Q

Which of the following statements is true regarding head and neck carcinogenesis and molecular therapy?

A. A synchronous second primary tumor is defined as the one developing within 1 year of the initial cancer.

B. Human papillomavirus (HPV) has been shown to be a factor in the development of certain head and neck cancers.

C. Cetuximab (IMC-C225) is a monoclonal antibody therapy targeted against the transforming growth factor-β.

D. p53 is commonly underexpressed in head and neck cancers.

E. The most frequently mutated tumor suppressor gene in
head and neck cancers is cyclin D1.

A

ANSWER: B

COMMENTS: Head and neck carcinogenesis is a complex process with multiple etiologic factors and a wide variety of genetic alterations that have been identified.

The incidence of second primary tumors in head and neck cancers is not insignificant (3%–7%), and they should be surveyed for in the upper aerodigestive tract, esophagus, and lung at the time of initial diagnosis.

Synchronous versus metachronous lesions are defined in relation to the time of diagnosis from the initial discovery of the tumor; synchronous lesions are found within 6 months, with metachronous lesions being diagnosed after 6 months.

In recent years, HPV has been identified as an etiologic factor in certain head and neck cancers, specifically the oropharynx.

Protooncogenes are genes that produce proteins involved in the normal cell regulation and function.

Mutation of these genes, including those for epidermal growth factor receptor (EGFR), cyclin D1, and vascular endothelial growth factor, occurs in head and neck cancers.

Tumor suppressor genes such as p53 and p16-ARF encode proteins that halt tumor growth and carcinogenesis.

Cetuximab is a novel monoclonal antibody therapy that has been designed to target EGFR.

Trials are ongoing regarding its efficacy in combination with standard treatments such as radiotherapy and chemotherapy.

Mutated p53 is poorly degraded and therefore commonly overexpressed in head and neck cancers.

It is the most commonly altered tumor suppressor gene in human cancers, with p16-ARF being the most commonly altered gene locus in head and neck cancers.

127
Q

A 75-year-old man with T2N0M0 squamous cell carcinoma (SCC) of the lower lip undergoes primary resection of the tumor, with full-thickness resection resulting in a defect involving about 55% of the central lower lip but not involving the oral commissure. Which of the following is the best option for the reconstruction of the defect?

A. Primary closure

B. Abbe two-stage cross-lip transfer flap

C. Radial forearm free flap transfer

D. Nasolabial transposition flap

E. Estlander flap

A

ANSWER: B

COMMENTS: The lip begins at the vermilion border and is a subsite of the oral cavity. Risk factors for lip carcinoma include sun exposure and tobacco use.

The lower lip is by far the most common site of tumor development, with SCC being the most common pathology.

Basal cell carcinoma is the most frequent pathology found on diagnosis for upper lip cancers. Initial evaluation should include testing for mental nerve involvement, evaluation for man- dibular invasion, and examination of the cervical lymphatics. CT can be useful, especially in staging the mandible and cervical nodes.

Treatment of the early-stage disease can consist of either primary external beam radiation or surgical resection.

Primary excision is typically the favored treatment. An elective lymph node neck dissection for a clinically negative neck is performed in patients with advanced T stage disease or in the setting of deep tumor invasion in which the risk for occult cervical lymph node disease is elevated.

Therapeutic neck dissection is appropriate in patients with positive nodal disease.

Indications for postoperative irradiation include advanced T stage, positive margins, perineural/ perivascular invasion, multiple or bulky nodal disease, and invasion of bone.

Fortunately, given its location, many lip cancers are detected early and can be treated with a high success rate. Stages I and II lesions have a 90% 5-year survival rate.

The survival rate drops to about 50% in patients with cervical lymph node involvement.

Reconstructive options following the excision of carcinoma of the lip generally depend on the location of the lesion (medial versus lateral with the involvement of the oral commissure) and the size of the defect.

Smaller lesions that involve less than a third to half of the lip can be closed primarily with the local advancement of the tissue.

Larger defects involving between half and two-thirds of the lip will probably require donated tissue such as a pedicled two- stage cross-lip transfer flap.

An Abbe flap is used for central defects, whereas Estlander-style flaps are used for lateral defects.

Larger defects can be repaired with local advancement techniques, local rotational flaps, or distal free flaps.

128
Q

A 68-year-old woman with chronic respiratory failure underwent a tracheostomy for long-term ventilator support. The procedure was uneventful, and the patient was dis- charged to a long-term care facility after an initial tracheos- tomy tube change 1 week later. She is readmitted 2 weeks later with a report at the nursing facility of a 1-min episode of brisk, bright red bleeding from the tracheostomy site that resolved without intervention. Her hemoglobin concentration is 10.2 g/dL, and coagulation studies are normal. What is the most likely diagnosis?

A. Pneumonia

B. Tracheitis

C. Bleeding of granulation tissue in the stoma

D. Tracheoinnominate fistula

E. Bleeding from the anterior jugular vein

A

ANSWER: D

COMMENTS: This is a case of classic “sentinel” bleeding that occurs before the full rupture of the innominate artery.

It can occur with a low tracheostomy or a high innominate artery with the erosion of the anterior tracheal wall as a result of pressure necrosis.

Fiberoptic examination of the area should be performed to evaluate the situation.

If there is an obvious evidence of erosion, the patient might need a sternotomy and mediastinal exploration.

If the index of suspicion is lower or the examination is inconclusive, diagnostic imaging with a high-resolution CT or angiography (or both) may be appropriate.

This complication carries an extremely high mortality rate if massive bleeding occurs.

Attempts to control active bleeding can consist of direct digital pressure or inflation of a cuffed tube directly over the area for tamponade.

Any of the other choices would not cause rapid-onset bright red bleeding.

Bleeding from the anterior jugular veins usually occurs immediately in the postoperative setting if they were not ligated adequately during surgery.

Other complications of tracheostomy in the early period include infection, pneumothorax, bleeding, and tube obstruction.

Later complications can include wound breakdown, formation of granulation tissue, tracheal stenosis, tracheoesophageal fistula, and tracheoinnominate fistula, as just described.

129
Q

A 34-year-old woman is seen in the office with a 3-month history of a left-sided neck mass and pain with mastication. On examination, a 3-cm nontender mass is present at level II; it moves laterally but not in the craniocaudal direction. CT with intravenous (IV) contrast enhancement demonstrates a 3.5-cm mass at the carotid bifurcation. A follow-up angio- gram is obtained. Which of the following statements is most accurate regarding this case?

A. It represents the most common paraganglioma in the head and neck.

B. The rate of malignancy is about 40%.

C. The majority of these lesions show functional secretion of catecholamines.

D. A fine-needle aspiration biopsy (FNAB) is indicated to rule out malignancy.

E. Radiation therapy is the most effective treatment for these lesions.

A

ANSWER: A

COMMENTS:

A carotid body tumor is the most common paraganglioma in the head and neck.

Other paragangliomas that occur in this region include the vagal and jugulotympanic types.

The majority of paragangliomas are solitary and nonfamilial.

Multicentricity is reported in about 10% of cases, with about a 6% rate of malignancy.

Only about 1%–3% of them are considered functional and can be evaluated with a 24-h urine collection for the analysis of catecholamines.

The “Fontaine sign” refers to mobility in only the lateral direction on palpation, whereas pain occurring with chewing is called “first-bite syndrome.”

Histopathologic evaluation of this tumor demonstrates two types of cells: type I, chief cells/granular cells, and type II, sustentacular supporting cells.

Unlike most head and neck tumors, malignancy is not diagnosed by the presence of dysplastic changes in the primary tumor.

The presence of a metastatic disease either to the regional cervical lymphatics or to the distant sites is the only diagnostic criteria.

Imaging is the major modality for primary diagnosis since needle biopsy is contraindicated.

CT will show a hypervascular mass at the carotid bifurcation, with magnetic resonance imaging (MRI) demonstrating a classic “salt and pepper” appearance on T2-weighted sequences because of hemorrhage and flow voids.

On angiography, the internal and external carotid arteries will be bowed apart, a finding referred to as the “lyre sign.”

These tumors are best treated surgically, and vascular reconstruction may be necessary.

Radiation therapy can stop the growth of these lesions but not shrink their size.

Patients should be aware of the potential risk of injury to the vagus.

130
Q

Which statement is the most accurate regarding the histology of the head and neck?

A. The pharynx is lined exclusively by nonkeratinizing stratified squamous epithelium.

B. The minor salivary glands lie in the submucosa of the oral cavity and pharynx.

C. The Waldeyer ring consists of only two structures: the palatine tonsils and adenoids.

D. The adenoids have crypts lined by stratified squamous epithelium.

E. The nasal cavity consists entirely of ciliated respiratory epithelium.

A

ANSWER: B

COMMENTS: The pharynx is lined by both nonkeratinizing strat- ified squamous epithelium and ciliated respiratory epithelium.

The Waldeyer ring consists of the palatine tonsils, adenoids, and the lingual tonsils, which lie along the base of the tongue.

The palatine tonsils have crypts lined by stratified squamous epithelium.

The adenoids are covered by pseudostratified ciliated columnar epithelium with surface folds but, unlike the tonsils, do not have crypts.

The tonsillar crypts are designed to trap foreign antigens for presentation to the lymphoid follicles.

The nasal cavity is composed primarily of respiratory epithelium but also contains specialized sensory olfactory epithelium along the roof.

Hundreds of minor salivary glands lie in the submucosa of the oral cavity and pharynx.

131
Q

A 22-year-old man is brought to the emergency department by ambulance with a 2-day history of lower tooth pain and neck swelling. He was prescribed antibiotics by his primary care physician yesterday, but his condition had not improved overnight. This morning he felt his “throat beginning to close” and called the emergency medical service. He is febrile at 102°F with a heart rate of 105 beats/min, blood pressure of 110/70 mmHg, and respiratory rate of 20 breaths/min. His white blood cell count is 38,000/mm3. On examination, he is noted to have firm tender swelling of the submental region with skin erythema. In addition, severe edema of the floor of the mouth and tongue is present. His Sao2 is 100% on 40% O2 by face tent, and he is ventilating well, anxious, and sitting forward drooling. Flexible laryngoscopy shows no laryngeal edema. What is the next appropriate step in the management?

A. IV antibiotics and steroids with close observation of the airway in the intensive care unit

B. High-resolution CT of the neck with dye

C. Immediate cricothyrotomy in the emergency department under local anesthesia

D. Immediate transfer to the operating room (OR) for awake fiberoptic intubation and be prepared for tracheostomy if needed

E. Incision and drainage of a suspected neck abscess at the bedside

A

ANSWER: D

COMMENTS: This patient has Ludwig angina, probably sec- ondary to an odontogenic infection given the history of tooth pain.

This condition involves a rapidly evolving soft tissue cellulitis that spreads through the fascial planes of the sublingual space, submandibular space, and anterior aspect of the neck.

It can cause severe swelling of the floor of the mouth and tongue and possibly airway obstruction.

Initial management in this patient should involve securing his airway.

An emergency cricothyrotomy would be appropriate in the emergency department if the patient was currently decompensating and exhibiting obstruction and needed an airway immediately.

He is ventilating well but drooling and posturing forward.

This patient should be taken to the OR expedi- ently to perform fiberoptic intubation and a possible tracheostomy under controlled circumstances.

At that point, CT could be performed to evaluate for an abscess collection.

Exploration of the neck at the bedside, especially without securing the airway, is unsafe and not appropriate.

Tonsillitis is the most frequent cause of deep tissue neck infections in children with odontogenic sources, and an IV drug injection is a more common cause in adults.

Peritonsillar abscesses are the most commonly encountered sequelae.

Classic signs and symptoms include drooling, hot potato voice, otalgia [from irritation of cranial nerve (CN) IX], and unilateral swelling of the soft palate with uvular deviation to the opposite side.

Treatment includes drainage by transoral needle aspiration or incision and drainage with a scalpel blade.

A para- pharyngeal space abscess can manifest in a similar fashion to a peritonsillar abscess as severe throat and neck pain but without the appearance of soft palate/peritonsillar swelling. The tonsil may appear medially.

It is adjacent to many of the other spaces and can allow spread along the carotid sheath (“Lincoln highway of the neck”) or damage to CNs IX to XII.

Drainage is generally performed in a transcervical fashion if necessary.

Retropharyngeal space abscesses are more common in young children and require some clinical suspicion to detect. Initial signs may include fever, lack of appetite, cervical adenopathy, and torticollis.

This area can usually be approached transorally if the infection is limited but may require transcervical drainage.

CT is mandatory to define its extent and to aid in planning access. The retropha- ryngeal space extends from the skull base to the mediastinum.

The retropharyngeal space has a midline raphe that causes unilateral shifting, whereas the prevertebral space does not, with these collections appearing more midline.

132
Q

A 75-year-old man with a long history of tobacco use comes to the office with a newly discovered tongue mass. A 3-cm ulcerated lesion is noted on the right anterior aspect of the tongue. Biopsy of the lesion demonstrates SCC. CT of the neck with IV contrast enhancement shows an enlarged 2-cm lymph node in the level II region on the left side. Findings on chest radiography are clear. How would you stage this patient’s disease?

A. T1N1M0

B. T2N1M1

C. T4N2cM0

D. T2N3M0

E. T2N2cM0

A

ANSWER: E

COMMENTS:

The T classification as defined by the American Joint Committee on Cancer (AJCC) refers to the primary tumor.

Carcinoma of the oral cavity is staged according to size.

T2 lesions are 2 to 4 cm, as in this patient. Oral cavity T staging is as follows:

T1, less than 2 cm;

T2, 2 to 4 cm;

T3, greater than 4 cm; and

T4, invasion of adjacent structures.

Clinical staging of the neck has been made more accurate by high-resolution imaging (CT).

Had this node been ipsilateral, it would have been staged as N1.

However, since it is contralateral to the primary disease, it is classified as N2c.

Table 30.1 applies to primary tumors of the oral cavity, oropharynx, hypopharynx, and larynx:

NX Regional lymph nodes cannot be assessed
N0 No regional lymph node metastasis
N1* Metastasis in a single ipsilateral lymph node, 3 cm or less in
greatest dimension
N2* Metastasis in a single ipsilateral lymph node, more than 3 cm
but not more than 6 cm in greatest dimension; or in multiple ipsilateral lymph nodes, none more than 6 cm in greatest dimension; or in bilateral or contralateral lymph nodes, none more than 6 cm in greatest dimension
N2a* Metastasis in a single ipsilateral lymph node more than 3 cm but not more than 6 cm in greatest dimension
N2b* Metastasis in multiple ipsilateral lymph nodes, none more than 6 cm in greatest dimension
N2c* Metastasis in bilateral or contralateral lymph nodes, none more than 6 cm in greatest dimension
N3* Metastasis in a lymph node more than 6 cm in greatest dimension

*Note: A designation of “U” or “L” may be used for any N stage to indicate metastasis above the lower border of the cricoids (U) or below the lower border of the cricoids (L). Similarly, clinical/radiological extracapsular spread (ECS) should be recorded as E− or E+, and histopathologic ECS should be designated as En, Em, or Eg.
.

133
Q

Which of the following statements is true regarding FNAB?

A. It is contraindicated for the diagnosis of parotid masses due to the risk of tumor seeding.

B. Rates of false-negative and false-positive diagnoses are low, and the accuracy of detecting malignant lesions is high.

C. FNAB is not effective in the diagnosis of lymphoma.

D. FNAB should be done using an 18- to 21-gauge needle.

E. Once an FNAB specimen is obtained, it should be placed in saline for transport to pathology.

A

ANSWER: B

COMMENTS:

FNAB is not contraindicated for parotid mass diagnosis.

Large-bore needle biopsies have an increased risk of tumor seeding.

FNAB has low false-negative and -positive rates, and malig- nancy detection is high.

It is effective in lymphoma diagnosis and should be performed with a 21- to 25-gauge needle.

FNAB specimens should not be placed in saline as it is a nonphysiologic medium.

FNAB has become the standard technique for the initial office evaluation of a neck mass due to the technique’s high diagnostic accuracy.

The technique reliably can differentiate between reactive and inflammatory processes not requiring surgery from neoplasia as well as between benign and malignant tumors.

The overall diagnostic accuracy is approximately 95% for all masses in the head and neck including 95% for benign lesions and 87% for malignant lesions.

If the result is nondiagnostic, which occurs approximately 10% of the time, the FNAB can easily be performed again or an excisional biopsy can be performed if indicated.

FNAB is additionally useful in the diagnosis of lymphoma. Immunotyping can be performed on FNAB specimens that can detect surface antigens and cell lineage (T cell or B cell) and can confirm the presence of monoclonal light or heavy chains.

Even if the FNAB cannot absolutely confirm lymphoma, it typically still rules out SCC and thus would lead the surgeon to a possible open biopsy of the mass to further evaluate for lymphoma rather than pursue a workup of head and neck SCC, which would include laryngoscopy and esophagoscopy to search for a primary tumor.

The FNAB technique is typically performed in the office setting under local anesthesia, with the application of 1% lidocaine with epinephrine.

A 21- to 25-gauge needle on a 10- to 20-mL syringe is then inserted into the mass and moved in and out of the mass in short vibratory strokes of 2 to 3 mm while aspirating on the syringe.

These vibratory strokes continue until cellular material enters the needle hub at which point the needle is removed from the mass.

This process is repeated at least twice and as many as four times according to the literature. Pressure should then be held on the mass to stop bleeding and a bandage applied.

The aspirated material can either be expelled directly onto slides and taken to a pathologist or fixed directly in CytoLyt.

The advantage of placing the material directly on slides is that the specimen does not need to be spun down, and analysis with microscopy can begin immediately.

134
Q

A 47-year-old male immigrant from southern China has a complaint of right-sided hearing loss for the past 3 weeks. He has no history of a recent upper respiratory infection
or chronic ear disease. His clinical history is otherwise unremarkable. Serous otitis media is noted in his right ear. In addition, a 1.5-cm level V lymph node is palpated on the left side of the neck. The rest of the physical examination is within normal limits. What is the next step in management?

A. Treatment with amoxicillin for 10 days

B. Flexible nasopharyngoscopic examination

C. Ototopical antibiotic drops

D. Oral and topical decongestants for 2 weeks with reexamination

E. Observation with follow-up in 2 weeks if the symptoms do not resolve

A

ANSWER: B

COMMENTS: Any adult patient with a new onset of unilateral middle ear effusion and no related history of recent upper respira- tory illness should undergo evaluation of the nasopharynx.

Another clinical clue in this patient is the presence of a level V node, which is a common location for cervical metastasis from nasopharyngeal cancer.

This is especially true in this patient given his history of a southern Chinese background.

This area, particularly the Guangdong Province, has an increased incidence of nasopharyngeal carcinoma.

The etiologic factors are multifactorial and include genetic and environmental factors.

Infection with Epstein-Barr virus (EBV) has been shown to have an etiologic role.

In addition, dietary factors, including the intake of salted fish, have a strong association with the development of this disease.

Nasopharyngeal carcinoma is divided by the World Health Organization into two groups, keratinizing and nonkeratinizing carcinoma, with the second group including both differentiated and undifferentiated subtypes.

The diagnosis of nasopharyngeal carcinoma is made using nasopharyngoscopy and biopsy.

High serologic levels of immunoglobulin (Ig)A to EBV viral capsid antigen and early antigen are seen in patients with this disease.

Nasopharyngoscopy and biopsy has been advocated as a screening tool in high-risk populations based on a study showing a 5.4% diagnosis rate of subclinical nasopharyngeal cancer in patients with elevated IgA levels to EBV who live in the Guangdong Province.

Treatment consists mainly of chemotherapy and radiation therapy. Surgery has a limited role in treating nasopharyngeal cancer in patients with limited primary site recurrence and nodal recurrence.

135
Q

Which of the following statement is most accurate regarding the use of chemoradiation therapy for head and neck cancers?

A. Chemotherapy can be used as a single-modality primary therapy with an intent to cure in many head and neck cancers.

B. Induction chemotherapy plus radiation therapy has superior overall survival results when compared with surgery plus radiation therapy for the treatment of advanced-stage laryngeal cancer.

C. Postoperative concomitant chemotherapy with radiation therapy improves overall survival compared with postoperative radiation therapy alone in high-risk, locally advanced head and neck cancers.

D. Chemotherapy has no role in the palliative setting for metastatic head and neck cancers.

E. Chemotherapy in the postoperative adjuvant setting in conjunction with radiation therapy has the advantage of improving survival while not increasing mucosal and overall toxicity.

A

ANSWER: C

COMMENTS: Chemotherapy has developed an increasing role over the past two decades in the treatment of head and neck SCCs.

For early-stage patients, treatment consists of either radiation therapy or surgery, with chemotherapy having little to no role in the treat- ment.

It is never used as a primary single-modality treatment for head and neck cancers with an intent to cure.

For patients with advanced metastatic or recurrent disease, chemotherapy can be used in the palliative setting to inhibit tumor growth for a limited effective period.

Its main role is in the treatment of locoregionally advanced stage III/IV cancer.

A second role of chemotherapy in this group of patients is for organ preservation.

Patients with an advanced primary T stage are best served by total laryngectomy.

For advanced-stage unresectable tumors, concurrent chemoradiation therapy, compared with radiation therapy alone, has shown an improved locoregional control with a questionable overall survival benefit.

136
Q

A 66-year-old man with T3N2bM0 SCC on the base of the tongue undergoes concomitant chemotherapy and external beam irradiation for a total of 6 weeks. At the end of the treatment, he is noted to have a persistently enlarged, left neck level II lymph node about 5 cm in size, for which salvage surgical therapy and standard radical neck dissection are planned. Which of the following is true?

A. The phrenic nerve is commonly injured during surgery.

B. Radical neck dissection includes removing the internal jugular vein, sternocleidomastoid (SCM) muscle, and vagus nerve.

C. The patient will probably need postoperative physical therapy for his shoulder.

D. Resection of the internal carotid artery is a common component of this surgery.

E. Level VI is a part of a radical neck dissection.

A

ANSWER: C

COMMENTS: Neck dissection involves the removal of the cervi- cal lymphatic tissue and related structures for the treatment of head and neck cancers.

Radical neck dissection involves removing all of the cervical lymphatic tissue in neck levels I through V, in addition to removing the internal jugular vein, SCM muscle, and spinal accessory nerve.

A modified radical neck dissection is performed for the same clinical indications as a radical neck dissection.

The differentiating feature is that it preserves at least one of the follow- ing: the internal jugular vein, SCM muscle, or spinal accessory nerve.

Levels I through V lymph nodes are removed as for a tradi- tional radical neck dissection.

However, it is frequently possible to preserve one of the major structures just listed, especially when it is not involved with tumor.

Particularly in the case of the spinal accessory nerve, this is preferable to prevent postoperative shoulder dysfunction from denervation of the trapezius muscle.

The patient in this scenario will most certainly need postoperative physical therapy due to CN XII resection.

Selective neck dissection is used for removing a limited number of lymph node levels, as opposed to resecting levels I through V.

Furthermore, the internal jugular vein, SCM muscle, and CN XII are preserved. The philosophy behind performing this type of dissection is to remove the clinically uninvolved lymphatic groups thought to be most at risk for future metastatic disease.

Therefore the type of dissection is predicated on where the primary tumor is located.

Oral cavity cancers typically metastasize to levels I, II, and III. These levels would be included in what is termed a supraomohyoid selective neck dissection.

Laryngeal, hypopharyngeal, and oropharyngeal cancers commonly spread to levels II, III, and IV, which would be incorporated into a lateral selective neck dissection.

Posterolateral neck dissection (levels II, III, IV, and V) is most commonly used in the setting of cutaneous malignancies involving the posterior region of the scalp.

Sentinel lymph node biopsy is under investigation for its efficacy in SCC of the head and neck.

It is currently used in the management of cutaneous melanoma of the head and neck.

However, the internal carotid artery is not considered a standard part of the routine neck dissection, and there is controversy regarding the utility and benefit of resection.

The phrenic nerve is at risk when dissecting along the floor of the neck.

A level VI or central compartment dissection is not part of a standard radical neck dissection.

It is typically used in patients with thyroid tumors, tracheal tumors, or laryngeal cancers with extensive subglottic extension.

Therapeutic neck dissection has been used to treat positive metastatic cervical disease (N+) through either a radical or a modified radical neck dissection.

Currently, some surgeons are beginning to advocate the use of a more limited selective neck dissection if the patient has limited N1 disease.

In addition to being used as primary treatment, it also can be used as salvage treatment after failed nonsurgical therapy, such as external beam radiation.

For patients with clinically negative disease in the neck (N0), elective neck dissection can be performed to target the most likely sites of metastatic drainage.

Such dissection is based on the primary site of the tumor. Sentinel lymph node biopsy in the treatment of SCC of the head and neck is currently under investigation and not yet advocated widely as a proven standard of care.

137
Q

A 55-year-old man with an advanced-stage SCC of the larynx is undergoing a total laryngectomy with bilateral modified radical neck dissection. While dissecting on the right side, you inadvertently enter the internal jugular vein and proceed to clamp the vessel with the intention of ligating it for hemostasis. Shortly thereafter, the patient is noted to be hypotensive. Bilateral breath sounds are auscultated, but a mill-wheel murmur is heard over the precordium. What is the next step in management?

A. Place the patient in the left lateral decubitus position and insert a central venous catheter.

B. Place the patient in the right lateral decubitus position and insert a central venous catheter.

C. Place the patient in the reverse Trendelenburg position.

D. Call for an intraoperative chest radiograph.

E. Pack the wound and place the patient in the prone position.

A

ANSWER: A

COMMENTS: This patient has a venous air embolism as a result of the internal jugular vein being inadvertently opened.

With all surgeries above the heart, there is a risk for air embolism.

This is especially true in neurosurgical procedures, where there is signifi- cant elevation of the wound relative to the heart.

In head and neck surgery, patients are typically placed in the reverse Trendelenburg position, which places them at an additional risk for entrance of air into the venous system.

A “mill-wheel murmur” is the traditional finding on cardiac examination and may be detected on precordial Doppler ultrasound.

The clinical manifestations can include cardio- vascular, pulmonary, and neurologic findings.

Cardiac findings can include tachyarrhythmias, right-sided heart strain, and myocardial ischemia.

Pulmonary findings can include hypercapnia with decreased O2 saturation.

A decreased cardiac output results in decreased cerebral perfusion, but a direct air embolism to the central nervous system can occur through a patent foramen ovale.

Initial treatment involves placing the patient in the left lateral decubitus position (Durant position) to try to force the air to stay in the right side of the heart in an attempt to prevent it from traveling into the pulmonary circulation.

Placement of a central venous catheter with aspiration of air from the right atrium can also be attempted.

Other supportive efforts may need to include cardiopulmonary resuscitation if the situation deteriorates.

Hyperbaric oxygen therapy has been shown to be of benefit in some studies.

138
Q

A 53-year-old male with a history of diffuse large B-cell lymphoma is admitted with neutropenic fevers and started on broad-spectrum IV antibiotics. Over the next 2 days, he complains of a progressive left-sided vision loss, decreased sensation in the left V2 distribution, nasal congestion, and epistaxis. Nasal endoscopy is significant for a blackened eschar on the left middle turbinate. He is not sensate in this region, and the tissue does not bleed. Which of the following is true regarding an invasive fungal sinusitis?

A. The causative organism is most frequently Candida albicans.

B. This disease process is rarely lethal.

C. The pathophysiology of this disease is characterized by the organism invading soft tissue, bone, and vessel walls, which results in vascular occlusion, infarction, and tissue necrosis.

D. This disease requires a quantitative neutrophil deficit as a precursor.

E. The gold standard for diagnosis is fungal culture.

A

ANSWER: C

COMMENTS: The causative organisms are most commonly Mucor, characterized by nonseptate hyphae with a 90-degree branching pattern, and Aspergillus, characterized by septate hyphae with a 45-degree branching pattern.

This disease is fatal in 50%–80% of patients.

The pathophysiology is characterized by the fungal organism invading the soft tissues and bone, causing vascu- lar occlusion, infarction, and tissue necrosis.

This disease occurs almost exclusively in immunocompromised patients (those with diabetic ketoacidosis, undergoing chemotherapy, with bone marrow transplant) and can occur in those with qualitative deficiency (e.g., diabetic ketoacidosis) as well as quantitative deficiency (chemo- therapy, acquired immunodeficiency syndrome).

The gold standard for diagnosis is histopathology.

Invasive fungal sinusitis is characterized by mucosal invasion of mycotic organisms and by angioinvasion, leading to tissue necrosis and potential for invasion into nearby structures such as the orbit and brain.

The population at risk includes immunocom- promised patients.

Typical presenting symptoms include neutrope- nic fever, nasal congestion, vision change, V2 numbness, and soft tissue swelling.

Physical examination may yield periorbital skin necrosis or blackened eschar of the hard or soft palate.

Nasal endoscopy is characterized by mucosal changes of nasal structures such as the septum and middle turbinate.

The mucosa can appear pale or may be covered by a gray or black eschar.

Nasal mucosa is often insensate in these patients and does not bleed.

Treatment requires an urgent surgical debridement, often coupled with a long-term IV antifungal therapy.

Histopathology is the gold standard for diagnosis.

Fungal culture can be used as an adjunct, but it is less sensitive and may take weeks to provide a diagnosis, which results in decreased utility in such an acute and life-threatening disease process.

139
Q

A 65-year-old male undergoes parotidectomy for a parotid neoplasm. Which of the following is true regarding parotidectomy?

A. The facial nerve is typically found 3-cm anterior, inferior, and deep to the tragal cartilage.

B. Frey’s syndrome is caused by injury to the greater auricular nerve and presents with sweating and reddening of the skin during meals.

C. A common complication of parotidectomy is injury to the greater auricular nerve, which causes perioral numbness.

D. Frey’s syndrome is the result of an aberrant innervation of cutaneous sweat glands by postganglionic parasympa- thetic fibers.

E. The parotid gland is divided into superficial and deep lobes by the facial vein.

A

ANSWER: D

COMMENTS: The facial nerve is found 1 cm anterior, inferior, and deep to the tragal cartilage (tragal pointer).

Frey’s syndrome is the result of an injury to the auriculotemporal nerve, which results in an aberrant innervation of cutaneous sweat glands by postgan- glionic parasympathetic fibers.

Injury to the greater auricular nerve causes numbness to the skin overlying the mastoid bone and portions of the pinna.

The parotid gland is divided into superficial and deep lobes by the retromandibular vein.

Facial nerve identification is crucial while performing paroti- dectomy in order to preserve its integrity and function.

The facial nerve can be identified by utilizing multiple landmarks.
• Tragal pointer: The facial nerve is 1 cm anterior, inferior, and deep to the tragal cartilage.
• Tympanomastoid suture line: The facial nerve is 6 to 8 mm deep to the inferior edge of the tympanomastoid suture line.
• The digastric tendon insertion point onto the digastric ridge
defines the plane of the facial nerve.
• The facial nerve main trunk can additionally be identified
exiting the skull base at the stylomastoid foramen or, alternatively, can be found by performing retrograde dissection after finding a distal branch.

There are numerous potential complications of parotidectomy.

First, facial nerve injury can occur and would result in facial paresis or paralysis on the affected side. If this is identified intraoperatively, the nerve should be immediately repaired.

Second, greater auricular nerve hypoesthesia can be the result of a traction injury or transection. This results in periauricular numbness and typically resolves within 9 months if not deliberately transected.

Third, salivary fistulas can occur and typically resolve in 2 to 3 weeks. Fourth, Frey’s syndrome occurs as a result of injury to the auriculotemporal nerve (which contains sympathetic fibers) and results in an aberrant innervation of cutaneous sweat glands by postganglionic parasympathetic fibers.

It presents as sweating and reddening of the skin during meals.

Other complications include hematoma, infection, and flap necrosis.

140
Q

Which of the following is true regarding a cutaneous
malignant melanoma of the head and neck?

A. The most powerful predictor of survival is the depth of invasion and the presence or absence of ulceration of the primary lesion.

B. The most powerful predictor of survival is the regional nodal status.

C. The Clark level of invasion is a key portion of the tumor-node-metastasis (TNM) staging system.

D. An elective neck dissection is the recommended way to initially evaluate the regional nodal status.

E. The mitotic rate is not included in the TNM staging system.

A

ANSWER: B

COMMENT: The depth of invasion and ulceration are key factors of the TNM staging system, but the regional nodal status is the most powerful predictor of survival for patients with malignant melanoma of the head and neck.

T staging takes into account the depth of invasion, not Clark levels.

Sentinel lymph node biopsy has become the approach to the initial evaluation of the regional nodal status, not elective neck dissection.

Mitotic rate plays a role in TNM staging.

The classic finding that raises suspicion for malignant mela- noma is the presence of a pigmented lesion that changes over the course of weeks to months.

Change in diameter, border irregularity, color change, ulceration, itching, pain, and bleeding are all signs that should raise suspicion for melanoma.

Excisional biopsies are the favored method to elucidate diag- nosis, while incisional or punch biopsies are acceptable alternatives if there are anatomic or cosmetic constraints.

If biopsy is positive for an invasive melanoma, even if accomplished by an excisional biopsy with negative margins, further resection of the primary site with larger margins is necessary.

T staging is based on thickness, presence of ulceration, and mitotic rate. N status is based on the regional nodal status, which, as previously stated, is the most powerful predictor of survival in patients with malignant melanoma of the head and neck.

The decision to evaluate the regional nodes is based on the characteristics of the primary tumor and the risk of nodal metastasis.

Patients with T1 lesions have a 3% risk of metastasis to a sentinel lymph node; T2, 8%–12%; T3, 23%–27%; and T4, 24%–44%.

Patients with lesions that are 0.75 mm or less in depth do not require evaluation of regional nodes, and observation is appropriate for them.

Patients with lesions between 0.76 and 1.0 mm in depth without ulceration and less than 1 mitosis/mm2 should consider sentinel lymph node biopsy.

Patients with lesions between 0.76 and 1 mm in depth with ulceration or greater than 1 mitosis/mm2 or with lesions greater than 1 mm in depth should be recommend a sentinel lymph node biopsy.

141
Q

A 10-year-old girl presents to your office with a history of a recurrent painful right lateral neck mass. She has required multiple rounds of antibiotics, which successfully resolve the pain and overlying skin erythema; however, the mass has always persisted. You obtain imaging that is consistent with a second branchial cleft cyst. Which of the following is true regarding branchial cleft anomalies (BCAs)?

A. The spinal accessory nerve is at risk during the excision of the first BCA.

B. The second BCA is characterized by a neck mass at the anterior border of the SCM with a tract that runs between the internal and external carotid arteries and terminates at the ipsilateral tonsillar fossa.

C. Third BCAs are the most common branchial cleft lesion.

D. The second BCA can be differentiated from the third and fourth BCAs by its position anterior to the SCM.

E. 90% of the third BCAs occur on the right side.

A

ANSWER: B

COMMENT: The facial nerve is at risk during the excision of the first BCA.

The second BCA is characterized by a neck mass at the anterior edge of the SCM and has a tract that runs between the internal and external carotid arteries, passes superior to the glossopharyngeal and hypoglossal nerves, and finally ends at the tonsillar fossa.

Second BCAs are the most common anomaly. The second BCA cannot be differentiated from the third and fourth BCAs on the basis of position of SCM alone.

All three anomalies have a lesion anterior to the SCM. They are differentiated based on the path of their tracts.

Moreover, 90% of the third BCAs occur on the left side.

BCAs are common in the pediatric population and are a result of the failure of involution of embryonic structures.

These anomalies may remain asymptomatic, not appreciable on physical examination, and often do not require treatment.

Alternatively, these anomalies can enlarge, which can raise concern for neoplastic process; can become secondarily infected; or can cause cosmetic deformity.

These are all possible indications for surgical excision.

There are four types of BCAs. Each may be associated with a characteristic deep tract.

If the deep tract is incompletely excised, there is an increased risk for recurrence.

The first BCA is not common. It characteristically has a mass located near the lobule of the ear. A CT or MRI is critical to differentiate this lesion from dermoid cysts, lymphatic malformations, parotid masses, or other tumors. The first BCAs typically appear as fluid-filled cystic lesions on CT. During a surgical excision, it is vital to remember that the cyst may have a deep component that may pass either lateral or medial to the facial nerve, which must be identified in the entire first BCA lesion, and is to be safely removed.

The second BCA is the most common of the brachial cleft lesions, and it is characterized by a lesion at the anterior border of the SCM. Its deep tract runs between the internal and external carotid arteries, superior to the glossopharyngeal and hypoglossal nerves, and ends in the tonsillar fossa.

The third BCA is also characterized by a lesion at the anterior border of the SCM. Its associated tract passes posterior to the internal and external carotid arteries, between the glossopharyngeal and hypoglossal nerves, and terminates at the apex of the pyriform sinus.

The fourth BCA additionally presents as a lesion at the anterior border of the SCM. Its tract runs from the apex of the pyriform sinus and travels inferiorly in the tracheoesophageal groove; then, it runs posterior to the thyroid gland and into the thorax where it loops below the aorta on the left and below the subclavian artery on the right. It then ascends posterior to the common carotid artery and loops over the hypoglossal nerve, ending at the anterior border of the SCM.

The spinal accessory nerve exits the skull base via the jugular foramen and enters level II by passing anterior to the jugular vein but deep to the posterior belly of the digastric muscle and the SCM.

The nerve heads in a posteroinferior direction and gives off a branch to the SCM before entering the substance of the SCM and exiting deep to Erb’s point (the point where the greater auricular nerve sweeps around from the posteroinferior aspect of the SCM) and traverses level V in a superficial plane before innervating the trapezius muscle.

For these reasons, CN XI injury risk is the highest while dissecting lesions deep to the SCM and during a level V dissection.

142
Q

A patient is extubated after undergoing a total thyroidectomy. The patient is immediately noted to be stridorous; however, the patient’s voice is normal and strong. The patient requires a 4-L nasal cannula and is saturating 100%. Which of the following statements is true?

A. The patient likely has unilateral true vocal cord paralysis with the vocal cord in a lateralized position. The second true vocal cord has a normal movement.

B. The patient likely has unilateral paralysis with the true vocal cord in the paramedian position. The second true vocal cord has a normal movement.

C. The patient likely has bilateral vocal cord paralysis with bilateral true vocal cords in the paramedian position.

D. Stridor is expected after thyroidectomy.

E. A flexible fiberoptic laryngoscopy is contraindicated in this patient.

A

ANSWER: C

COMMENTS: The patient likely has a bilateral vocal cord paralysis with both true vocal cords found in the paramedian position due to a bilateral RLN injury sustained during a thyroidectomy.

The patient is saturating well on nasal cannula; therefore a flexible laryngoscopy would not be contraindicated.

The laryngeal neuromuscular anatomy is key to understanding the vocal cord position.

The laryngeal musculature can be classified into two groups: abductors and adductors.

Adductors include lateral cricoarytenoid muscle, thyroarytenoid muscle, cricothyroid muscle, and the interarytenoid muscle.

There is only one abducting muscle: the posterior cricoarytenoid muscle.

There are only two nerves that supply motor innervation to these muscles: the RLN and the external branch of the superior laryngeal nerve.

The RLN is responsible for innervating each one of the aforementioned muscles, except the cricothyroid muscle, which is supplied by the external branch of the superior laryngeal nerve.

Therefore when a vocal cord is paralyzed due to iatrogenic trauma to the RLN, innervation to all the muscles except the cricothyroid, an adductor, is lost.

Therefore the true vocal cord sits in an “adducted” position just lateral to midline. This position is called the paramedian position.

When there is bilateral true vocal cord paralysis, both cords sit in a paramedian position, which creates a slit-like airway. This results in a significant stridor.

However, as the true cords are in proximity to each other, the voice can still be normal.

The unilateral vocal cord paralysis typically presents with normal breathing and a normal, breathy, or hoarse voice.

143
Q

A 47-year-old patient presents with a slowly enlarging right parotid mass. You are suspicious for a neoplastic process and perform a fine-needle aspiration (FNA). Which of the following is true regarding parotid neoplasms?

A. Approximately 75%–80% of parotid gland tumors are malignant.

B. The most common parotid tumor is mucoepidermoid carcinoma.

C. Pleomorphic adenoma is seen bilaterally in 50% of patients.

D. An open biopsy of a parotid mass is the preferred method of obtaining a diagnosis and providing treatment.

E. Warthin’s tumor is the second most common benign salivary gland tumor. Smoking is an etiologic risk factor.

A

ANSWER: E

COMMENTS: Around 75%–80% of parotid tumors are benign.

The most common parotid tumor is pleomorphic adenoma.

Warthin’s tumor, not pleomorphic adenoma, is seen bilaterally in 12% of patients with a Warthin’s tumor.

A surgical biopsy is not recommended due to a high risk of recurrence.

Warthin’s tumor is the second most common benign salivary gland tumor and is associated with smoking as a risk factor.

Salivary glands are divided into major and minor glands.

The major glands include the parotid, submandibular, and sublingual glands. These glands are paired.

The minor glands include 600 to 1000 glands found throughout the upper aerodigestive tract.

About 70% of salivary gland tumors occur in the parotid gland, of which 75%–80% are benign.

The most common benign parotid tumor is pleomorphic adenoma followed next in incidence by Warthin’s tumor, which is associated with smoking and an increased rate of being found bilaterally (12%).

The most common malignant neoplasm of the parotid is the mucoepidermoid carcinoma followed by the adenoid cystic carcinoma.

Facial paralysis related to a parotid mass should be seen as a concerning sign that the mass is malignant.

In malignant neoplasms, pain is typically indicative of neural invasion by the tumor and portends a worse prognosis.

However, pain itself is not a predictable indicator of malignancy because benign neoplasms can be associated with pain in the setting of a secondary infection, cystic enlargement, or intralesional hemorrhage.

FNA is an accurate and safe aid for diagnosis. It is primarily used to help guide which patients need surgery, rather than to provide a definitive diagnosis. It is very effective in helping to avoid surgery in patients with reactive lymph nodes, lymphoma, or nonneoplastic disorders.

Surgical biopsies are not recommended because excisional biopsies and capsule violations are associated with an increased risk of recurrence, even with benign masses like pleomorphic adenoma.

The standard surgical approach is to perform a lateral parotidectomy with a cuff of tissue around the mass in addition to the facial nerve dissection and preservation in order to avoid facial nerve injury.

144
Q

Which of the following statements about the larynx is true?

A. Laryngeal cancer is the most common type of head and neck cancer worldwide.

B. The HPV status has overtaken smoking and drinking as the greatest risk factor for the development of laryngeal SCC.

C. The glottis is a subunit of the larynx that consists of the false and true vocal cords.

D. Sensation of the larynx is supplied completely by the RLN.

E. Lymphatic drainage from the supraglottis goes to levels II, III, and IV and can be bilateral.

A

ANSWER: E

COMMENTS: Laryngeal cancer is the second most common cancer worldwide.

The greatest risk factor for laryngeal cancer remains smoking and drinking; the HPV status is the most important risk factor pertaining to oropharyngeal SCC (OPSCC).

The glottis contains the true cords and extends 1 cm inferiorly. The false cords are a supraglottic structure.

Sensation to the larynx is supplied by both the internal branch of the superior laryngeal nerve and the RLN.

The supraglottis lymphatic drainage tends to be bilateral and to levels II, III, and IV.

Epidemiologically, laryngeal cancer is the second most common cancer of the head and neck worldwide, after oropharyngeal cancer. There is a male-to-female preponderance of 6:1.

The most important risk factors for the development of laryngeal SCC are smoking and drinking, and these have a multiplicative effect.

HPV has also been associated with the development of laryngeal SCC (HPV types 16 and 18) but not as strongly as in OPSCC.

Occupational exposure to wood dust, polycyclic hydro- carbons, and asbestos is an additional risk factor.

Anatomically, the larynx consists of three subunits: the supraglottis, glottis, and subglottis.

The supraglottis extends from the epiglottis to the ventricular apices and encompasses the false cords, aryepiglottic folds, arytenoids, and laryngeal surface of the epiglottis.

The glottis consists of the true vocal cords, the anterior commissure, the interarytenoid region, and the floor of the ventricles and extends 1 cm inferiorly below the apex of the ventricle.

The subglottis extends from 1 cm below the ventricle to the inferior border of the cricoid cartilage.

The blood supply to the supraglottis is delivered via the superior laryngeal artery, which is a branch of the superior thyroid artery.

The blood supply to the glottis and subglottis is supplied by the inferior laryngeal branch of the inferior thyroid artery.

Lymphatic drainage of the supraglottis is supplied by a rich network.

The supraglottis tends to drain to level II but can extend to levels III and IV. Bilateral drainage can occur.

The glottis has essentially no lymphatic drainage. The subglottis drains primary to level VI.

Sensation above the true cords is supplied by the internal branch of the superior laryngeal nerve, while the sensation of the true cords is supplied by the RLN.

The external branch of the superior laryngeal nerve provides motor innervation to the cricothyroid muscle.

The RLN is responsible for the motor innervation to the remainder of the laryngeal musculature.

145
Q

Which of the following statements is true about neck
dissection?

A. A radical neck dissection removes all nodal levels in the cervical neck as well as the SCM muscle, external jugular vein, and spinal accessory nerve.

B. An important landmark for finding the hypoglossal nerve during neck dissection is the transverse process of C2.

C. The spinal accessory nerve most frequently passes anteriorly to the jugular vein in level II before giving off a branch to the SCM muscle.

D. The hypoglossal nerve exits the base of the skull and passes posterior to both the internal jugular vein and the external carotid artery and its branches before passing deep to the posterior belly of the digastric muscle to enter the submandibular triangle.
E. Level III extends from the hyoid bone to the level of the thyroid cartilage.

A

ANSWER: C

COMMENTS: A radical neck dissection is defined as the removal of all nodal levels in addition to the SCM, internal jugular vein, and spinal accessory nerve.

The transverse process of C2 is an important landmark for the identification of the spinal accessory nerve.

The spinal accessory nerve most frequently passes anteriorly to the inter- nal jugular vein in level II.

The hypoglossal nerve passes posterior to the internal jugular vein but anterior to the external carotid and its branches.

Level III extends from the hyoid bone to the inferior border of the cricoid cartilage.

Radical neck dissection is rarely performed today due to significant morbidity such as shoulder droop, an increased risk of facial swelling due to the sacrifice of the internal jugular vein, and an increased risk of carotid blowout due to less superficial tissue coverage after removal of the SCM muscle.

It is still performed if there are clinically positive modes with the involvement of the SCM muscle, CN XI, and internal jugular vein.

A modified radical neck dissection is defined as the removal of nodes but spares CN IX in type I or CN IX and the internal jugular vein in type II.

The selective neck dissection, which is the most commonly performed, does not remove all nodal levels, but specifically those at most risk depending on the site of the primary tumor.

The SCM muscle, CN IX, and internal jugular vein are spared.

Nodal levels have specific boundaries.

Level IA includes the submental triangle and is bound by the anterior belly of the digas- tric muscle bilaterally, the mandible anteriorly, and the hyoid inferiorly.

Level IB includes the submandibular triangle and is bound by the anterior belly of the digastric medially, the mandible superiorly, the stylohyoid ligament laterally, and the hyoid bone inferiorly.

Level II includes the jugular digastric region from the level of the posterior belly of the digastric down to the hyoid bone. Level II is further divided into B and A segments by the spinal accessory nerve—level IIB is the segment superior and level IIA is the segment inferior.

Level III extends from the hyoid bone to the inferior border of the cricoid cartilage.

Level IV nodes extend from the inferior border of the cricoid cartilage to the clavicle.

Level V is bound by the posterior border of the SCM anteriorly and the anterior border of the trapezius posteriorly.

Level IV contains the pretracheal and paratracheal nodes found inferior to the thyroid gland.

146
Q

Which of the following statements about the oral cavity is true?

A. The upper lip demonstrates significant lymphatic crossover. As a result, upper lip neoplasms tend to metastasize to bilateral level IB, periparotid lymph nodes, and level II lymph nodes.

B. The floor of the mouth is composed of a series of muscles. From inferior to superior these include anterior belly of the digastric, the mylohyoid muscle, and the geniohyoid muscle.

C. The intrinsic muscles of the tongue are responsible for gross tongue movement while the extrinsic muscles of the tongue are responsible for intricate mobility required for speech and deglutition.

D. The lingual nerve supplies motor innervation to the tongue.

E. Taste and sensation from the anterior two-thirds of the tongue are supplied via the mandibular division of the trigeminal nerve.

A

ANSWER: B

COMMENTS: The upper lip tends to drain ipsilaterally, not bilaterally.

The floor of the mouth is composed of the anterior belly of the digastric, the mylohyoid muscle, and the geniohyoid muscle from inferior to superior.

The extrinsic muscles of the tongue are responsible for gross tongue movements, while the intrinsic muscles of the tongue are responsible for intricate mobility.

The hypoglossal nerve supplies tongue motor innervation.

Taste to the anterior two-thirds of the tongue is supplied by the facial nerve via chorda tympani and the lingual nerve while sensation to this part of the tongue is supplied by the mandibular division of the trigeminal nerve.

The oral cavity is a complex anatomic space composed of seven distinct subunits. These subunits are important to understand as distinct entities because they each have specific lymphatic drainage pathways and different propensities for local invasion.

The oral cavity extends from the vermilion borders of the lips anteriorly to its posterior limit, which is demarcated by the vertical line drawn from the junction of the hard and soft palate to the circumvallate papillae.

Laterally, the boundary is the buccal mucosa. The seven subunits are the lips, alveolar ridge, floor of the mouth, oral tongue, hard palate, retromolar trigone, and the lateral buccal mucosa.

The arterial supply to the lips is provided by the labial arteries, which are a branch off the facial artery.

The lower lip drains to levels IA and IB and tends to have contralateral drainage. The upper lip tends to drain ipsilaterally to level IB, periparotid lymph nodes, and level II lymph nodes.

The lower alveolar ridge contains the tooth-bearing component of the mandible. This is a thin bone that tends to involute when dentition is lost.

The alveolar ridge blood supply is provided by endosteal and periosteal vessels. Lymphatic drainage is based on the anatomic position.

Anterior lesions drain to level I and have bilateral drainage, while lateral lesions tend to drain to the ipsilateral levels IB and II.

The floor of the mouth extends from the attached gingiva of the medial alveolar ridge to the muscular tongue and extends posteriorly to the palatoglossus muscle.

The primary vascular supply is via the lingual artery and branches of the facial system.

The floor of the mouth lymphatic system has superficial and deep drainage pathways.

The superficial system drains bilaterally while the deep system drains to ipsilateral level I and the upper jugular chain.

The oral tongue consists of three paired extrinsic muscle groups that achieve gross tongue movement: genioglossus, hypoglossus, and styloglossus.

The intrinsic musculature includes the lingual muscles as well as the vertical and transverse muscular units, which work together to perform high-fidelity tongue move- ments that assist in speech and swallow.

Motor innervation is supplied via the hypoglossal nerve.

Taste from the oral tongue is supplied by way of the chorda tympani/lingual nerve system. Lymphatic drainage depends on the anatomic position.

Lesions on the tip of the tongue and midline dorsum tend to spread bilaterally as well as to level IA. Lateral oral tongue lesions tend to spread to the ipsilateral level IB and level II.

The upper alveolar ridge receives blood supply from the facial artery and internal maxillary artery system.

Lymphatic drainage tends to be to ipsilateral level IB, periparotid lymph nodes, and level II.

The hard palate tends to drain to levels I and II.

It contains three paired foramina that allow for potential tumor spread: the greater palatine, the lesser palatine, and the incisive foramen.

The retromolar trigone is a mucosally covered triangular area overlying the ascending ramus of the mandible.

The retromolar trigone mucosa is tightly adherent to the mandible; therefore even small mucosal lesions can easily spread to the mandible.

Lymphatic drainage is to ipsilateral levels I and II. The lateral buccal mucosa lymphatics drain to levels I and II and periparotid lymph nodes.

147
Q

A 68-year-old male presents with the lesion pictured in Fig. 30.1. An FNA in the office is performed and is consistent with SCC. A CT soft tissue neck is obtained, which shows the lesion has a depth of invasion of 4.5 mm. There is no radiographic or clinical neck lymphadenopa- thy, and the remainder of the metastatic workup is negative. What is the appropriate management for this patient?

A. Partial glossectomy with ipsilateral radical neck dissection

B. Total glossectomy only

C. Radiation only

D. Hemiglossectomy with ipsilateral radical neck dissection

E. Partial glossectomy with ipsilateral selective neck dissection

A

ANSWER: E
COMMENTS: The patient has a lateralized tongue SCC with an invasion of 4 mm in depth.

Adequate margins should be attainable with a partial glossectomy rather than a hemiglossectomy or total glossectomy.

This will preserve articulation and swallow function.

The patient, even though currently with an N0 neck, is at a high risk of developing cervical metastasis due to a depth of invasion of 4.5 mm.

An ipsilateral selective neck dissection is recommended.

SCC of the tongue tends to occur in middle-aged males with a history of tobacco and alcohol use.

However, there is a subset of patients who tend to be 20 to 40 years of age with no known risk factors.

Cancer in this group tends to be more aggressive and carries a poor prognosis despite appropriate therapy.

The rate of cervical node metastasis is high, even in T1 or T2 primary cancers.

Multiple studies have found that the increasing rate of metastasis is seen in tumors that have a vertical depth of invasion greater than 4 mm.
The initial treatment can be surgical or with radiation.

A surgical excision is favored for most patients due to the ability to obtain adequate margins while still preserving articulation and the ability to swallow.

T1 and T2 cancers can be treated with a partial glos- sectomy with the goal of attaining a 1-cm cuff of normal tissue around the lesion in all directions.

T3 and T4 lesions tend to involve adjacent structures such as the floor of the mouth or mandible.

These lesions tend to require a hemiglossectomy or total glossectomy to achieve an adequate margin.

Further, these patients may require a segmental mandibulectomy or floor of the mouth resection to adequately resect the tumor.

Management of the clinically/radiographically N0 neck tends to be more aggressive in even T1/T2 cancers. The reason for this is that even early-stage cancer tends to have a high rate of nodal metastasis, with some studies reporting a rate as high as 30%.

If the depth of invasion is 4 mm, a selective neck dissection should be recommended in the clinically/radiographically N0 neck and should even be considered if the depth of invasion is at least 3 mm.

Selective neck dissection should include levels I–IV in a clinically/ radiographically negative neck.

Level IIB does not need to be dissected unless metastasis is evident intraoperatively.

148
Q

A 58-year-old female develops an increased work of breathing and stridor 10 h after undergoing a C3–5 anterior cervical diskectomy and fusion. Multiple attempts at intubation with GlideScope and fiberoptic technique fail, and a bedside cricothyroidotomy is performed. Which of the following is true regarding cricothyroidotomy?

A. Surgical revision to a formal tracheotomy should be performed within 48 h, if clinically acceptable, due to increased risk of subglottic stenosis with prolonged cricothyroidotomy.

B. Surgical revision to a formal tracheotomy should be performed due to increased risk of supraglottic stenosis with prolonged cricothyroidotomy.

C. Surgical revision to a formal tracheotomy should be performed due to the increased risk of laryngotracheal separation with prolonged cricothyroidotomy.

D. Cricothyroidotomy is accomplished by incising the thyroid cartilage in the midline to create enough space for tube placement.

E. Cricothyroidotomy is indicated in neonates and children.

A

ANSWER: A

COMMENTS: Surgical revision to a formal tracheotomy should be performed due to an increased risk of subglottic stenosis.

Laryngotracheal separation is not associated with cricothyroidotomy.

Cricothyroidotomy is accomplished by making an incision through the cricothyroid membrane, not the thyroid cartilage.

It is contra- indicated in children due to a high risk of cricoid injury and sub- glottic stenosis as well as a low likelihood of successfully entering the airway.

Cricothyroidotomy is an important maneuver when intubation is not possible in an emergent setting.

It is performed by placing the patient in a supine position with shoulder roll to assist with neck extension.

A skin preparation solution is used if time permits.

The thyroid cartilage is then grasped between the thumb and second and third fingers of the nondominant hand.

A vertical incision is then made with a number 11 or 15 blade to expose the cricothyroid membrane.

A vertical incision is made to avoid injury to the anterior jugular veins.

Further, it will allow for more exposure of the airway. Once exposed, a horizontal incision is made through the membrane, staying closer to the superior border of the cricoid to avoid injury to the cricothyroid vessels.

If avail- able, a tracheal dilator or clamp (Kelly, hemostat, mosquito) can be used to dilate the cricothyroidotomy.

If available, a cuffed tracheostomy tube can then be placed through the cricothyroidotomy and a ventilating device attached.

Once breath sounds are confirmed, the tracheostomy tube should be secured with sutures, tracheostomy tube ties if available. If no tracheostomy tube is available, any tube that will keep the airway patent and attach to a ventilating system (such as an endotracheal tube) is appropriate.

Once secured, the cricothyroidotomy should then be converted to a formal tracheostomy within 48 h due to an increased risk of subglottic stenosis.

There is an increased risk because the crico- thyroid space is 7 mm in vertical dimension.

Since the cricothy- roid space is usually smaller than the outer diameter of most tracheostomy tubes (6 Shiley disposable cuffed trach tube (DCT) is 10 mm in outer diameter), the risk of cartilage injury is high.

A combination of pressure against the cricoid cartilage, shearing forces from the cricothyroid muscle tensing, and concomitant infection create a favorable environment for chondritis, necrosis, and, ultimately, subglottic stenosis.

149
Q

Which of the following statements about the oropharynx is true?

A. Major risk factors for the development of OPSCC do not include smoking and drinking.

B. The anatomic subunits within the oropharynx include the soft palate, the palatine tonsillar fossa and pillars, the base of tongue, and the epiglottis.

C. HPV 23 and 40 are the most strongly associated HPV subtypes in an oropharyngeal cancer.

D. HPV-positive OPSCC is rarely identified.

E. Patients diagnosed with HPV-positive oropharyngeal carcinoma are more responsive to treatment and have a survival advantage.

A

ANSWER: E

COMMENTS: The major risk factors for OPSCC include HPV status, smoking, and drinking.

The anatomic subunits of the oropharynx include the soft palate, the tonsillar fossa and pillars, the base of tongue, and the pharyngeal walls, while the epiglottis is a part of the larynx.

HPV 16 is the most strongly associated subtype with OPSCC. HPV-positive SCC is commonly identified in OPSCC.

Patients with HPV-positive SCC are more responsive to treatment and have a survival advantage.

The oropharynx is the middle third of the pharynx and con- nects with the nasopharynx superiorly and hypopharynx inferiorly.

Its superior and inferior limitations are defined by horizontal lines drawn through the hard palate and hyoid bone, respectively, while its anterior border is a vertical line drawn through the hard palate/ soft palate junction through the circumvallate papillae.

Its posterior border is the posterior pharyngeal wall, which lies anterior to the prevertebral fascia.

Its lateral boundary includes the tonsillar fossa and pillars as well as the lateral pharyngeal walls.

The oropharynx consists of four subunits: the soft palate, tonsillar fossa and pillars, base of tongue, and pharyngeal walls.

Historically, the most important risk factor for the development of OPSCC was exposure to tobacco and alcohol.

Smokers have a 5 to 25 times greater risk of developing head and neck cancer than nonsmokers.

The relative risk of developing SCC rises to 8.8 in those who consume 30 alcoholic beverages per week compared with a relative risk of 1.2 in those who consume 1 to 4 drinks per week.

Smoking and drinking have a cumulative effect: for instance, a patient who has a 40-pack-year smoking history and drinks 5 alcoholic beverages daily has a relative risk of 40.

In the last three decades, there has been a demographic shift in the etiologic agent in the development of SCC.

HPV is now the most common cause. HPV 16 is the most common detected subtype.

HPV-positive tumors are more likely to originate in the oropharynx, be poorly differentiated, and have basaloid features.

They tend to occur in younger and healthier patients and tend to present with a lower T stage but a higher N stage.

HPV-positive tumors are independently associated with sexual behavior and marijuana use in some studies.

These patients have a greater response to treatment and have a survival advantage compared with HPV-negative tumors.

150
Q

A 64-year-old male with a 50-pack-year smoking history presents with right ear pain, odynophagia, and hoarseness for a duration of 3 months. He initially presented to his primary care physician, who prescribed a course of Augmentin for sus- pected upper respiratory tract infection with acute otitis media. He reports this treatment helped for a while, but his symptoms quickly returned. On examination, the patient has inspiratory stridor, prolonged inspiratory phase, and dysphonia. A flexible laryngoscopy shows a right glottic mass, causing fixation of the right true vocal cord and near-complete obstruction of the airway. What is the next best step in management?

A. Perform an office biopsy of the lesion in order to obtain diagnosis.

B. Obtain CT soft tissue neck and chest with IV contrast and have the patient follow up in clinic as soon as the scans are complete to discuss further management.

C. Proceed to the OR for standard bag mask induction with GlideScope intubation followed by a direct laryngoscopy, a biopsy of the laryngeal mass, and a tracheotomy.

D. Proceed to the OR to perform an awake tracheotomy followed by a direct laryngoscopy and a biopsy of the laryngeal mass.

E. Proceed to the OR for a rapid sequence induction flexible fiberoptic intubation followed by a direct laryngoscopy and a biopsy of the laryngeal mass, selective neck dissection, and tracheotomy.

A

ANSWER: D

COMMENTS: The patient has a history and physical examination consistent with SCC of the larynx.

The diagnosis should be obtained with a biopsy. Furthermore, CT scans must be obtained to determine the extent of the patient’s disease and to determine staging.

However, the primary act in management must be to secure the patient’s airway in as safe a manner as possible.

The best option for this patient is to undergo an awake tracheotomy.

Putting the patient to sleep prior to an attempted intubation would not be recommended as his obstructing laryngeal mass would likely prevent him from being ventilated via bag mask ventilation.

Further, an obstructing mass would make it very difficult to pass an endotracheal tube successfully.

Laryngeal cancer commonly presents with dysphonia and odynophagia.

Clinicians must also be suspicious if an adult patient presents with odynophagia or ear pain that is present for more than 2 to 3 weeks.

Ear pain is a common presenting sign due to referred pain from the throat via Jacobson’s nerve and Arnold nerve.

Jacobson’s nerve, a branch of CN IX, directly innervates the ear but also has pharyngeal, lingual, tonsillar, parapharyngeal space, and retropharyngeal space innervation.

Any pathologic process involving these areas can result in referred otalgia.
Arnold nerve, a branch of CN X, supplies the vallecula, lingual, and laryngeal surfaces of the epiglottis, supraglottic larynx, and pyriform sinuses.

The Arnold nerve also directly innervates the ear, which can result in referred otalgia.

151
Q

A patient with a history of laryngeal cancer initially treated with radiation therapy presents with a T4 recurrence. A total laryngectomy is performed. The patient’s pharyngeal defect is closed using the myocutaneous regional pedicled flap shown in Fig. 30.2. What is the blood supply to this flap?

A. Branch from the thoracoacromial artery

B. Thoracodorsal artery

C. Perforating vessels from the occipital artery

D. Superficial temporal artery

E. Perforating vessels of the internal mammary artery

A

ANSWER: A

COMMENTS: The image is of a pectoralis myocutaneous flap based on the pectoral branch of the thoracoacromial artery.

The thoracodorsal artery supplies the latissimus dorsi myocutaneous flap. The perforating vessels of the occipital artery supply the supe- rior one-third of an SCM myocutaneous flap.

The superficial temporal artery supplies the temporoparietal fasciocutaneous flap.

The perforating vessels of the internal mammary artery supply the deltopectoral fasciocutaneous flap.

Patients undergoing a total laryngectomy require a pharyngeal closure.

In patients who have not previously received radia- tion, this can typically be done primarily.

However, in patients who have received radiation, the local vasculature is tenuous and these patients are at a higher risk of breakdown of the pharyngeal closure without bolstering the pharyngeal closure with healthy tissue.

The pectoralis major flap is a regional pedicled flap that is used frequently for its reliability and excellent reach (can reach up to midface, some report to lateral canthus). Its blood supply is from the pectoral branch of the thoracoacromial artery.

152
Q

A patient with a SCC of the left lower alveolus undergoes a segmental mandibulectomy of a 6-cm segment of bone. Which reconstructive option is the most appropriate?

A. Radial forearm osseocutaneous free flap

B. Latissimus dorsi myocutaneous free flap

C. Anterolateral thigh fasciocutaneous free flap

D. Gracilis muscle free flap

E. Fibula osseocutaneous free flap

A

ANSWER: E

COMMENTS: In the reconstruction of a segmental mandibulectomy, the surgeon must reconstruct the mandible using the strong bone.

A radial forearm osseocutaneous flap provides a bone, but the bone is thin. The latissimus, anterolateral thigh, and gracilis flaps cannot be harvested with a bone and cannot be used to repair a mandibular defect.

The best option for reconstruction is the fibula osseocutaneous free flap.

The free fibular graft provides the longest possible segment of revascularized bone (up to 25 cm). It can span nearly any mandibular defect. The flap is supplied by the peroneal artery and accompanying veins.

153
Q

A 52-year-old male with no other medical history presents with a 4-cm right level II neck mass. An office FNA is performed and is consistent with SCC. A thorough physical examination including a flexible laryngoscopy does not indicate an obvious primary tumor. The P16 status is pending. Which of the following is true?

A. An appropriate next step in the management would be to perform a direct laryngoscopy and esophagoscopy with a biopsy of any suspicious lesions. A right tonsillectomy should be performed if no clear lesions are found else- where, and a left-sided tonsillectomy should be considered.

B. Esophagoscopy is unnecessary.

C. Bilateral tonsillectomy and base of tongue resection are indicated for the initial diagnostic workup.

D. A positron emission tomography (PET) scan is the only indicated imaging.

E. The P16 status will not provide prognostic information.

A

ANSWER: A

COMMENTS: A direct laryngoscopy and esophagoscopy would be the next appropriate step.

Any suspicious lesions should be biopsied.

If no obvious primary site is found during this examination, a right tonsillectomy should be performed and a left tonsillectomy considered.

A base of tongue resection would not be indicated at this stage in the patient’s workup.

However, a PET scan can be useful; anatomic studies such as CT soft tissue neck or MRI neck should be obtained to fully evaluate for any suspicious lesions.

A CT chest with IV contrast should also be obtained to assess for any metastatic disease.

P16 positivity would indicate the HPV status.

Patients presenting with a head and neck cancer of unknown primary make up about 2%–9% of all patients with head and neck cancer.

The diagnosis of a primary tumor is of crucial significance because surgical and oncologic management of the primary lesion may be efficiently coordinated with the management of the neck disease that has improved the survival rates from 44% up to 100%.

Further, the type and degree of treatment change based on whether a primary tumor is found.

The typical diagnostic protocol includes a physical exami- nation, CT neck and CT chest with IV contrast, and pan- endoscopy in the OR under general anesthesia (including laryngoscopy and esophagoscopy) with tonsillectomy and biopsies of suspicious mucosal areas.

The CT/MRI detection rate for a primary tumor varies from 9% to 23%.

The likelihood of discovering a primary tumor by an endoscopy and random biopsies is low.

Without suspicious findings in clinical or radiologic exami- nations, a successful result can be obtained in 20%–29% of cases in some studies.

When an endoscopic biopsy can be directed according to a suspicious clinical or radiologic finding, a detection rate of up to 60% has been reported.

Ipsilateral tonsillectomy yields an 18%–44% rate in primary tumor.

Some authors and institutions support bilateral tonsillectomy, because bilateral or contralateral tumors have been found in up to 10%–23% of cases.

154
Q

A 60-year-old woman underwent surgery for a 2.5cm follicular neoplasm diagnosed by FNAB. The frozen section turned to be benign and a lobectomy was carried out. The final histopathology however is papillary carcinoma with perithyroidal soft tissue invasion. In this patient:

A. No further surgery is immediately indicated

B. RAI therapy should be instituted

C. A completion thyroidectomy should be done

D. Suppression therapy should be started immediately.

E. A completion thyroidectomy with central node dissection

A

?

155
Q

In medullary type thyroid carcinoma with a lesion of 2cm but without clinically palpable cervical lymph node, which one of the following management would be ideal?

A. Total thyroidectomy and close surveillance after

B. Total thyroidectomy with Radioactive Iodine Ablation therapy

C. Total thyroidectomy with central lymph node dissection

D. Bilateral modified radical neck dissection after total thyroidectomy

A

?

156
Q

A 30-year-old male with a 1.5cm parotid mass located in the deep lobe, firm, movable, no enlarged cervical nodes and no paralysis of facial muscles. What is the most appropriate procedure?

A. Superficial parotidectomy

B. Total parotidectomy with nerve preservation

C. Total parotidectomy with neck dissection

D. Deep parotid lobectomy

E. Enucleation of mass

A

?

157
Q

Three months after surgery for a mixed benign tumor of the parotid gland, the patient develops sweating in the parotid area when eating. This syndrome is due to injury to the:

A. Zygomatic branch of the facial nerve

B. Buccal branch of the facial nerve

C. Mandibular branch of the facial nerve

D. Auriculo-temporal nerve

A

?

158
Q

A 36-year-old male underwent right submandibular sialadenectomy, the surgeon inadvertently transected the lingual nerve. In this case, there will be

A. Loss of sensation to the posterior two-thirds of the tongue

B. Loss of sensation in the anterior two-thirds of the tongue

C. Loss of sensation and taste in the anterior two-thirds of the tongue

D. Loss of sensation and taste in the entire tongue

A

?

159
Q

A 55-year-old male smoker, otherwise healthy, has a 3cm ulcer on the left lateral border of the anterior aspect of his tongue with no involvement of the floor of the mouth or mandible. Biopsy demonstrates SCC with a 6mm depth of tumor invasion. CT of his neck shows no evidence of cervical lymphadenopathy. His remaining metastatic work-up is negative. Which of the following choices is the best option for treatment?

A. Concomitant chemotherapy and external beam radiation

B. Partial glossectomy with left supraomohyoid neck dissection

C. Total glossectomy with left supraomohyoid neck dissection

D. Primary chemotherapy with 5-fluorouracil and cisplatin

A

?

160
Q

A T4N0M0 SCC in the midline anterior floor of the mouth is recently diagnosed in a 72-year-old man. The tumor is approximately 4.3cm in diameter with invasion into the anterior aspect of the cortical bone of the mandible. What is the best option for treatment in this patient?

A. Local resection with curettage of involved bone

B. Composite tumor resection with total mandibulectomy and bilateral radical neck dissection

C. Primary external beam radiation therapy

D. Composite tumor resection with segmental mandibulectomy and bilateral selective neck dissection

E. Primary chemotherapy

A

?

161
Q

A 20-year-old male complaining of progressive enlargement of the horizontal ramps of the mandible with an x-ray showing a lesion with a soap bubble appearance. In this patient:

A. Lesion is likely to be a dentigerous cyst

B. Curettage and bone grafting is indicated

C. Excision with or without bone grafting is appropriate

D. Radiation therapy is the treatment of choice.

A

?

162
Q

A 70-year-old male presents with an isolated 5cm metastatic cervical node at level V, right. Biopsy revealed SCCA. The appropriate therapy for this patient is

A. Multimodal therapy: surgery, radiation, chemotherapy

B. Right RND

C. Brachytherapy, nasopharynx and right neck

D. RT from nasopharynx to neck

E. Concurrent chemotherapy and RT from nasopharynx to neck

A

?

163
Q

A 67-year-old man has a 6-month history of right level II neck mass. FNA biopsy is performed and demonstrates SCC. Office examination of the head and neck fails to demonstrate a primary tumor site. CT confirms the enlarged node with no additional information. PET also shows increased uptake at the site of the lymph node with no other activity noted. What is the next most appropriate step in management?

A. Endoscopy under anesthesia with guided biopsies and bilateral tonsillectomy

B. Neck dissection

C. Irradiation of the next and upper aerodigestive tract

D. Excisional node biopsy

E. Repeated FNA

A

?

164
Q

An 85-year-old hypertensive man is evaluated in the ER for recent onset epistaxis. His BP is 150/80 and Hct is 39%. What is the most likely source of bleeding?

A

The source of 90% of anterior nosebleeds is within Kiesselbach’s plexus (also known as Little’s area) on the anterior nasal septum. There are five named vessels whose terminal branches supply the nasal cavity:

1) Anterior ethmoidal artery
2) Posterior ethmoidal artery
3) Sphenopalatine artery
4) Greater palatine artery
5) Superior labial artery

The watershed area of these five vessels is in the anterior nasal septum, comprising Kiesselbach’s plexus. This lies at the entrance to the nasal cavity and so is subject to extremes of heat and cold, and of high and low moisture, and is easily traumatized. The mucosa over the septum in this area is especially thin, making this the site of the majority of epistaxis.

165
Q
  1. An 85-year-old ventilator-dependent male
    was endotracheally intubated 10 days ago. He
    remains unresponsive and is not a candidate
    for early extubation. The intensive care unit
    (ICU) attending elects to perform tracheotomy
    at the bedside. During the procedure, copious
    dark blood is encountered. This is most likely
    due to transection of which of the following:
    (A) Anterior jugular vein
    (B) External jugular vein
    (C) Internal jugular vein
    (D) Middle thyroid vein
    (E) Inferior thyroid vein
A
  1. (A) The anterior jugular vein can cross the midline overlying the proximal trachea. Midline
    cervical dissection without adequate visualization can injure the vein and require open surgical repair. The other veins do not cross the
    midline and are not generally at risk in tracheotomy.
166
Q
2. A 43-year-old teacher underwent left parotidectomy. Upon awakening from surgery, paralysis
of the left lower lip was observed. This complication was most likely due to injury to which
of the following:
(A) Parotid duct
(B) Facial nerve - temporal branch
(C) Facial nerve - cervical branch
(D) Facial nerve - main trunk
(E) Platysma muscle
A
  1. (C) The cervical branch of the facial nerve
    innervates the lower lip through the marginal
    mandibular branch of the nerve (Fig. 3–7). As
    no cross innervation exits to other branches of
    the facial nerve, marginal mandibular branch
    injuries always yield paralysis of the same side
    of the lower lip. Injuries of the main trunk of
    the facial nerve or its temporal branch would
    usually produce upper facial paralysis as well.
167
Q
  1. A 70-year-old male complains of progressive
    weight loss and hoarseness. Ear, nose, and
    throat (ENT) evaluation reveals right vocal cord
    paralysis and several right neck masses, which
    fine needle aspiration reveals to be squamous
    cell carcinoma. The patient undergoes right
    hemilaryngectomy and right radical neck dissection. Postoperatively, right hemidiaphragm
    paralysis is noted. This is due to injury of which
    of the following:
    (A) Vagus nerve
    (B) Brachial plexus
    (C) Cervical plexus
    (D) Spinal accessory nerve
    (E) Phrenic nerve
A
  1. (E) The phrenic nerve is the only component
    of the cervical plexus, which is not sacrificed
    during a radical neck dissection. It can be
    identified as superficial to the anterior scalene muscle (Fig. 3–8) with a nerve stimulator, although direct visualization is usually sufficient. Injury to the phrenic nerve can result
    in paralysis of the corresponding side of the
    diaphragm.
168
Q
  1. A 65-year-old woman complains of severe,
    acute onset left temporal headache and changes
    in left eye vision. She presents to her physician
    with sweating, malaise, and temperature of
    99ºF. Medical evaluation reveals:
    (A) Myocardial infarction (MI)
    (B) Pneumonia
    (C) Diabetes
    (D) Cerebral vascular accident
    (E) Temporal arteritis
A
  1. (E) Temporal artery inflammation causes severe
    throbbing unilateral temporal headache, lowgrade fever, visual changes, sweating, and
    malaise. Acute treatment with prednisone can
    be followed by temporal artery resection.
169
Q
  1. Tracheotomy is performed uneventfully in a
    79-year-old ventilator-dependent encephalopathic male. After several spontaneous breaths,
    however, the patient stops breathing. The anesthesiologist continues to assist the patient’s
    breathing for several minutes, after which the
    patient again breathes spontaneously. The most
    likely cause of apnea is:
    (A) A mucus plug blocked the tracheotomy
    tube.
    (B) Bleeding in the trachea.
    (C) Preoperative respiration was driven by
    hypoxia.
    (D) The patient was allergic to Latex.
    (E) Surgery created a tracheoesophageal
    fistula.
A
  1. (C) Hypoxia can become the primary stimulation for respiration. Sudden elevation of PO2
    can eliminate the respiratory drive, which will
    resume by lowering the PCO2
    .
170
Q
  1. While conversing with admirers at a postconcert cocktail party, a trumpet player complains
    of acute onset intermittent hoarseness and nonproductive cough. Subsequent medical evaluation reveals:
    (A) MI
    (B) Vocal cord paralysis
    (C) Pneumonia
    (D) Vocal cord polyp
    (E) Supraglottic prolapse
A
  1. (E) Prolapse of mucosa overlying the true vocal
    cords (ventricle of Morgagni) can occur in
    people who routinely elevate air pressure in
    the chest and larynx. Treatment involves direct
    laryngoscopy and excision of the protruding
    mucosa. The other choices, although less likely,
    must also be considered.
171
Q
  1. While lecturing to her advanced psychology
    students, a 55-year-old college professor complains of acute onset strained, raspy fluctuating voice, forcing her to discontinue her
    lecture and seek urgent ear, nose, and throat
    (ENT) evaluation. Which diagnosis is most
    likely:
    (A) Vocal cord paralysis
    (B) Vocal cord hematoma
    (C) Vocal cord polyp
    (D) Vocal cord spasm
    (E) Vocal cord cancer
A
  1. (D) Spasmodic dysphonia often occurs in both
    men and women over 40-years old who strain
    their voices. Fluctuation with normal speech
    can occur, unlike the other choices presented.
    The vocal cords appear normal on routine
    laryngeal exam. Temporary improvement
    using botulinum toxin is reported.
172
Q
  1. A 16-year-old high school wrestler complains
    of difficulty breathing after being held in a tight
    choke hold. He is rushed to the nearest emergency room, where the ENT consultant performs a fiberoptic laryngoscopy. Most likely
    finding is:
    (A) Unilateral vocal cord paralysis
    (B) Thyroid cartilage fracture
    (C) Thyroid gland bleeding
    (D) Parathyroid gland bleeding
    (E) Laryngeal tumor
A
  1. (A) Compression of the recurrent laryngeal nerve
    due to compression of the cricothyroid joint can
    occur subsequent to choking injury of the neck.
    Although fracture and internal bleeding can also
    occur, these are less likely. Acute onset dyspnea
    secondary to laryngeal tumor is unlikely.
173
Q
  1. A 60-year-old veteran with a 40-pack year
    smoking history underwent supraglottic laryngectomy and right radical neck dissection for
    laryngeal squamous cell cancer. Postoperatively,
    he complained of difficulty swallowing. The
    most likely cause of his symptom was which of
    the following:
    (A) Recurrent cancer
    (B) Recurrent laryngeal nerve injury
    (C) Superior laryngeal nerve injury
    (D) Sternocleidomastoid muscle injury
    (E) Brain metastasis
A
  1. (C) Dysphagia and aspiration can result from
    disruption of the internal branch of the superior
    laryngeal nerve, resulting in sensory loss of the
    larynx. Contralateral superior laryngeal nerve
    compensation usually corrects these symptoms.
174
Q
  1. A 3-year-old child presented to the emergency
    room with thin, gray pus dripping from her
    left nostril. Her foster mother stated that the
    child “always had a cold” for as long as she
    knew her during the past year. Prior treatment
    with oral antibiotics failed to relieve the symptoms. What was the most likely source of the
    chronic discharge?
    (A) Sinusitis
    (B) Tumor
    (C) Foreign body
    (D) Polyp
    (E) Trauma
A
  1. (C) Unilateral nasal discharge in a young child is often due to foreign body in the nose. Although
    the other choices should also be considered in the
    differential diagnoses of rhinorrhea, nasal foreign body in the child is most likely.
175
Q
  1. While shaving, a 45-year-old teacher notices a
    marble-sized mass beneath his left ear. The
    mass is eventually excised, revealing which of
    the following benign parotid gland lesions?
    (A) Glandular hypertrophy, secondary to
    vitamin A deficiency
    (B) Cystic dilation
    (C) Mikulicz’s disease
    (D) Pleomorphic adenoma
    (E) Papillary cystadenoma (Warthin’s tumor)
A
  1. (E) Papillary cystadenoma lymphomatosum is
    also called Warthin’s tumor. It occurs mainly in
    men. The epithelial component is interspersed
    with lymphoid tissue that shows germinal centers. The most common tumor of the parotid
    gland is a pleomorphic adenoma, with papillary cystadenoma (although much less frequent) as the second most common tumor.
    Mikulicz’s disease involves chronic inflammation and swelling of the salivary glands, which
    is benign and usually painless.
176
Q
  1. Following a vacation in Florida, a 43-yearold man notes shortness of breath. He is a nonsmoker. His wife points out that his face
    has become slightly swollen. On examination,
    his blood pressure is normal. His pupils are
    equal and respond to light. Dilated veins are
    noted around the shoulders, upper chest, and
    face. An x-ray of the chest reveals an opacity in
    the superior mediastinum. What is the most
    likely diagnosis?
    (A) Thymoma
    (B) Neurogenic tumor
    (C) Lymphoma
    (D) Teratodermoid tumor
    (E) Pheochromocytoma
A
  1. (B) The most common cause of primary mediastinal tumor is a neurogenic tumor (20–25%),
    and 10% are malignant (more likely in children). They usually arise from an intercostal
    nerve or sympathetic ganglion. Varieties of
    neurogenic tumors include neurilemmoma
    (schwannoma), neurofibroma, ganglioneuroma, and neuroblastoma. Next in frequency
    (of primary mediastinal tumors) are thymoma,
    congenital cysts, and lymphoma. New diagnostic techniques have resulted in detection of
    larger numbers of these lesions.
177
Q
  1. A 64-year-old assistant hair stylist undergoes a
    vaginal hysterectomy under spinal anesthesia.
    Bleeding occurs when an attempt is made to
    separate and exclude the right ureter from the
    operating field. After a short interval, respiratory arrest occurs and intubation must be
    instituted. What is the most likely cause of respiratory arrest during this procedure under
    spinal anesthesia?
    (A) Paralysis of the intercostal muscle
    (B) Paralysis of the diaphragm (phrenic
    nerves)
    (C) Centrally induced mechanism secondary
    to decreased cardiac output
    (D) Diffusion of anesthetic to the level of the
    pons
    (E) Diffusion of anesthetic to the level of the
    medulla
A
  1. (C) Spinal anesthesia induces venous vasodilation because of sympathetic blockade. Venous
    pooling can seriously impair venous return. It is
    the sympathetic blockade and not somatic nerve
    blockade that is responsible for the vasomotor
    and respiratory changes. It is important to ensure that volume depletion is corrected before
    spinal anesthesia, because venous return and
    hence cardiac output are diminished. These
    changes are aggravated by keeping the head up.
178
Q
  1. In the evaluation of a 64-year-old woman
    with fluctuating neurological signs of ptosis,
    eleventh and twelfth cranial nerve palsy, and
    generalized extremity weakness are noted.
    Edrophonium (Tensilon) given intravenously
    results in clinical improvement. A computed
    tomography (CT) scan shows a lesion in the
    anterior mediastinum, and a biopsy confirms
    the presence of a thymoma. She should undergo
    which of the following?
    (A) High-dose steroid administration
    (B) Irradiation of the anterior mediastinum
    (C) Calcium administration
    (D) Thymectomy
    (E) Pneumococcal vaccination
A
  1. (D) The role of thymectomy in treating patients
    with myasthenia gravis who have a thymoma
    is well established. The thymus gland is located
    in the anterior mediastinum and can be
    approached by a cervical or mediastinal
    approach. It arises from the third and fourth
    branchial arches. Thymectomy is frequently
    advised for patients with myasthenia gravis
    who do not have a thymoma; however, there
    are some authorities who would treat these
    patients initially with an anticholinesterase
    drug such as pyridostigmine (Mestinon).
    Corticosteroid therapy may be indicated when
    thymectomy has failed, but it must be undertaken cautiously, because the drug may
    precipitate severe weakness. Pneumococcal
    infections (which may occur after splenectomy
    performed in children) are not a specific complication noted after thymectomy.
179
Q
  1. A 54-year-old construction worker has smoked
    two packs of cigarettes daily for the past 25
    years. He notes swelling in his upper extremity
    and face, along with dilated veins in this
    region. A computerized tomography (CT) scan
    and venogram of the neck are performed. What
    is the most likely cause of the obstruction?
    (A) Aortic aneurysm
    (B) Metastasis
    (C) Bronchogenic carcinoma
    (D) Chronic fibrosing mediastinitis
    (E) Granulomatous disease
A
  1. (C) Bronchogenic carcinoma accounts for
    70–80% of all cases of superior vena cava (SVC)
    obstruction; primary mediastinal tumors are
    the second most common cause. The main
    bronchial lymphatics are located at the tracheal
    bifurcation and immediately to the right and
    left of the trachea. Tuberculosis and mycotic
    infections are the most likely causes of chronic
    fibrosing mediastinitis.
180
Q
  1. During a routine chest x-ray offered by a
    department store to all its employees, a 42-
    year-old business manager is found to have a
    1.5-cm nodule in the upper lobe of the lung
    with a central core of calcium. He has no symptoms. The management of this lesion should
    involve which of the following?
    (A) Transbronchial biopsy
    (B) Percutaneous needle biopsy
    (C) Thoracotomy
    (D) Periodic x-ray, follow-up evaluation
    (E) Mediastinoscopy
A
  1. (D) CT is useful, because it delineates the calcification and shows the pattern of the calcification. There is no need for intervention at this
    stage, because most lesions of this nature are
    probably benign granulomas.
181
Q
  1. A 54-year-old manager of a bank is noted to
    have a solitary 1.5-cm nodule on a routine chest
    x-ray. He is asymptomatic. The most suggestive
    feature of malignancy would be the finding of
    which of the following?
    (A) A lesion in the lingula lobe
    (B) Central calcification
    (C) A laminated calcium pattern
    (D) Indistinct margins
    (E) A lesion in the left lobe
A
  1. (D) By definition, a solitary nodule is one that is
    5 cm or less in diameter. In most series, 60% of
    such lesions are benign, and 40% are malignant.
    The presence of irregular margins, the absence of
    calcification, a recent onset of symptoms, or an
    increase in size of the lesion within a relatively
    short period (several months) indicate the
    greater likelihood of malignancy. Thin-section
    CT scanning may add further information.
182
Q
  1. An asymptomatic 56-year-old man is found on
    routine chest x-ray to have a 2-cm nodule-central tumor in the upper lobe of the right lung.
    The lesion is not calcified. No previous x-rays
    exist. What is the most appropriate initial step
    toward making a diagnosis?
    (A) Fiberoptic bronchoscopy
    (B) Bone scan
    (C) Thoracotomy
    (D) Observation at follow-up examination
    in 6 months
    (E) Mediastinoscopy
A
  1. (A) Bronchoscopy is the initial step, particularly
    if the patient is a smoker and a good risk for surgery. If cancer is confirmed, thoracotomy will
    probably be undertaken. Patient age is an important consideration in the management of a solitary pulmonary nodule; malignancy occurs in
    less than 1% of patients under 35 years of age.
    Benign lesions, such as bronchopulmonary
    sequestration, which usually affects the posterior aspect of the inferior lobes of the lung, must
    be considered. Bronchopulmonary sequestration is usually asymptomatic, unless complications occur.
183
Q
  1. At the age of 46, an accountant has developed
    hoarseness due to an inoperable cancer of the
    left upper lung lobe. He has smoked heavily
    since the age of 14. Which of the following features of cancer of the lung indicates distant
    spread?
    (A) Hypercalcemia
    (B) Cushing-like syndrome
    (C) Gynecomastia
    (D) Syndrome of inappropriate secretion of
    antidiuretic hormone (SIADH)
    (E) Brachial plexus lesion (Pancoast’s
    syndrome)
A
  1. (E) In apical lung cancers, the malignant tumor
    may extend above the thoracic inlet, penetrate
    the suprapleural membrane, and infiltrate the
    structures found at the root of the neck. The first
    thoracic nerve and lower trunk of the brachial
    plexus are most likely to be involved initially, as
    T1 passes along the inner border of the first rib
    to reach the neck. If the sympathetic nerve is
    involved, pupil constriction and ptosis may be
    evident (Horner syndrome). The other listed
    items are all features of the paraneoplastic syndrome associated with lung cancer and do not
    necessarily indicate extranodal metastasis.
    Cushing’s syndrome in lung cancer occurs more
    frequently in men and in an older age group
    and has a more rapid downhill course than typical Cushing’s syndrome. SIADH should be suspected if the patient with a lung lesion develops
    unexplained mental changes and an extremely
    low serum sodium level. Fluid restriction is
    required. Urine osmolarity is low.
184
Q
  1. Surgery is indicated in the initial management
    of lung cancer in the presence of which of the
    following?
    (A) Hypercalcemia
    (B) Vocal cord paralysis
    (C) Superior vena cava syndrome
    (D) Small-cell anaplastic carcinoma
    (E) Chest wall and anterior abdominal wall
    metastasis
A
  1. (A) Hypercalcemia is attributed to the secretion of parahormone from a localized squamous cell carcinoma (paraneoplastic effect); as
    such, improvement may be seen after surgical
    resection. Following extension of the tumor
    into the chest wall, radiotherapy and subsequent extensive resection carried out in selected
    cases may occasionally be indicated. Small-cell
    carcinoma (also known as oat-cell carcinoma)
    accounts for 20–25% of cases of bronchogenic
    carcinoma, arises centrally and tends to metastasize widely. The initial treatment is combination chemotherapy followed by radiotherapy
    in those whose cancer responds.
185
Q
  1. Pneumonectomy for carcinoma of the lung is
    contraindicated with which of the following?
    (A) Total atelectasis of the involved lung
    (B) PCO2 over 60 mm Hg
    (C) Cardiac index (CI) of 3 L/min
    (D) PO2 of 80 mm Hg
    (E) Maximal breathing capacity of 75% of
    predicted value
A
  1. (B) Uncorrected hypercarbia is the major contraindication to total pneumonectomy. Surgery
    is the treatment of choice for nonsmall-cell
    carcinoma of the lung. Although 25% of
    patients with bronchogenic carcinoma may
    undergo thoracotomy, many of these patients
    will have unresectable lesions. Patients with a
    FEV1 less than 2 L, a FVC under 70% of predicted value, and maximal voluntary ventilation (MVV) under 50% are likely to tolerate
    operation poorly. The normal FEV1
    /FVC ratio
    is 0.7 or less; in severe obstructive dysfunction,
    it is under 0.45. Total atelectasis of the lung
    may be associated with obstruction of the main
    bronchus by the tumor.
186
Q
  1. After undergoing a percutaneous needle biopsy,
    a 49-year-old electrical engineer is found to have
    small-cell carcinoma. The chest x-ray shows a
    lesion in the peripheral part of the right middle
    lobe. The patient should be advised to undergo
    which of the following?
    (A) Right lobectomy
    (B) Right pneumonectomy
    (C) Excision of lesion and postoperative
    radiotherapy
    (D) Combination chemotherapy
    (E) Radiotherapy
A
  1. (D) Patients with small-cell carcinoma should
    not be treated initially by thoracotomy. This
    cancer responds favorably to combination
    chemotherapy, but few patients survive for
    more than 1 year. More than 160,000 cases of
    bronchogenic carcinoma are diagnosed in the
    United States per year. It accounts for 33% of all
    cancer deaths in men and 20% of all cancer
    deaths in women. The most common cancers of
    the lung are squamous carcinoma, 30% (tumor
    tends to be central); adenocarcinoma, 30%,
    (tumor tends to be peripheral) small-cell carcinoma, 20% (tumor tends to be central); and
    large-cell carcinoma, 15% (tumor tends to be
    peripheral).
187
Q
  1. While walking to the train station from college,
    a sophomore is accosted and stabbed in the
    chest immediately above the second rib. On
    admission to the hospital, he is bleeding from
    the wound and the blade of the knife is protruding from the skin. A chest x-ray reveals
    that the knife is at the level of the inferior
    margin of the fourth thoracic vertebra. The
    patient has a blood pressure of 100/60 mm Hg.
    Which structure is the most likely cause of
    bleeding?
    (A) Arch of the aorta
    (B) Left ventricle
    (C) Hemizygous vein
    (D) Vertebral artery
    (E) Right subclavian artery
A
  1. (A) The arch of the aorta is above the arbitrary
    line drawn between the manubriosternal joint
    (angle of Louis) and the lower border of the
    fourth thoracic vertebra level. Therefore, it is
    located entirely in the superior mediastinum
    behind the manubrium. The projected line
    passing behind the manubriosternal junction
    and the lower border of T4 is a key surgical
    anatomic landmark of this region. The ascending aorta (anteriorly), descending aorta (posteriorly), and pulmonary trunk are below this
    level. The left recurrent laryngeal nerve curves
    around the ligamentum arteriosus, the arch of
    the azygous vein enters the SVC, and the upper
    third of the esophagus are separated arbitrarily
    from the middle third at this level. On a plain
    film of the chest, the tracheal bifurcation
    (carina) is a useful marker of this line.
188
Q
  1. Four years previously, a 56-year-old fisherman
    underwent thyroidectomy for cancer of the thyroid gland. He is now noted to have a single
    4-cm lesion in the upper lobe of the left lung.
    There is no other evidence of disease, and he is
    in excellent health. Endobronchial biopsy confirms that the lesion is malignant but the organ
    of origin cannot be determined. What should
    he be given?
    (A) Radiotherapy
    (B) Combination chemotherapy
    (C) Attempted curative lung resection
    (D) Exploration of the neck for thyroid
    recurrence
    (E) Androgen therapy
A
  1. (C) Although there is a history of previous thyroid cancer, the presence of a solitary nodule on
    chest x-ray is more likely to represent a primary carcinoma of the lung than a solitary secondary metastasis. In metastasis, the lesions
    are more often multiple, they frequently appear
    bilaterally, and they more commonly present in
    the lower area of the lungs. A CT scan would be
    helpful in delineating the pulmonary findings.
189
Q
  1. A 72-year-old retired miner complains of progressive dyspnea, chest pain, and a 20-lb
    weight loss. He is a nonsmoker. Examination
    reveals clubbing of the fingers. CT scan shows
    a pleural effusion and nodular, irregular thickening of the right lung and involvement of the
    celiac lymph nodes. Cytology, repeated on several occasions, is not helpful. Which test will
    most likely establish the diagnosis?
    (A) Laparoscopy
    (B) Bronchoscopy
    (C) Open pleural biopsy
    (D) Repeat cytology
    (E) Gastroscopy
A
  1. (C) The most likely diagnosis, malignant
    epithelioma, commonly occurs following chronic
    exposure (>20–40 years) to asbestos. We need to
    consider this diagnosis in those employed in
    milling and construction, as well as in workers
    in pipe, textiles, gaskets, and other industries in
    which asbestos (especially crocidolite form) is
    used. In 75%, the diffuse (malignant) form
    occurs; less than 25% will survive more than 1
    year after the diagnosis is established. In
    advanced disease, lesions below the diaphragm
    are frequently encountered.
190
Q
  1. A 28-year-old bank employee undergoes investigation for infertility that revealed oligospermia. On further inquiry, it is found that he has
    suffered from repeated bouts of coughing since
    childhood and episodes of recurrent pancreatitis. Clubbing of the fingers is evident. Which
    test is most likely to reveal the cause of his
    chronic lung disease?
    (A) Chest x-ray
    (B) X-ray of the humerus
    (C) Sweat chloride elevated to over
    80 mEq/L
    (D) Sweat chloride reduced to less than
    50 mEq/L
    (E) Aspergillus in the sputum
A
  1. (C) Cystic fibrosis is the most common cause of
    chronic obstructive lung disease (COLD) in children and adolescents. It is an autosomal-recessive disease that affects widespread exocrine
    glands. COLD is evident in all patients who
    survive childhood.
191
Q
  1. A 58-year-old male factory worker scheduled
    to undergo a left inguinal hernia repair is noted
    to have a severe chronic cough. Further pulmonary function tests revealed reduction of
    forced expiratory volume in 1 second (FEV1
    )
    and reduction of FEV1
    /FVC (forced vital capacity) ratio associated with emphysema. Before
    rescheduling surgery, which of the following
    would improve residual function?
    (A) Trial of ipratropium bromide
    bronchodilator therapy
    (B) Cromolyn
    (C) Cough suppressants
    (D) Bilateral carotid body resection
    (E) Intermittent positive-pressure breathing
    (IPPB)
A
  1. (A) Ipratropium bromide bronchodilator therapy will frequently improve pulmonary function in patients with COPD. Two to four
    inhalations every 4–6 hours are prescribed.
    COPD is due to emphysema (COPD type A) or
    chronic bronchitis (COPD type B). In the early
    stages, small airway dysfunction (abnormal
    closing volume) is found. As the disease proceeds, the FEV1 is reduced, then the FEV1
    /FVC
    ratio (<0.7).
192
Q
  1. The chest x-ray of a 62-year-old woman who
    complains of weakness, dyspnea, and hemoptysis shows multiple nodules in the right lung.
    She states that the dyspnea is worse in the
    supine position (platypnea) and improves on
    sitting up. On examination, the physician notes
    multiple hemorrhagic telangiectasia in the
    mouth and in the skin of the upper chest wall.
    There is a mild increase in the erythrocyte
    count, and the PO2 is 90. An angiogram shows
    multiple pulmonary arteriovenous (AV) fistula
    in both lungs. What should be the next step in
    treatment?
    (A) Needle biopsy of lesion
    (B) Irradiation
    (C) Therapeutic embolization
    (D) Endobronchial biopsy
    (E) Sympathomimetic inhalation therapy
A
  1. (C) The presence of multiple masses on a chest
    x-ray should alert the physician to the possible
    diagnosis of pulmonary AV fistula. Needle
    biopsy and endobronchial biopsy of the lesion
    should not be attempted, because severe hemorrhage may be precipitated. Paradoxical emboli,
    brain abscess, and hemothorax are recognized complications. If the fistula is localized,
    resection is undertaken; in multiple lesions, therapeutic embolization is done. In addition to
    AV malformation, multiple masses on a chest xray could be due to metastasis, granulomatous
    infection, or sarcoid or rheumatoid arthritis.
193
Q
  1. A 32-year-old male janitor complains of a
    swollen face during the past week. A CT scan
    reveals an expanding hematoma in the superior
    mediastinum. Mediastinal tamponade is most
    likely to manifest as which of the following?
    (A) Hypertension
    (B) Increased pulse pressure during
    inspiration
    (C) Paresis of the right arm
    (D) Venous congestion in the upper extremity
    (E) Hyperhidrosis
A
  1. (D) In mediastinal tamponade, hypotension,
    dyspnea, cyanosis, and a decrease in pulse
    pressure will be evident. During inspiration,
    the pulse pressure is further impeded to cause
    obstruction to transmitted ACV waves in the
    neck; in congestive cardiac failure, the ACV
    waves recorded in the neck are more prominent. Paresis of the arm is unlikely to occur,
    because the lower part of the brachial plexus
    (T1) passes along the inner border of the first
    rib to reach the neck.
194
Q
  1. After returning from vacation, a 67-year-old
    retired judge is admitted to the emergency
    department with severe dyspnea. On examination, an inspiratory stridor, ecchymosis in
    his neck, and swelling of soft tissue and veins
    in his face and upper extremity veins are evident. The CT scan shows an expanding superior mediastinal hematoma. What is the most
    common source of mediastinal hemorrhage?
    (A) Parotid gland surgery
    (B) Trauma
    (C) Dissecting thoracic aneurysm
    (D) Mediastinal tumor
    (E) Hemorrhagic diathesis
A
  1. (B) Mediastinal hemorrhage after trauma may
    result from blunt or penetrating injuries. Tamponade should be suspected if hypotension,
    cyanosis, dyspnea, and venous congestion occur.
195
Q
  1. A 42-year-old man known to have Marfan’s
    syndrome is admitted to the emergency department with severe chest pain radiating to the
    back. His blood pressure is 190/130 mm Hg. An
    electrocardiogram (ECG) shows no evidence of
    myocardial infarction. A type I (ascending aorta)
    dissecting aneurysm is detected on angiography. What should he undergo?
    (A) Percutaneous transluminal coronary
    angioplasty (PTCA)
    (B) Nitroprusside and attempted resection
    of the ascending aorta
    (C) Intra-aortic balloon pumping (IABP)
    (D) Immediate thoracotomy
    (E) Steroid administration
A
  1. (B) Dissecting thoracic aneurysm should be
    suspected in Marfan’s syndrome, pregnancy,
    bicuspid aortic valves, and coarctation. The
    onset of pain is sudden and severe and radiates
    to the back. Pulse discrepancy is frequently seen.
    The hypertension should be treated initially
    before any surgery is contemplated. The mortality rate of immediate (within 24 hours) surgical resection of the ascending aorta (type A) is
    20%; however, if surgery is delayed, the mortality is 50% at 2 weeks and 90% at 3 months.
196
Q
  1. In interpreting a follow-up x-ray to exclude
    metastatic disease in an elderly man with prostatic cancer, the radiologist reports sclerotic
    metastasis to all floating rib(s). Floating rib refers
    to ribs:
    (A) 1
    (B) 2
    (C) 3–7
    (D) 8–10
    (E) 11 and 12
A
  1. (E) The eleventh and twelfth ribs are free anteriorly and like other ribs articulate with costal
    cartilage anteriorly. However, the anterior margins of these ribs are free (float) and do not form
    part of the costal margins. The pleural reflection
    posteriorly intersects the twelfth rib. Ribs that
    articulate with the sternum are called true (ribs
    1–6 or 7); the remainder are called false ribs.
197
Q

A 36-year-old man is crossing a bridge when he is
suddenly swept by a torrent into the river. After
rescue and resuscitation, he is admitted to the ICU
of the local hospital with adult respiratory distress
syndrome (ARDS).

Which of the following associated features
would suggest a diagnosis of ARDS?
(A) High lung compliance
(B) Activation of surfactant
(C) Consolidation confined to the lingula
(D) Interstitial edema with normal
pulmonary capillary wedge pressure
(PCWP)
(E) Hypoxia responding rapidly to oxygen
therapy
A
  1. (D) In ARDS, poor alveolar gas exchange and
    interstitial edema are evident in the presence of
    normal or lowered PCWP. The changes are
    caused by damage to capillary and alveolar
    epithelial cells consequent to the release of
    proinflammatory cytokines, which, in turn, arise
    from stimulated lymphocytes and macrophages.
    Other clinical features suggestive of ARDS are
    diffuse (“fluffy”) pulmonary infiltrates and
    refractory hypoxemia (PaO2
    /FiO2 <200).
198
Q

A 36-year-old man is crossing a bridge when he is suddenly swept by a torrent into the river. After
rescue and resuscitation, he is admitted to the ICU
of the local hospital with adult respiratory distress
syndrome (ARDS).

Which is one of the most important principles
of treatment of ARDS?
(A) Steroid use
(B) Avoidance of positive end-expiratory
pressure (PEEP)
(C) Tracheobronchial toilet
(D) Use of large amount of fluids
(E) Early and vigorous use of PEEP and
highest FiO2
A
  1. (C) ARDS is caused by pulmonary or systemic
    insult. It is characterized by bilateral pulmonary infiltrates, hypoxemia, noncompliant
    lungs, and a normal or low PCWP, and steroids
    have no proved value in the management of
    ARDS, and their use may be deleterious in the
    presence of sepsis. PEEP, if required, should be
    used cautiously with the lowest FiO2 that maintains the PO2 above 60 mm Hg. Excessive fluid
    overload should be avoided. Ventilation should
    be with lower tidal volumes and decreasing
    peak airway pressures. Management strategies
    in ARDS are aimed largely at supportive care,
    maintaining tissue oxygenation and preventing
    further lung injury secondary to mechanical
    ventilation. Smaller tidal volumes are used
    (5–7 mL/kg) to prevent volutrauma as well as
    to decrease the peak inspiratory pressures.
199
Q

On his return from a 3-year visit to India, a
United Nations research worker complains of
night sweats, cough, weight loss, and tiredness. An x-ray shows an apical radiopaque
lesion (Fig. 3–1).

Several enlarged glands are palpable in the posterior triangle of the neck.

The next step toward establishing the diagnosis should involve which of the following?

(A) Determination of antitrypsin 3 level
(B) Kveim skin test
(C) Examination of sputum for cytology
(D) Thoracotomy and open-lung biopsy
(E) Sputum culture for mycobacterium
A
  1. (E) Culture for mycobacterium usually requires
    6 weeks before a diagnosis can be made. The
    clinical and x-ray features are suggestive of
    tuberculosis. The sedimentation rate is increased
    in the presence of active disease. The incidence
    of tuberculosis in the United States has
    increased in recent years. In many cases of primary infection, resolution occurs without
    symptoms. The pulmonary focus remains dormant (Ghon) and may become activated, causing caseation of the lung, formation of a
    thick-walled fibrous cavity, tuberculous bronchitis, bronchiectasis, hemoptysis, and cavitation. Sputum examined for acid-fast bacilli may
    be inadequate, because saprophytic organisms
    may be detected. The Kveim test is used in the
    differential diagnosis to exclude sarcoid.
200
Q
  1. A student with known human immunodeficiency syndrome (HIV) infection has lost 6 lb in
    weight and his sedimentation rate is increased
    to 40. He has no other symptoms. The Mantoux
    (tuberculin) test results show a change of 7 mm
    (positive), and x-ray findings reveal a small
    lesion in the apex of the right lobe of the lung.
    How should this patient be managed?
    (A) Hospitalized in a public ward
    (B) Hospitalized in an isolated hospital room
    (C) Treated as an outpatient with triple
    antituberculous drug therapy for 2 weeks
    (D) Treated as an outpatient with multiple
    antituberculous drug therapy for
    2 months and then appropriate
    antituberculous drugs for 4 more months
    (E) Observed and should undergo repeat
    skin test after 8 weeks
A
  1. (D) In the early stages of HIV infection, skin
    testing for tuberculin is intact; however, falsepositive results occur because of infection by
    nontuberculous mycobacteria and are common
    because of the immune disorder. Most patients
    with tuberculosis, who are compliant, can be
    treated on an outpatient basis.
201
Q
  1. A 12-year-old girl with leukemia develops a
    lower respiratory tract infection with hemoptysis that is shown to be due to right-sided
    bronchiectasis. In addition to treatment for the
    underlying leukemia, the patient should receive
    which of the following?
    (A) Undergo right pneumonectomy
    (B) Receive selective antibiotics,
    physiotherapy, and bronchodilator
    therapy
    (C) Undergo tracheostomy
    (D) Have cough-suppressant medication
    (E) Undergo weekly suction by endotracheal
    intubation
A
  1. (B) Bronchiectasis is caused by a congenital or
    acquired dilation of the segmental, subsegmental, or branches of the bronchi. Patients
    with B lymphocyte disorders are more likely to
    develop lower respiratory tract infection and
    bronchiectasis than those with impaired T-cell
    immunity. Treatment is aimed at obtaining
    maximal drainage. Resection may be indicated
    when the disease is localized and persistent
    symptoms and complications occur.
202
Q
  1. An 18-year-old man develops a severe cough
    with productive sputum due to Pseudomonas
    aeruginosa. He has had similar episodes in the
    past, and previous studies revealed bronchiectasis. Which of the following will help elucidate the most likely underlying cause of
    bronchiectasis?
    (A) Small-intestinal obstruction successfully
    treated at birth
    (B) Low concentration of deoxyribonucleic
    acid (DNA) in the bronchial sputum
    (C) Mycobacterium culture from sputum
    (D) Fungus grown from sputum
    (E) Immunodeficiency studies
A
  1. (A) Cystic fibrosis is an autosomal-recessive
    disorder of the exocrine glands and occurs in
    neonates who survive an episode of intestinal
    obstruction due to mucoviscidosis. Cystic fibrosis accounts for more than one-half of the cases
    of bronchiectasis seen today. Other causes
    include acute and chronic lung infections,
    humoral immunodeficiency, and localized
    bronchial obstruction (e.g., carcinoma). The
    basal segments of the lung, lingula, and right
    middle lobes are involved most frequently.
203
Q
  1. After suffering an episode of hemoptysis, a
    14-year-old boy is found, on chest x-ray, to have
    a well-circumscribed mass that contains both
    fluid and air. Surgical excision is carried out,
    and a localized mass adjacent to the carina is
    excised. What is the most likely diagnosis?
    (A) Tuberculosis
    (B) Bronchogenic carcinoma
    (C) Bronchogenic cyst
    (D) Chronic obstructive pulmonary disease
    (COPD)
    (E) AV fistula
A
  1. (C) Bronchogenic cysts are located in the mediastinum but are seen most frequently behind
    the carina. They have a thin wall, are lined by
    bronchial epithelium, and contain mucus.
    Bronchogenic cysts are usually asymptomatic,
    but symptoms may occur because of compression, with cough, wheezing, and possibly dysphagia. Bronchogenic cysts may become infected
    and rupture into surrounding organs. Cysts
    constitute 20% of mediastinal mass lesions. The
    most common mediastinal cyst is the pericardial cyst, which is found most often at the right
    costophrenic angle.
204
Q
  1. Following thoracotomy, in a 20-year-old man a
    lesion is detected in the right lower lung lobe
    and is found to be nonfunctioning lung tissue
    that is served by vessels separate from those of
    the adjacent lung tissue. What is the most likely
    diagnosis?
    (A) Mesothelioma
    (B) Hiatal hernia
    (C) Glomus tumor
    (D) Bronchopulmonary sequestration
    (E) Cystic hygroma
A
  1. (D) Bronchopulmonary sequestration can be
    differentiated from a bronchogenic cyst; in that
    it is composed of nonfunctioning lung tissue
    that is disconnected from the remaining lung;
    it has a separate blood supply. Glomus tumors
    are rare tumors that arise in the middle ear or
    jugular bulb. Patients complain of tinnitus and
    hearing loss.
205
Q
  1. A 64-year-old man complains of pain in the
    lower chest. A CT scan confirms the presence of
    a tumor of the lung at T10 level to the left of the
    midline and invading the surrounding left
    lung base. Because of the structure most likely
    involved and penetrating the diaphragm at this
    level, what could be associated?
    (A) Hoarseness
    (B) Latissimus dorsi palsy
    (C) Budd-Chiari syndrome (hepatic venous
    outlet obstruction)
    (D) Dysphagia
    (E) Tracheobronchial fistula
A
  1. (D) There are three major openings of structures that penetrate the diaphragm at differing thoracic vertebra levels. The IVC enters at
    T8 (to the right of the midline), the esophagus
    at T10 (to the left of the midline), and the aorta
    at T12 in the midline.
206
Q
  1. An 8-year-old girl with a prominent chest wall
    deformity that pushes the sternum inward (i.e.,
    in a posterior direction) is asymptomatic, and
    she participates fully in athletic activities at
    school. Surgical correction is recommended.
    What is the most likely cause of the deformity?
    (A) Funnel chest (pectus excavatum)
    (B) Pectus carinatum (protrusion at the
    sternum)
    (C) Flail chest
    (D) Cystic hygroma
    (E) Rickets
A
  1. (A) Funnel chest (Fig. 3–9) is the most important congenital chest wall deformity. It is usually present at birth, and there is marked
    asymmetry. The heart is displaced to the left.
    There is often a familial history, and associated
    congenital heart disease may frequently be
    encountered. Correction is recommended in
    asymptomatic patients with prominent deformity to avoid permanent cardiopulmonary
    changes. In flail chest, paradoxical respiration
    occurs as the chest wall deformity is sucked
    inward during inspiration. It occurs after extensive rib trauma where individual ribs are separated in two different sites.
207
Q
  1. After suffering a respiratory tract infection, a
    64-year-old female biochemist develops chronic
    lung disease requiring intubation in the ICU for
    an 8-week period. Tracheal stenosis is noted.
    What is the most likely cause of tracheal stenosis?
    (A) Prolonged intubation
    (B) Tuberculosis
    (C) Scleroderma
    (D) Riedel struma (fibrous thyroiditis)
    (E) Achalasia
A
  1. (A) Any object that compromises the blood
    supply to the tracheal mucosa or cartilage can
    cause stenosis. When the mean intramural
    pressure exceeds 20–30 mm Hg, damage may
    be anticipated. Riedel struma is a rare fibrosing
    thyroid condition, which must be differentiated from carcinoma and may cause severe tracheal stenosis. Achalasia is a neuromuscular
    defect at the lower end of the esophagus causing dysphagia, because of nonmechanical
    esophageal obstruction.
208
Q
  1. In evaluating the chest x-ray findings in a
    60-year-old man with pleural effusion, which
    of the following constitutes an abnormal finding of the pleural cavity?
    (A) Communication between the right and
    left pleural cavities
    (B) Intersection of the twelfth rib posteriorly
    (C) Existence of both a parietal and visceral
    layer in the upper parts
    (D) Existence of different attachments on the
    right and left sides
    (E) Extension of the cavity above the levels
    of the clavicles
A
  1. (A) On the left side, the pleural reflection deviates to the left, anteriorly between the fourth
    and sixth costal cartilages, to accommodate the
    cardiac notch. Although the right and left pleural reflections approach each other in the midline, there is no direct communication between
    the two sides.
209
Q
  1. Because of his involvement in a motor vehicle
    accident, a 23-year-old football player has a
    chest wall injury. The only abnormal findings
    on clinical and radiologic examination are a
    fracture of the left fifth to seventh ribs and a
    small hemothorax. What should treatment
    include?
    (A) Insertion of an intercostal drain to avoid
    pneumothorax
    (B) Thoracotomy to treat a small hemothorax
    in the left base
    (C) Insertion of a metal plate to fix the
    fracture
    (D) Administration of analgesic medication
    (E) Administration of cortisone to prevent
    callus formation
A
  1. (D) Frequently, a fracture cannot be seen on the
    chest x-ray; however, the patient should be
    treated for fracture, although none is seen on the
    x-ray. Pneumothorax may occur if more than
    one rib is involved, but a chest tube is indicated
    only if it is of substantial size or increasing in
    amount. Hemothorax usually occurs because
    of a tear in the intercostal or other intrathoracic
    vessels.
210
Q
  1. In chest surgery, which is true regarding a thoracoabdominal incision?
    (A) It should be used for most abdominal
    and thoracic procedures.
    (B) It enters the third to fifth intercostal
    space.
    (C) It causes less postoperative pain.
    (D) It allows division of the costal margin
    and the diaphragm.
    (E) It causes severe denervation of the
    anterior abdominal wall.
A
  1. (D) A thoracoabdominal incision is still used
    occasionally where access to both the upper
    abdomen and posterior thoracic structures is
    required. The main reasons for less frequent
    use of this incision are poor healing of the
    divided costal margin, postoperative pain, and
    an increased risk of infection in both the thoracic and abdominal compartments.
211
Q
  1. A rope used to elevate a heavy metal object
    breaks causing the object to fall on a 55-year-old
    factory worker and producing chest wall injury.
    Which is true of associated sternal injury?
    (A) It occurs most commonly at the work site.
    (B) It usually involves the body of the
    sternum.
    (C) It usually is vertical.
    (D) It involves the hemizygous system.
    (E) It causes miosis of the pupil owing to
    sympathetic injury.
A
  1. (B) Sternal injuries usually involve the body
    or manubriosternal junction in a transverse
    direction and frequently cause displacement.
    Sternal injuries occur most commonly as a
    result of injury by steering wheel impact in car
    accidents. It is important to exclude cardiac
    and major vessel injury in such injuries.
212
Q
  1. After undergoing an emergency operation for
    dehiscence of a colon suture line, a 62-yearold patient requires endotracheal intubation.
    Following prolonged intubation, it is noted that
    she has tracheal stenosis. What is the most
    appropriate treatment?
    (A) Administration of steroids
    (B) Resection of segment of tracheal stenosis
    (C) Irradiation
    (D) Treatment with an intrathoracic
    underwater drain if a tracheoesophageal
    fistula is present
    (E) Dilatation of the stenotic area
A
  1. (B) Most lesions of the trachea, except infiltrating adenoid cystic carcinoma, that cause tracheal stenosis should be resected when possible.
    In general, up to 50% of the trachea may be
    resected. Unequivocal postintubation stenosis is
    treated by surgical repair. Congenital tracheal
    stenosis should be treated by surgery if symptoms necessitate it. Dilatation can result in rupture of the trachea.
213
Q
  1. A 22-year-old student is scheduled to under
    go parathyroidectomy for hyperparathryoidism associated with familial multiglandular
    syndrome. His sister developed peptic ulcer
    disease secondary to a Zollinger-Ellison (hypergastrinemia) tumor of the pancreas. On examination, a swelling was noted over the posterior
    aspect of the patient’s fifth rib. What is the most
    likely finding?
    (A) Metastasis from a parathyroid carcinoma
    (B) Osteitis fibrosa cystica (brown tumor)
    and subperiosteal resorption of the
    phalanges
    (C) Dermoid cyst
    (D) Eosinophilic granuloma
    (E) Chondroma
A
  1. (B) Patients with hyperparathyroidism develop demineralization, and 1.5% shows osteitis
    fibrosa cystica. The presence of subperiosteal
    resorption of bone of the phalanges and lamina
    dura of the teeth are fairly diagnostic radiological findings of hyperparathyroidism. Chondromas account for 20% of benign tumors of
    the rib and occur at the costochondral junction.
    Osteochondromas arise from the cortex and usually occur in men. Eosinophilic granuloma
    results in a destructive lesion apparent on x-ray.
214
Q
  1. After suffering a severe bout of pneumonia, a
    46-year-old renal transplantation patient develops a lung abscess. She has been receiving
    immunosuppression therapy since her last
    kidney transplantation 3 years ago. What is the
    most appropriate treatment?
    (A) Needle aspiration
    (B) Urgent thoracotomy
    (C) Antituberculous therapy
    (D) Antibiotics and vigorous attempts to
    obtain bronchial drainage
    (E) Insertion of an intercostal pleural drain
A
  1. (D) Antibiotics and vigorous attempts to obtain
    bronchial drainage will treat the abscess adequately in the majority of cases. Lung abscesses
    commonly are associated with aspiration pneumonia, where the abscess is found posteriorly.
    In the presence of an unexplained lung abscess,
    bronchoscopy is essential to exclude a foreign
    body or tumor that could cause bronchial
    obstruction.
215
Q
  1. An 18-year-old girl developed a neck mass
    anterior to the right sternum mastoid muscle
    following a upper respiratory tract infection
    (URTI). What is most characteristic of branchial
    cleft cysts?
    (A) They usually appear in the axilla.
    (B) They may become infected after an
    URTI.
    (C) They may be traced to the stomach.
    (D) They arise from endodermal tissue.
    (E) They frequently cause brachial plexus
    lesions.
A
  1. (B) Branchial cleft cysts (Fig. 3–9) arise from
    the second and third branchial clefts. Branchial
    cysts may become evident after an URTI and
    present as a mass anterior to the sternocleidomastoid muscle. Intraoperatively, they can be
    traced to pass between the and external carotid
    artery to the piriform sinus or tonsillar fossa.
216
Q
  1. A 9-year-old boy complains of a swelling on the
    left side of his neck in the supraclavicular
    region. The swelling is translucent; a diagnosis
    of cystic hygroma (Fig. 3–2) is established.
    What is true of cystic hygroma?
    (A) It arises from sweat glands in the neck.
    (B) It is usually an anterior midline structure.
    (C) It may occur in the mediastinum.
    (D) Its lesions are usually easy to enucleate.
    (E) It is premalignant.
A
  1. (C) Cystic hygromas are relatively rare tumors.
    Most are encountered in the posterior triangle
    of the neck, but occasionally they are found in
    the mediastinum, axilla, or groin. They are
    often noted at birth and represent persistence of
    primary lymphatic buds. They extend into the
    surrounding tissues but are not associated with
    malignancy. Transillumination is a useful sign
    to diagnose this lesion.
217
Q
  1. A 48-year-old woman presents with a 6-month
    history of intermittent cranial nerve palsy that
    has become progressively worse in the past
    2 weeks. On examination, ptosis and diplopia
    are evident. Her condition shows a favorable
    response to the anticholinesterase inhibitory
    drug prostigmin (neostigmine). What is the
    most likely diagnosis?
    (A) Cerebral palsy
    (B) Pineal gland tumor
    (C) Adenoma of the pituitary
    (D) Myasthenia gravis
    (E) Tetany
A
  1. (D) Females are affected by myasthenia gravis
    twice as commonly as males. It is an autoimmune disease that produces antibodies to acetylcholine receptors. The external ocular and
    other cranial muscles are often involved at an
    early stage. There is a deficiency in acetylcholine
    receptors, and thymectomy is often helpful.
218
Q
  1. Squamous cell carcinoma of the lip is least
    likely to develop in which of the following?
    (A) Scandinavian fisherman
    (B) Redheaded pornographic actress with a
    gorgeous year-round tan
    (C) Man from Lohatchie, AL, who smokes a
    clay pipe
    (D) Brunette secretary who constantly
    drinks tea
    (E) Mentally defective man who smokes
    40 cigarettes a day and keeps the butt in
    his mouth
A
  1. (D) Squamous carcinoma of the lip comprises
    15–20% of all malignant tumors of the oral
    cavity. In approximately 30%, there is a clear
    association with heavy exposure to the sun.
    The incidence increases in those areas where
    there is more southerly latitude, the air is dry,
    and the altitude is higher.
219
Q
  1. A 43-year-old male tennis champion develops
    cancer of the lip. What is true of this condition?
    (A) It involves the upper lip in 90% of
    patients.
    (B) It is more common at the lateral
    commissure than in the middle.
    (C) It usually occurs beyond the vermilion
    border.
    (D) It results in cure in about 60% of cases.
    (E) It requires radical neck dissection.
A
  1. (C) If the lesion is treated early, patients will
    achieve a cure in most cases. The upper lip is
    involved in 10% of patients.
220
Q
  1. A 58-year-old fisherman has been heavily
    exposed to the sun for more than 30 years. He
    develops a thickened, scaly lesion extending
    over two-thirds of the lower lip. There is no
    ulceration. Histology reveals hyperkeratosis.
    What should he undergo?
    (A) Steroid ointment application three times
    daily
    (B) Antihistaminic medications
    (C) Lip stripping and resurfacing with
    mucosal advancement
    (D) Radical neck dissection
    (E) Observation and biopsy of any new
    ulcers
A
  1. (C) Hyperkeratosis of the lip is a premalignant
    lesion and usually occurs in people exposed
    excessively to the sun. The mucosa undergoes
    metaplasia to keratosquamous epithelium. The
    lip becomes pale, thin, and fragile, with cracks
    and fissures, and is covered with a white base.
221
Q
  1. A 24-year-old computer technician notes a progressive increase in the size of his left jaw. After
    x-rays are taken and a biopsy is done, a diagnosis of ameloblastoma is established. What
    should be the next step in management?
    (A) Radiotherapy
    (B) Laser beam therapy
    (C) Curettage and bone graft
    (D) Excision of lesions with 1–2 cm of normal
    mandible
    (E) Mandibulectomy with bilateral radical
    neck dissection
A
  1. (D) Ameloblastoma is a benign tumor and
    usually occurs at the junction of the body and
    ramus of the mandible. Although it is a benign
    tumor, it recurs if inadequately excised. It is
    relatively radioresistant. Histologically, odontogenic epithelium is seen in connective
    tissue stroma with extensive areas of cystic
    degeneration.
222
Q
  1. A 62-year-old man undergoes excision of a
    cylindroma of the submandibular gland. He is
    most likely to have an injury to which of the
    following?
    (A) Maxillary branch of the trigeminal nerve
    (B) Lingual nerve
    (C) Vagus nerve
    (D) Floor of the maxilla
    (E) Frontozygomatic branch of the facial
    nerve
A
  1. (B) The lingual nerve swings forward deep to
    the mylohyoid muscle and crosses twice over
    the submandibular (Wharton’s) duct. The
    mandibular branch of the facial nerve (not
    listed in the answer choices) may accidentally
    be injured below the angle of the mandible
    (Fig. 3–10). Injury to this branch causes serious facial deformity.
223
Q
  1. A 62-year-old alcoholic presents with an
    indurated ulcer, 1.5 cm in length, in the left
    lateral aspect of her tongue (not fixed to the
    alveolar ridge). There are no clinically abnormal glands palpable in the neck, and a biopsy
    of the tongue lesion reveals squamous cell carcinoma (Fig. 3–3). What should she undergo?
    (A) Chemotherapy
    (B) Local excision of the ulcer
    (C) Wide excision and left radical neck dissection
    (D) Antibiotic therapy and should be
    encouraged to stop smoking
    (E) Wide excision of ulcer and radiotherapy
A
  1. (C) Squamous cell carcinoma of the tongue frequently (40–60%) metastasizes to the lymph
    glands. Carcinoma of the tongue usually commences at the tip or side. The 5-year survival
    rate for carcinoma of the tongue is 40%, but it
    improves to 55% if lymph nodes are not
    involved.
224
Q
  1. A 59-year-old woman has discomfort in the
    posterior part of her tongue. A biopsy confirms
    that the lesion is a carcinoma. What is true in
    carcinoma of the posterior third of the tongue?
    (A) Lymphoid tissue is absent.
    (B) Lymph gland spread is often
    encountered.
    (C) There is an excellent prognosis.
    (D) The tissue is well differentiated.
    (E) The recurrent laryngeal nerve is
    infiltrated.
A
  1. (B) Carcinoma of the posterior third of the
    tongue is often detected late and carries a
    worse prognosis. Posterior-third tongue tumors
    are also called lymphoepitheliomas. These
    lesions can be poorly differentiated.
225
Q
61. Adenocarcinoma is the predominant malignant
lesion in which of the following?
(A) Hard palate
(B) Lip
(C) Anterior two-thirds of the tongue
(D) Larynx
(E) Esophagus
A
  1. (A) Tumors of the hard palate usually arise
    from the minor salivary glands. Cancers of the
    lip, tongue, esophagus, and larynx are often
    squamous cell carcinoma.
226
Q
62. The prognosis for squamous carcinoma of the
floor of the mouth is adversely affected by
which of the following (Fig. 3–4)?
(A) Poor differentiation of tumor
(B) Nonverrucous carcinoma
(C) Presence on left side
(D) No tongue involvement
(E) Keratosis of the lower lip
A
  1. (A) Poorly differentiated squamous cell carcinoma of the floor of the mouth tends to be more
    invasive than better-differentiated tumors.
    Poorer prognosis can be expected when invasion is larger than 9 mm, perineural invasion is
    noted, and lymph node metastasis is evident.
227
Q
  1. A 15-year-old immigrant from China presents
    with a mass in the left supraclavicular region.
    He is asymptomatic. Findings on endoscopy
    and biopsy show that this is a metastatic
    nasopharyngeal tumor. Clinical evidence of
    complications of this tumor would most likely
    be indicated by which of the following?
    (A) Decreased growth hormone levels
    (B) Bitemporal hemianopsia
    (C) Lateral rectus palsy
    (D) Hoarseness
    (E) Deviation of tongue to the side of lesion
A
  1. (C) Nasopharyngeal carcinoma is prevalent in
    China. It has been associated with high levels
    of Epstein-Barr virus titers. The most common
    sign is a neck mass, even when the primary
    lesion is microscopic. The first cranial nerve to
    be affected is the abducent (VI) and indicates
    cranial extension.
228
Q
  1. A 49-year-old man suffering from depression
    attempts suicide by jumping out of the window
    of his third floor apartment. He requires multiple operations during a prolonged, complicated hospital stay. Endotracheal intubation is
    attempted in the ICU but is unsuccessful
    because of tracheal stenosis, which is attributed to which of the following?
    (A) Prolonged nasotracheal intubation
    (B) Orotracheal intubation
    (C) Tracheostomy tubes
    (D) High oxygen delivery
    (E) Tracheal infection
A
  1. (A) Any object that compromises the blood
    supply to the tracheal mucosa or cartilage can
    cause stenosis. When the mean intramural pressure exceeds 20–30 mm Hg over a prolonged period, damage occurs.
229
Q
  1. A 46-year-old Texan develops a lesion in the
    vestibule of his mouth that on histological
    examination is revealed to be verrucous carcinoma of the upper aerodigestive tract. What is
    true of this lesion?
    (A) It is most commonly found on the inside
    of the cheek.
    (B) It is associated with a high metastatic
    rate.
    (C) It is ulcerating in appearance.
    (D) It is best treated with radiation.
    (E) It is more common in the northeastern
    part of the United States.
A
  1. (A) Verrucous carcinoma is a low-grade malignancy and is seen more frequently in the southern part of the United States. It is found most
    commonly on the gingival-buccal junction in
    tobacco chewers. It is grayish white and exophytic. Radiation is associated with possible
    metastases. If not excised, the lesion tends to
    invade locally.
230
Q
  1. A 16-year-old boy complains of difficulty in
    breathing through his nose. Endoscopy
    reveals a tumor infiltrating the nasopharynx.
    Histology reports this as a juvenile nasopharyngeal hemangiofibroma. The boy’s anxious
    mother requests information concerning the
    lesion. What should she be told?
    (A) It is a premalignant lesion.
    (B) It usually occurs with laryngeal
    obstruction.
    (C) It is treated with radiotherapy.
    (D) It may proceed to destroy surrounding
    bone.
    (E) It is found equally in teenaged girls and
    boys.
A
  1. (D) Juvenile nasopharyngeal hemangiofibromas are rare nonmalignant tumors containing
    both fibrous and vascular tissue. They occur
    exclusively among boys.
231
Q
  1. A 52-year-old woman has metastatic epidermoid carcinoma on the left side of her neck.
    Complete head and neck workup fails to identify the primary tumor. What is the recommended treatment?
    (A) Close follow-up monitoring until the
    primary tumor is found
    (B) Exploratory laparotomy
    (C) Radical neck dissection
    (D) Full course of radiotherapy to the head
    and neck
    (E) Combination chemotherapy using
    5-fluorouracil (5-FU), vincristine, and
    prednisone
A
  1. (C) If the neck nodes are removed, some patients
    have surgically curable primary disease. In some
    series, as many as 20% remain free of disease for
    more than 5 years without any manifestation of
    a primary tumor. It is essential to search extensively for a primary source before labeling the
    lesion as a possible branchial cleft carcinoma,
    which is extremely rare.
232
Q
  1. A 58-year-old woman undergoes excision
    biopsy of a tumor in the left posterior triangle
    of her neck. Histology suggests that this is a
    metastatic cancer. What is the most likely site of
    the primary tumor?
    (A) Ovary
    (B) Adrenal gland
    (C) Kidney
    (D) Piriform fossa
    (E) Stomach
A
  1. (D) More than 80% of neck gland tumors arise
    from structures above the clavicle. The piriform fossa is lateral to the aryepiglottic folds
    and is a major site where a primary cancer
    may remain hidden from early detection.
    Twenty percent of neck gland tumors are primary and 80% represent metastatic disease.
233
Q
  1. Arterial infusions via the external carotid artery
    with methotrexate and 5-FU for head and neck
    carcinoma have shown a 50% response rate.
    Widespread use, however, is limited. Why?
    (A) The internal carotid is inadvertently
    perfused in a large percentage of
    patients.
    (B) Ipsilateral facial slough has occurred in
    3% of patients.
    (C) Blindness occurs in 30% of patients.
    (D) The response is transient, lasting only
    2–3 months.
    (E) There is a prohibitive incidence of
    leukemia.
A
  1. (D) The response is transient, lasting only
    2–3 months. The results of this type of therapy,
    combined with those of artery occlusion, radiotherapy, or other modes of chemotherapy,
    require further evaluation.
234
Q
70. The classic complete neck dissection for palpable adenopathy in the posterior triangle of
the neck includes removal of which of the
following?
(A) The transverse process, C2–C4
(B) The spinal accessory nerve
(C) Both thyroid lobes
(D) The trapezius
(E) The vagus
A
  1. (B) The classic block dissection includes sternocleidomastoid muscle, the external and
    internal jugular veins, the spinal accessory
    nerve, the submandibular gland, and the lymphatic tissue of the lateral compartment of the
    neck. Procedures that preserve muscle, nerve,
    or vessels are called modified neck dissection.
235
Q
  1. A 69-year-old endocrinologist complains of
    progressive facial weakness and loss of taste
    sensation on the right side of her tongue. What
    is the most likely structure affected?
    (A) Lingual nerve
    (B) Middle ear
    (C) Ansa hypoglossi
    (D) Twelfth cranial nerve
    (E) Ninth cranial nerve
A
  1. (B) The chorda tympani (branch of the facial
    nerve) join the lingual nerve in the infratemporal
    fossa to supply the anterior two-thirds of the
    tongue with taste fibers (cell stations in the geniculate ganglion of the facial nerve). It also contains the secretory parasympathetic fibers to the
    submandibular and sublingual glands.
236
Q
  1. A 22-year-old female student was found to
    have an anterior mediastinal mass on a chest
    x-ray for a persistent cough. What finding is
    true regarding the thymus gland?
    (A) It is located in the posterior mediastinum.
    (B) It arises from the first branchial arch.
    (C) It controls calcium metabolism.
    (D) It is usually excised through an incision
    along the anterior branch of the
    sternomastoid.
    (E) It results in severe pneumococcal
    infection when removed in adults.
A
  1. (D) The thymus gland is removed in certain
    cases of myasthenia gravis. It is located in the
    anterior mediastinum and can be approached
    by a cervical or mediastinal approach. It arises
    from the third and fourth branchial arches.
    Pneumococcal infections are particularly likely
    to develop after splenectomy performed in
    children.
237
Q
  1. A 29-year-old woman develops difficulty in
    swallowing. Examination reveals acute pharyngitis. Which organism is most likely to be
    isolated?
    (A) Viral
    (B) Treponema
    (C) Anaerobic
    (D) Staphylococcus aureus
    (E) Escherichia coli
A
  1. (A) The most common organisms isolated in
    acute pharyngitis are streptococci, virus,
    Neisseria gonorrhoeae, and mycoplasma.
238
Q
74. A 43-year-old man suddenly develops odynophagia. Which organism is most likely to be
isolated on throat culture?
(A) Mononucleosis
(B) S. aureus
(C) Normal pharyngeal flora
(D) Group A streptococci
(E) Diphtheroid
A
  1. (C) Odynophagia is a sensation of sharp retrosternal pain on swallowing. It is usually
    caused by severe erosive conditions such as
    Candida, herpes virus, and corrosive injury following caustic ingestion.
239
Q
75. A 72-year-old man presents to the emergency
department complaining of frequent nosebleeds. What is the most likely site of acute
epistaxis?
(A) Turbinate
(B) Septum
(C) Maxillary sinus
(D) Ethmoid sinus
(E) Sphenoid sinus
A
  1. (B) The most common source of epistaxis is
    Kisselbach’s vascular plexus on the anterior
    nasal septum. Predisposing factors include
    foreign bodies, forceful nose-blowing, nosepicking, rhinitis, and deviated septum.
240
Q
  1. A 40-year-old woman is suspected of having
    a carotid body tumor. Which one of the following is most characteristic of such a tumor
    (Fig. 3–5)?
    (A) They secrete catecholamines.
    (B) They are more common at sea level.
    (C) They arise from structures that respond
    to changes in blood volume.
    (D) They arise from the structures that
    respond to changes in PO2
    (E) They are usually highly malignant
A
  1. (D) Carotid body tumor is the most common
    type of paraganglioma in the head and neck
    region, followed by the glomus jugular tumor.
    Carotid body tumor grows slowly, rarely metastasizes, and may secrete catecholamines. The
    tumor usually is supplied by the external carotid
    artery, and dissection to remove it off the carotid
    bifurcation may be difficult and cause bleeding.
    Malignancy occurs in 6% of patients.
241
Q
  1. A 32-year-old pregnant female presents with a
    1-day history of drooping of the right side of
    her face. A thorough history and physical
    examination do not reveal an obvious cause of
    the condition. What is the most likely cause of
    the patient’s facial nerve weakness?
    (A) Labyrinthitis
    (B) Parotid tumor
    (C) Lyme disease
    (D) Herpes zoster
    (E) Idiopathic
A
  1. (E) Bell’s palsy (of the facial nerve) has been
    attributed to an inflammatory condition of the
    facial nerve at the site where it exits through the
    stylomastoid foramen. Its cause remains unclear,
    and recent studies indicate a possible association with reactivation of herpes simplex virus
    in some cases. Facial paresis usually comes on
    abruptly
242
Q
  1. A 6-year-old girl complains of otalgia, fever,
    and irritability. Physical examination reveals a
    stiff, bulging, red tympanic membrane. Previous
    history of ear infections is denied. Clinical
    response to amoxicillin is maximized on which
    of the following durations?
    (A) 1 day
    (B) 5 days
    (C) 7 days
    (D) 10 days
    (E) 2 weeks
A
  1. (B) Prospective nonrandomized evaluations of
    treatment duration of acute otitis media reveal
    no difference in outcome if given over 5-day,
    7-day, or 10-day duration. However, 10-day
    treatment is indicated for children with history
    of acute otitis media within the preceding
    month.
243
Q
  1. After undergoing a minor nasal operation, a
    65-year-old man is given a neuroleptic agent.
    Which is the most commonly used neuroleptic?
    (A) Droperidol (inapsine)
    (B) Ketamine
    (C) Fentanyl
    (D) Morphine
    (E) Thiopental (tentothal) be achieved for as
    long as 4–5 minutes
A
  1. (A) Neuroleptic anesthesia is the use of agents
    that suppress psychomotor activity. Droperidol
    produces marked sedation and tranquilization.
    The onset of action is 3–10 minutes after injection, but the full effect may not be noted until
    30 minutes after injection. The sedative action
    lasts 2–4 hours. It potentiates the action of central nervous system (CNS)-depressant drugs,
    can cause hypotension, and causes mild aadrenergic blockade.
244
Q
  1. Following surgical resection of a large thyroid
    mass, a patient complains of persistent hoarseness and a weak voice. What is the most likely
    cause of these symptoms?
    (A) Traumatic intubation
    (B) Prolonged intubation
    (C) Injury to the recurrent laryngeal nerve
    (D) Injury to the superior laryngeal nerve
    (E) Scar tissue extending to the vocal cords
A
  1. (C) The most common complication of thyroid
    and parathyroid surgery is iatrogenic injury to
    the recurrent laryngeal nerve, which can result
    in temporary (up to 7.1%) or permanent (up to
    3.6%) paralysis of the vocal cord.
245
Q
  1. A 9-month-old girl is brought to the physician’s
    office for noisy breathing. The child is otherwise healthy, and her gestation and delivery
    were uncomplicated. On physical examination,
    mild inspiratory stridor is heard. What is the
    most likely cause of stridor in an infant?
    (A) Bilateral vocal cord paralysis
    (B) Laryngomalacia
    (C) Tracheal stenosis
    (D) Epiglottitis
    (E) Arnold-Chiari malformation
A
  1. (B) Laryngomalacia is characterized by inspiratory stridor and is caused by redundant
    epiglottis and aryepiglottic folds in young children. The condition usually resolves without
    surgical intervention.
246
Q
  1. A 32-year-old teacher presents at her physician’s office complaining of hearing loss in her
    right ear. Physical examination reveals cerumen completely obstructing the ear canal. Ear
    wax removal is recommended using which of
    the following?
    (A) Jet irrigation (Water Pik)
    (B) 3% hydrogen peroxide ear drops
    (C) Irrigation of the eardrum if perforated
    (D) Aqueous irrigation if a bean is present
    (E) Aqueous irrigation if an insect is present
    far in the ear
A
  1. (B) Jet irrigation (e.g., Water Pik) should
    be avoided to remove cerumen impaction.
    Detergent ear drops (such as 3% hydrogen peroxide) may be used. Aqueous irrigation should
    be avoided if organic material is present,
    because further swelling will be induced.
247
Q
  1. A 4-year-old boy requires prolonged intubation and nasogastric tube placement in an
    intensive care setting following a closed head
    injury incurred in a car accident. He develops
    recurrent fever but is hemodynamically stable.
    What is the most likely source of sepsis?
    (A) Sinusitis
    (B) Bacterial tracheitis
    (C) Epiglottitis
    (D) Small-bowel necrosis
    (E) Deep vein thrombosis (DVT)
A
  1. (A) The presence of a nasogastric tube causes
    swelling and irritation of the nasal mucosa.
    This, in turn, may partly occlude drainage of
    the sinus into the meatus.
248
Q
84. What is the most common site for foreign
bodies in the head and neck?
(A) Eye
(B) Ear
(C) Nose
(D) Throat
(E) Esophagus
A
  1. (B) Foreign bodies in the ear canal are more
    frequently encountered in children than adults.
    In general, foreign bodies in the ear are
    removed under microscopic control to avoid
    further injury.
249
Q
  1. A 25-year-old accountant is seen by her family
    practitioner for a sore throat. Her physician
    performs a Streptococcus A direct swab test
    (SADST). What is the specificity of SADST as
    compared to the standard culture method for
    the diagnosis of streptococcal pharyngitis?
    (A) 25%
    (B) 45%
    (C) 65%
    (D) 80%
    (E) 85%
A
  1. (D) Rapid testing for streptococci with latex
    agglutination (LA) antigen test is much less
    sensitive than with solid-phase enzyme-linked
    immunoassay (ELISA).
250
Q
  1. A 33-year-old female noted a discharge from a
    sinus in the overlying skin below the right angle
    of the mandible. She recalls previous episodes of
    fullness and mild pain in this region over the
    past several years. What is the most likely cause?
    (A) Thyroglossal duct cyst
    (B) Branchial cyst
    (C) Teratoma
    (D) Myeloma
    (E) Trauma to the neck
A
  1. (B) Pharyngeal (branchial arch) remnants
    account for many cysts or fistulas in the lateral
    neck. The associated tract can be found in various locations. Thyroglossal duct cysts associated
    with the decent of the thyroid gland are usually
    midline, extending as high as the base of tongue.
251
Q
  1. An 85-year-old hypertensive man is evaluated
    in the emergency department for recent onset
    epistaxis. His blood pressure is 150/80 mm Hg,
    and hematocrit is 39%. What is the most likely
    source of bleeding?
    (A) Posterior nasal septum
    (B) Anterior nasal septum
    (C) Inferior turbinate
    (D) Middle turbinate
    (E) Floor of nose
A
  1. (A) Epistaxis in children and young adults usually arises from the anterior nasal septum
    (Kiesselbach’s plexus). In elderly persons, however, spontaneous rupture of a sclerotic blood
    vessel in the posterior nasal septum is usually
    the cause of bleeding, especially in combination
    with hypertension.
252
Q
  1. An elderly man complains of ear pain. During
    evaluation, the physician asks if the patient has
    tinnitus. What is tinnitus?
    (A) A subjective sensation of noise in the
    head
    (B) A complication of chronic metal ingestion
    (C) An audible cardiac murmur
    (D) Dizziness with sounds
    (E) Nystagmus
A
  1. (A) Tinnitus is the perception of abnormal noise
    in the ear or head. It is usually attributed to a
    sensory loss; pulsatile tinnitus occurs with conductive hearing loss and is due to carotid pulsations becoming more apparent.
253
Q
  1. A 4-year-old girl is diagnosed with bilateral
    otitis media and is treated for 10 days with an
    oral broad-spectrum antibiotic. The patient
    completes the full course of antibiotics and
    returns for regular follow-up visits. In most
    children, the appearance of the tympanic membrane returns to normal following a single
    antibiotic regimen for an episode of otitis
    media within what period?
    (A) 1 week
    (B) 2 weeks
    (C) 3 weeks
    (D) 1 month
    (E) 3 months
A
  1. (E) Surgical drainage of otitis media (myringotomy) is performed in either chronic infections
    or severe infections. Complications of otitis
    media include mastoiditis and meningitis.
254
Q
  1. During an examination, the dentist notices a
    lump between the earlobe and mandible in
    6-year-old boy. It feels soft, but it is difficult to
    distinguish from the rest of the parotid gland.
    What is the most likely diagnosis?
    (A) Lymphoma
    (B) Squamous cell carcinoma
    (C) Metastatic skin cancer
    (D) Benign mixed tumor
    (E) Hemangioma
A
  1. (E) These may be difficult to excise because of the focal nerve involvement by the hemangioma.
255
Q
  1. During a baseball game, the pitcher is hit in
    the left eye with a hard-hit line drive. He is
    rushed to the nearest emergency department
    where CT scan reveals left orbital rim and floor
    fractures and fluid in the left maxillary sinus.
    What are physical findings likely to include?
    (A) Exophthalmos
    (B) Lateral diplopia
    (C) Cheek numbness
    (D) Epistaxis
    (E) Blindness
A
  1. (D) Blowout fractures of the orbit exhibit epistaxis; subcutaneous emphysema and periorbital
    ecchymosis are also frequently encountered.
    Treatment of most severe injuries is by surgical repair, often by a lower lid blepharoplasty
    incision. Enopthalmous, lateral diplopia and
    blindness do not generally occur.
256
Q
  1. A 2-year-old child undergoes tympanostomy
    tube placement for treatment of chronic bilateral serous otitis media. Which of the following
    complications is least likely to occur subsequent to surgery?
    (A) Otorrhea
    (B) Chronic perforation
    (C) Cholesteatoma
    (D) Tympanosclerosis
    (E) Scarring of the external auditory canal82. A 32-year-old teacher presents at her physician’s office complaining of hearing loss in her
    right ear. Physical examination reveals cerumen completely obstructing the ear canal. Ear
    wax removal is recommended using which of
    the following?
    (A) Jet irrigation (Water Pik)
    (B) 3% hydrogen peroxide ear drops
    (C) Irrigation of the eardrum if perforated
    (D) Aqueous irrigation if a bean is present
    (E) Aqueous irrigation if an insect is present
    far in the ear
  2. A 4-year-old boy requires prolonged intubation and nasogastric tube placement in an
    intensive care setting following a closed head
    injury incurred in a car accident. He develops
    recurrent fever but is hemodynamically stable.
    What is the most likely source of sepsis?
    (A) Sinusitis
    (B) Bacterial tracheitis
    (C) Epiglottitis
    (D) Small-bowel necrosis
    (E) Deep vein thrombosis (DVT)
  3. What is the most common site for foreign
    bodies in the head and neck?
    (A) Eye
    (B) Ear
    (C) Nose
    (D) Throat
    (E) Esophagus
  4. A 25-year-old accountant is seen by her family
    practitioner for a sore throat. Her physician
    performs a Streptococcus A direct swab test
    (SADST). What is the specificity of SADST as
    compared to the standard culture method for
    the diagnosis of streptococcal pharyngitis?
    (A) 25%
    (B) 45%
    (C) 65%
    (D) 80%
    (E) 85%
  5. A 33-year-old female noted a discharge from a
    sinus in the overlying skin below the right angle
    of the mandible. She recalls previous episodes of
    fullness and mild pain in this region over the
    past several years. What is the most likely cause?
    (A) Thyroglossal duct cyst
    (B) Branchial cyst
    (C) Teratoma
    (D) Myeloma
    (E) Trauma to the neck
  6. An 85-year-old hypertensive man is evaluated
    in the emergency department for recent onset
    epistaxis. His blood pressure is 150/80 mm Hg,
    and hematocrit is 39%. What is the most likely
    source of bleeding?
    (A) Posterior nasal septum
    (B) Anterior nasal septum
    (C) Inferior turbinate
    (D) Middle turbinate
    (E) Floor of nose
  7. An elderly man complains of ear pain. During
    evaluation, the physician asks if the patient has
    tinnitus. What is tinnitus?
    (A) A subjective sensation of noise in the
    head
    (B) A complication of chronic metal ingestion
    (C) An audible cardiac murmur
    (D) Dizziness with sounds
    (E) Nystagmus
  8. A 4-year-old girl is diagnosed with bilateral
    otitis media and is treated for 10 days with an
    oral broad-spectrum antibiotic. The patient
    completes the full course of antibiotics and
    returns for regular follow-up visits. In most
    children, the appearance of the tympanic membrane returns to normal following a single
    antibiotic regimen for an episode of otitis
    media within what period?
    (A) 1 week
    (B) 2 weeks
    (C) 3 weeks
    (D) 1 month
    (E) 3 months
  9. During an examination, the dentist notices a
    lump between the earlobe and mandible in
    6-year-old boy. It feels soft, but it is difficult to
    distinguish from the rest of the parotid gland.
    What is the most likely diagnosis?
    (A) Lymphoma
    (B) Squamous cell carcinoma
    (C) Metastatic skin cancer
    (D) Benign mixed tumor
    (E) Hemangioma
  10. During a baseball game, the pitcher is hit in
    the left eye with a hard-hit line drive. He is
    rushed to the nearest emergency department
    where CT scan reveals left orbital rim and floor
    fractures and fluid in the left maxillary sinus.
    What are physical findings likely to include?
    (A) Exophthalmos
    (B) Lateral diplopia
    (C) Cheek numbness
    (D) Epistaxis
    (E) Blindness
  11. A 2-year-old child undergoes tympanostomy
    tube placement for treatment of chronic bilateral serous otitis media. Which of the following
    complications is least likely to occur subsequent to surgery?
    (A) Otorrhea
    (B) Chronic perforation
    (C) Cholesteatoma
    (D) Tympanosclerosis
    (E) Scarring of the external auditory canal82. A 32-year-old teacher presents at her physician’s office complaining of hearing loss in her
    right ear. Physical examination reveals cerumen completely obstructing the ear canal. Ear
    wax removal is recommended using which of
    the following?
    (A) Jet irrigation (Water Pik)
    (B) 3% hydrogen peroxide ear drops
    (C) Irrigation of the eardrum if perforated
    (D) Aqueous irrigation if a bean is present
    (E) Aqueous irrigation if an insect is present
    far in the ear
  12. A 4-year-old boy requires prolonged intubation and nasogastric tube placement in an
    intensive care setting following a closed head
    injury incurred in a car accident. He develops
    recurrent fever but is hemodynamically stable.
    What is the most likely source of sepsis?
    (A) Sinusitis
    (B) Bacterial tracheitis
    (C) Epiglottitis
    (D) Small-bowel necrosis
    (E) Deep vein thrombosis (DVT)
  13. What is the most common site for foreign
    bodies in the head and neck?
    (A) Eye
    (B) Ear
    (C) Nose
    (D) Throat
    (E) Esophagus
  14. A 25-year-old accountant is seen by her family
    practitioner for a sore throat. Her physician
    performs a Streptococcus A direct swab test
    (SADST). What is the specificity of SADST as
    compared to the standard culture method for
    the diagnosis of streptococcal pharyngitis?
    (A) 25%
    (B) 45%
    (C) 65%
    (D) 80%
    (E) 85%
  15. A 33-year-old female noted a discharge from a
    sinus in the overlying skin below the right angle
    of the mandible. She recalls previous episodes of
    fullness and mild pain in this region over the
    past several years. What is the most likely cause?
    (A) Thyroglossal duct cyst
    (B) Branchial cyst
    (C) Teratoma
    (D) Myeloma
    (E) Trauma to the neck
  16. An 85-year-old hypertensive man is evaluated
    in the emergency department for recent onset
    epistaxis. His blood pressure is 150/80 mm Hg,
    and hematocrit is 39%. What is the most likely
    source of bleeding?
    (A) Posterior nasal septum
    (B) Anterior nasal septum
    (C) Inferior turbinate
    (D) Middle turbinate
    (E) Floor of nose
  17. An elderly man complains of ear pain. During
    evaluation, the physician asks if the patient has
    tinnitus. What is tinnitus?
    (A) A subjective sensation of noise in the
    head
    (B) A complication of chronic metal ingestion
    (C) An audible cardiac murmur
    (D) Dizziness with sounds
    (E) Nystagmus
  18. A 4-year-old girl is diagnosed with bilateral
    otitis media and is treated for 10 days with an
    oral broad-spectrum antibiotic. The patient
    completes the full course of antibiotics and
    returns for regular follow-up visits. In most
    children, the appearance of the tympanic membrane returns to normal following a single
    antibiotic regimen for an episode of otitis
    media within what period?
    (A) 1 week
    (B) 2 weeks
    (C) 3 weeks
    (D) 1 month
    (E) 3 months
  19. During an examination, the dentist notices a
    lump between the earlobe and mandible in
    6-year-old boy. It feels soft, but it is difficult to
    distinguish from the rest of the parotid gland.
    What is the most likely diagnosis?
    (A) Lymphoma
    (B) Squamous cell carcinoma
    (C) Metastatic skin cancer
    (D) Benign mixed tumor
    (E) Hemangioma
  20. During a baseball game, the pitcher is hit in
    the left eye with a hard-hit line drive. He is
    rushed to the nearest emergency department
    where CT scan reveals left orbital rim and floor
    fractures and fluid in the left maxillary sinus.
    What are physical findings likely to include?
    (A) Exophthalmos
    (B) Lateral diplopia
    (C) Cheek numbness
    (D) Epistaxis
    (E) Blindness
  21. A 2-year-old child undergoes tympanostomy
    tube placement for treatment of chronic bilateral serous otitis media. Which of the following
    complications is least likely to occur subsequent to surgery?
    (A) Otorrhea
    (B) Chronic perforation
    (C) Cholesteatoma
    (D) Tympanosclerosis
    (E) Scarring of the external auditory canal
A
  1. (E) Tympanostomy tube placement is performed through the external auditory canal
    with microscopic guidance. The tympanic
    membrane is directly visualized after clearing
    wax and debris from external auditory canal.
    Otorrhea is the most common sequela, requiring tube removal in 13.5% of long-term tubes and
    0.9% of short-term tubes.
257
Q
(A) Recurrent laryngeal
(B) Internal laryngeal
(C) External laryngeal
(D) Pharyngeal branch of vagus
(E) Phrenic
(F) Sympathetic
(G) Glossopharyngeal
(H) Ansa hypoglossi
  1. After undergoing a left thyroid operation, a
    42-year-old opera singer notes no change in
    speech, but she has difficulty in singing highpitched notes. Which nerve is most likely to be
    injured? SELECT ONE.
A
  1. (C) The external laryngeal nerve supplies the
    cricothyroid muscle, which assists in tensing
    the cords.
258
Q

(A) Scrofula
(B) Carotid body tumor
(C) Ganglioneuroma
(D) Virchow node
(E) Sternomastoid tumor
(F) Glomus tumor
(G) Cervical rib
(H) Sarcoid
94. A 67-year-old woman has lost weight and
complains of night sweats. She had previously
undergone treatment for tuberculosis. She has
lymph node enlargement in the neck that has
broken down to form sinus with overhanging
bluish edges. What is the diagnosis? SELECT
ONE.

A
  1. (A) Scrofula is tuberculosis lymphadenitis. It
    may occur in immunocompromised hosts as
    well as in patients from underdeveloped countries. A chest x-ray must be obtained and a
    purified protein derivative (PPD) skin test must
    be carried out. The response rate to antituberculous drugs is good, but excision of the residual lesion may be required.
259
Q
Question 95
(A) Erysipelas
(B) Eczema
(C) Scarlet fever
(D) Mucor mycosis
(E) Coccydynia
(F) Ameba
(G) Schistosomiasis
(H) Actinomycosis
(I) Tuberculosis
  1. A 63-year-old man with insulin-dependent diabetes develops a black, crusting lesion in the
    nose and left maxillary sinus. Biopsy reveals
    nonseptate hyphae, which confirms the diagnosis of what? SELECT ONE.
A
  1. (D) Mucor is an opportunistic mold that causes
    mucormycosis. At least 50% of reported cases
    are associated with uncontrolled diabetes, and
    many of the remaining patients are immunosuppressed. It appears as black crusting in the
    nose and sinuses and spreads rapidly to
    involve the cerebrum. Biopsy reveals nonseptate hyphae, which confirms the diagnosis.
    Treatment is directed toward control for diabetic ketoacidosis and use of amphotericin B.
260
Q
Question 96
(A) Cholesteatoma
(B) Dermoid cyst
(C) Glomus tumor
(D) Neurofibroma
(E) Hemangioma
(F) Epidermoid cyst
(G) Mikulicz’s lesion
(H) Sarcoma
96. This develops along lines of embryological
fusion in the floor of the mouth. SELECT ONE.
A
  1. (B) Dermoid cysts arise along line of fusion of
    embryonic parts. In the floor of the mouth, the
    swelling forces the tongue upward. Alternatively,
    the swelling may occur below the mylohyoid
    muscle, where it gives the impression of a double
    chin. It is not a premalignant lesion. It has an
    epithelial lining and may contain secretions,
    sloughed-off cells, and hair.
261
Q
Question 97
(A) Optic neuroma
(B) Constricted pupil
(C) Cerebellar dysfunction
(D) Hamartomatous polyps in the small
intestine
(E) Diverticulitis
(F) Melanosis coli
(G) Cancer of the breast
(H) Melanoma
97. A 32-year-old man presents with abdominal
pain. On examination, he is noted to have pigmented spots in the buccal region. He is diagnosed to have Peutz-Jeghers syndrome, which
also results in what? SELECT ONE.
A
  1. (D) In Peutz-Jegher syndrome, pigmented
    melanin spots are found in the buccal and perineal region. The lesions are flat and greenish
    black in the buccal region and remain after
    puberty. Pigmentation is found inside the mouth,
    nostrils, palms, and feet. Usually at about
    20–30 years of age, hamartomatous polyps are
    found in the small intestine, but other parts of
    the alimentary tract may also be involved.
    Adenocarcinoma develops in 23% of patients.
262
Q
Question 98
(A) Lymphoma
(B) Squamous cell carcinoma
(C) Metastatic skin cancer
(D) Benign mixed tumor
(E) Hemangioma
(F) Neurofibroma
(G) Paget’s disease
(H) Ranula
98. A businessman notices a lump in front of his
ear while shaving one morning. His wife thinks it has been there for several months. What is
the most likely cause of a mass in the parotid
gland in this patient? SELECT ONE.
A
  1. (D) Benign mixed tumor (pleomorphic adenoma) requires appropriate excision (superficial parotidectomy). If the tumor is shelled out,
    recurrence is likely. Approximately 80% of
    tumors of the salivary glands occur in the
    parotid gland.
263
Q

Questions 99 and 100
(A) Foramen cecum
(B) Foramen ovale
(C) Foramen rotundum
(D) Foramen spinosum
(E) Foramen magnum
(F) Foramen jugulare
(G) Foramen of Munro
(H) Foramen of Magendie
99. A 46-year-old accountant notices that he keeps
cutting the right side of his lower face while
shaving. On self-examination, he notes a loss of
sensation of the skin and lower teeth on that
side. At his physician’s office, a CT scan is
ordered. Which structure should be carefully
evaluated for this patient’s complaint? SELECT
ONE.

A
  1. (B) The trigeminal nerve exits from the foramen
    ovale to enter the infratemporal fossa. The
    motor mandibular division of cranial nerve V
    also exits through the same opening.
264
Q
(A) Foramen cecum
(B) Foramen ovale
(C) Foramen rotundum
(D) Foramen spinosum
(E) Foramen magnum
(F) Foramen jugulare
(G) Foramen of Munro
(H) Foramen of Magendie
  1. A 4-year-old boy is brought to the physician’s
    office by his father for evaluation of small
    stature. A thyroid scan is ordered and shows no
    uptake in the neck. Which structure is embryologically related to the thyroid gland and
    should be carefully evaluated? SELECT ONE
A
  1. (A) If the thyroid gland is absent from the neck,
    it may be in the lingual position at the foramen
    cecum. Excision of this lesion from the tongue
    will require thyroid hormone replacement.
265
Q
Question 101
(A) Human papillomavirus (HPV)
(B) Epstein-Barr virus
(C) HIV
(D) Varicella zoster virus
(E) Herpes type 2 virus
(F) Microcytic anemia
(G) Autoimmune deficiency
(H) Meningioma
101. Mononucleosis in the blood is associated with
what? SELECT ONE.
A
  1. (B) Nasopharyngeal cancer is most closely
    associated with Epstein-Barr virus. This virus is
    also associated with infective mononucleosis
    and Burkitt’s lymphoma.
266
Q

Question 102
(A) Mental status change
(B) Anosmia
(C) Hypopituitarism
(D) Meningitis
(E) Neck mass
(F) Deafness
(G) Bitemporal hemianopsia
(H) Neck stiffness
102. A middle-aged woman from China presents at
her physician’s office with a history of nasopharynx cancer. A medical history is obtained about
her illness. What is the most common complaint
of patients presenting with nasopharynx cancer?
SELECT ONE.

A
  1. (E) If the tumor should extend upward into the
    sphenoid bone, the cavernous sinus may be
    involved.
267
Q

Question 103
(A) Lymphoma
(B) Squamous cell carcinoma
(C) Metastatic skin cancer
(D) Benign mixed tumor
(E) Hemangioma
(F) Sebaceous cyst
(G) Sjögren’s syndrome
(H) Ectopic thyroid
103. A 63-year-old bartender presents at his physician’s office complaining of a painful sore on
his tongue. On examination, it is found that he
has an ulcerated lesion on his tongue and a
mass in the submandibular gland triangle.
What is the most likely diagnosis? SELECT
ONE.

A
  1. (B) The tip of the tongue drains into the submental lymph nodes, whereas, the side of the
    tongue drains into the submandibular lymph
    nodes.
268
Q
Question 104
(A) Funnel chest (pectus excavatum)
(B) Pectus carinatum (protrusion at the
sternum)
(C) Flail chest
(D) Cystic hygroma
(E) Rickets
(F) Sebaceous cyst
(G) Dermoid cyst
(H) Nevi
(I) Lipoma
(J) Tay-Sachs disease
104. Midline swelling causing a double chin appearance is what? SELECT ONE.
A
  1. (G) Dermoid cysts form along lines of fusion of
    embryological dermatomes of the skin. In the
    neck, they are commonly above the thyroid
    cartilage and may be classified as one of four
    varieties, central or lateral in the midline and
    above or below the mylohyoid muscle.
269
Q

Question 105
(A) Metastasis from a parathyroid carcinoma
(B) Osteitis fibrosa cystica (brown tumor)
and subperiosteal resorption of the
phalanges
(C) Atypical mycobacterium
(D) Eosinophilic granuloma
(E) Chondroma
(F) Dermoid cyst
(G) Thyroglossal duct cyst
(H) Laryngocele
(I) Warthin’s tumor
105. A 5-year-old boy is taken to his pediatrician for
a laceration on his right knee. A mass on his
neck is noticed; his mother states it has been
there for several months and is slowly getting
larger. The mass is slightly to the left of midline.
Ultrasound findings are shown in Fig. 3–6. What
is the most likely diagnosis? SELECT ONE

A
  1. (G) The thyroid develops from the foramen
    cecum in the tongue and descends to its definitive position in the neck. Failure of the tract to
    close may result in a thyroglossal duct cyst.
270
Q
  1. A 60-year-old woman, complaining of joint
    pains and muscle ache has a normal physical
    examination. Her routine blood work reveals a
    normal hemoglobin (Hb). Serum potassium is
    4 mEq/L. BUN and creatine are normal. Serum
    calcium is 11–12 mgm/dL. Which one of the
    following is not associated with this condition?
    (A) Myoglobin in the urine
    (B) Serum, parathyroid hormones (PTH) is
    elevated
    (C) Increased urinary excretion of calcium
    (D) Pancreatic tumors may be present
    (E) Pituitary tumors
A
  1. (A) An elevated ionic calcium and elevated
    blood levels of intact PTH clinches the diagnosis
    of hyperparathyroid. Primary hyperparathyroidism can result in bone demineralization and
    renal calculi if neglected. Hyperparathyroidism
    may be manifestation of the multiple endocrine
    neoplasia syndrome. Pancreatic islet cell tumors,
    vi2 gastinomas and insulinomas, and pituitary
    tumors are part of the multiple endocrine neoplasia MEAN 1 syndrome.
    Myoglobin in the urine is present in patients
    with rhabodmyolysis and is not associated
    with elevated serum calcium, and is after associated with hyperkalemia.
271
Q
  1. A 5-year-old girl presents with difficulty
    breathing. On examination, of the oral cavity a
    3-cm mass is found in the midline on the posterior aspect of the tongue. The most likely
    diagnosis is:
    (A) Lingual tonsil
    (B) Lingual thyroid
    (C) Foreign body stuck to the tongue
    (D) Dermoid
    (E) Angioneurotic edema
A
  1. (B) An ectopic thyroid gland can be located
    anywhere from the base of the tongue to the
    mediastinum. It often results in the failure of
    descent resulting in either a linginal thyroid or
    ectopic thyroid in the midline of the neck. The
    so called lateral aberrant thyroid is usually a
    metastasis from papillary carcinoma of the
    thyroid.
    Angioneurotic edema is an acute allergic
    reaction, which causes a sudden swelling of
    the whole tongue with airway obstruction.
272
Q
  1. A 40-year-old woman presents with weight
    loss, palpitations, and exopthalmos. On physical examination, the thyroid gland is diffusely
    enlarged. Blood tests reveal primary hyperthyroidism. Which one of the following is not
    the treatment of hyperthyroidism?
    (A) Methimazoli
    (B) Lugols iodine
    (C) I131
    (D) Subtotal thyroidectomy
    (E) Steroids
A
  1. (E) Hyperthyroidism could be diffuse primary
    hyperthyroidisms, Graves’ disease, or a toxic
    nodular goiter. Graves’ disease is an autoimmune hyperthyroidism. The treatment consists
    of medical management with use of antithyroid
    drugs such as methimazole, or ablation of the
    gland with radioactive I131, or surgically with
    subtotal thyroidectomy. Failure of medical
    management requires oblation procedures
    either with I131 or surgery.
    Lugols iodine is used in preparation to surgery. Steroids are not used in the treatment of
    hyperthyroidism. It may be used in the management of thyroid storm, a life threatening
    condition.
273
Q
109. Which one of the following is not part of the
management of a patient with hyperparathyroidism
(A) Hydration with intravenous normal
saline
(B) Steroids
(C) Exploration of the neck for
parathyroidectomy
(D) Parathyroid scan
(E) Vitamin D
A
  1. (B) Definitive treatment of hyperparathyroidism is parathyroidectomy. When serum calcium is above 14 mgm/dL the patient is in hypercalcemic crisis. Immediate treatment of
    this condition requires hydration with normal
    saline and use of diuretics to bring down the serum calcium level.
    A positive parathyroid scan will help to
    locate an adenoma of the parathyroid preoperatively.